Sei sulla pagina 1di 262

I n t r o d u c t i o n to

Quantitative Techniques

. . .

--

--

I T M
UNIVERSITY

ONLINE

- -

I n t r o d u c t i o n to Q u a n t i t a t i v e T e c h n i q u e s

Table

eBook

of Content

1.

Business Mathematics and

Statistics

1.1

Introduction

1.2

Uses of Mathematics and

1.3

Applications of Statistics

1.4

Limitations of Statistics

10

1.5

Functions of Variables

11

1.6

Progression of Series

13

1. 7

Calculus

18

1.8

Chapter Summary

28

2.

Matrices and

7
Statistics

8
9

29

Its Application

2.1

Introduction

30

2.2

Definition of Matrices

31

2.3

Basic Types of Matrices

32

2.4

Basic Operations of Matrices

35

2.5

Main Operations of Matrices: Transpose of a Matrix

44

2.6

Main Operations of Matrices:

45

2. 7

Properties of Determinant

47

2.8

Applications of Determinant

49

2. 9

Inverse of a Matrix

56

2.10

Chapter Summary

61

3.

Determinant

Measures of Central Tendency

62

3.1

Introduction

63

3.2

Central Tendency or Average

64

3.3

Objectives of Averaging

64

3.4

Characteristics of a Good Average

65

3.5

Various Measures of Central Tendency

66

3.6

Arithmetic Mean

67

3.7

Geometric Mean

77

3.8

Harmonic Mean

80

3.9

Median

81

3.10

Mode

85

3.11

Chapter Summary

89

www.itmuniversityonline.org

Page 2

Introduction to Q u a n t i t a t i v e T e c h n i q u e s

Table

eBook

of Content

4.

Measures of Dispersion

90

4.1

Introduction

4.2

Meaning

4.3

Significance of Measures of Dispersion

92

4.4

Types of Dispersion

93

4.5

Choice of a Suitable Measure of Dispersion

114

4.6

Chapter Summary

115

5.

and

91
Definition

92

116

Correlation

117

5.1

Introduction

5.2

Correlation:

5.3

Significance of Correlation

5.4

Types of Correlation

5.5

Method of F i n d i n g

5.6

Karl

5.7

Chapter Summary

Meaning and

118

Definition

119
120

Correlation

using

123

Scatter Diagram

128

Pearson's Coefficient of Correlation

133

134
6.

Rank Correlation and

Regression Analysis
135

6.1

Introduction

6. 2

Spearman's Rank Correlation

6. 3

Regression Analysis

6.4

Applications of Regression Analysis

6. 5

Types of Regression

6. 6

Obtaining

6. 7

Assumptions of Linear Regression

6.8

Coefficient of Determination

6. 9

Properties of Coefficient of Determination

6.10

M u l t i p l e Regression

6.11

Chapter Summary

136
142
142
144
144
Regression

Lines

152
153
154

157
163

www. itmu niversityonl i ne.org

Page 3

I n t r o d u c t i o n to Q u a n t i t a t i v e T e c h n i q u e s

Table

eBook

of Content

7.

Business Forecasting and Time Series Analysis

164

7.1

Introduction

165

7.2

Time Series

166

7.3

Secular Trend

167

7.4

Seasonal Variation

169

7.5

Cyclical Variation

170

7.6

Random or Irregular Variation

171

7.7

Trend

172

7.8

Mathematical

7.9

Decomposition of a Time Series

174

7.10

Analysis of Time Series

184

7.11

Coding

189

7.12

Chapter Summary

193

Concept of Probability

194

8.

Measurement Methods
Model of Time Series

173

of Time Variable

8.1

Introduction

195

8.2

Basic Terminologies

196

8.3

Definitions of Probability

198

8.4

Basics of Probability

201

8. 5

Operations of Events

205

8.6

Probability

Laws

207

8. 7

Bayes' Theorem

221

8.8

Applications of Bayes' Theorem

225

8.9

Applications of Probability

228

8.10

Chapter Summary

229

9.

Probability Distribution:

230

Binomial

9.1

Introduction

231

9.2

Basic Terminologies

232

9.3

Probability

234

9.4

Expectation and

9.5

Binomial Distribution

237

9.6

Properties and Applications of B i n o m i a l Distribution

238

9.7

Mean and Variance of Binomial Distribution

239

9.8

Fitting

242

9.9

Chapter Summary

Functions

235

Variance

a Binomial

www.itmuniversityonline.org

Distribution

248

Page 4

Introduction to Q u a n t i t a t i v e T e c h n i q u e s

Table

eBook

of Content

10. Probability Distribution:

Poisson and

Normal

249

10.1

Introduction

250

10.2

Poisson

251

10.3

Applications of Poisson

10.4

Normal Distribution

257

10.5

Applications of Normal Distribution

260

10.6

Graph of Normal

260

10. 7

How to

10.8

Chapter Summary

Distribution

Find

Distribution

253

Distribution

the Area u n d e r a Normal Curve

www. itmu niversityonl i ne.org

261
268

Page

Business Mathematics
a n d Statistics

I T M

Introduction to Q u a n t i t a t i v e T e c h n i q u e s

01.

Business

Mathematics a n d

eBook

Statistics

1 . 1 Introduction
Every

organization

activities.
skills
to

to

This

collect

information

analyze

ultimately

and

form.

the

and
The

huge

amount

this

information,

organization's

making

manager to
graphical

In

this

is an

they

strive

to

transformation

integral

understand

decision-making.

you
You

managers

transform

process

while

requires

long-term

the techniques of gathering

will

learn

will

also

the

operating

the

make

qualitative
various

hence

useful

lot

methods

it

is

decisions

of

scope

information

decision-making

and

and

into

steps

important

will

basics

of
of

information,

data

that

tabulation,

quantitative
mathematics

techniques
for

in

business

better and

effective

statistics in

be able to:

List the uses of mathematics and

Identify the advantages of mathematics and

List the applications of statistics

List the limitations of statistics

Recognize various functions of variables and

Describe calculus and

statistics
statistics in

business

progression of series

its fundamental operations

www.itmuniversityonline.org

for every

business.

you

to

problems.

interpretation of results. T h i s chapter introduces the role of mathematics and

After reading t h i s chapter,

daily

With the requisite


effective

have

its

their interpretations.

importance

study

can

Managers

that

part of management

representation, data analysis and

chapter,

information

efficiency.

help managers to deal with short-term and

Decision

of

is useful for various levels of management.

interpret

increase

information

quantitative

generates

Page 7

Introduction to Quantitative T e c h n i q u e s

01.

Business

M a t h e m a t i c s and

eBook

Statistics

1 . 2 Uses of Mathematics and Statistics


Mathematics
skills
and

of an

and

individual.

financial

mathematical
queuing

Businesses
based
of

take

play

an

company.

statistical

theories,

time

of

also

logical

series

in the field

principles.

analysis,

They first

and

important
Modern
such

linear

rational

present

model.

algorithms,

thinking,

reasoning,

role

in

the

probability

programming,

They

strategies,

decisions

their complex
then

and

use

using

decision

theory,

systematic, and

theory,

theories

are

scientific

problems

modelling
solve

applied

also

other sciences.

various

to

use

decision

These

making

methods

accounting,

immensely

etc.

mathematical

analytical

statistical techniques facilitate rational,

by

problem-solving

operational,

organizations

as

and

of economics, social science, e n g i n ee r i n g , and

meaningful

mathematical

numerical
and

and

enhance

They

decisions

theory,

widely used

statistics

research

in

the form

techniques

them.

like

Mathematical

scientific decisions.

Advantages of Mathematics and Statistics

Mathematical

knowledge

helps

to

record

and

analyze

accounting

transactions

and

also to make financial decisions.

Mathematics

and

such as h o l d i n g

Mathematical
compound

statistics

help

in

calculation

costs, work-in-progress costs,

formulae

interest,

help

to

simplify

amortization

associated

calculations
tax

Business statistics aids in decision m a k i n g .

Statistics is useful for regular and

repetitive data.

It

of

payable,

payroll calculation,

sales forecasting, financial analysis,

cost

with

inventories

sales, distribution costs, etc.

the

schedules,

accounts a u d i t i n g , discount calculation,

of

is

simple

credit

interest,

mortgages,

etc.

used

in marketing

research,

production and operations management,

trend

analysis, etc.

Statistical

tools

distributions,

like

correlation,

regression,

help in simplifying day-to-day

www.itmuniversityonline.org

probability,

binomial

and

normal

problems of functional management.

Page 8

Introduction to Q u a n t i t a t i v e T e c h n i q u e s

01.

Business

Mathematics a n d

eBook

Statistics

1 . 3 Applications of Statistics
Statistics

plays

an

decision-making.
requirements

and

It

in

lot of data and

important
is

order

role

important
to

achieve

in
to

various
use

the

techniques

to

obtain

optimal

to

appropriate

desired

take decisions accordingly,

fields

results.

they

solutions.

need

facilitate

statistical

As

analysis

of

data

methods

as

per

managers

have

to

deal

to apply various statistical

Below

are

some

basic

for
the

with

methods

applications

of

statistics in an array of fields.

Statistics and State

Almost

all

countries

educational
knowledge
tabulated

use

ministry
of

the

statistical

can

type

solve

of

statistical data

decisions

can

techniques.
research,
good
The

and

be

etc.

human

only

problem

of

illiteracy

law

child
rate,

and

welfare.

education,
social

For example,

if

they

implications,

have
etc.

the

prior

well

Management

through

be

the

applied

plays a vital

statistical

resource

maintain

helps governments in better decision-making.

can

Statistics

relevant

to

business depends majorly on

taken

Statistics

the

population,

Statistics in Business and

The success of every

tools

data

department

to

in

role

systematic
financial
in

uses

application

analysis,

marketing

position

also

effective managerial decisions;

and

sales

a product,

strategize the

statistical

of

statistical

to

in

the

manage

and

market

it fulfils the

product

information

tools

forecasting,

as

these

need

of

market.

employee

performance and a p p r a i sa l .

Statistics in Business Analytics

Business

analytics

is

type

of

insights into the understanding

business

process

that

of new and emerging

applies

statistics

to

develop

new

b u s i n e s s opportunities.

Statistics in Operations Research

The

operations

solutions

to

research

complex

statistical data.
by a nation and

field

uses

problems.

This data

All

mathematical
national

and

modelling

and

organizational

helps to get clarity on the

internal and

algorithms
affairs

external

rely

to

obtain

deeply

problems faced

this helps to reach a suitable solution.

www.itmuniversityonline.org

on

Page 9

Introduction to Quantitative T e c h n i q u e s

01.

Business

M a t h e m a t i c s and

eBook

Statistics

1 . 4 L i m i t a t i o n s of Statistics
Statistics is useful
are

applied

statistical

with

Statistics

are

it

Data

is

example,
country

in

not
a

and

Sometimes,

cannot

be

the

conclusions

implemented.

For

drawn

this

from

reason,

one

be

directly

applied

on

the

to

terms

directly

in

that

the

can

study

be

measured

quantitatively.

of qualitative

characteristics

intelligence, etc.

is

not

basis

measured

quantitatively.

of marks obtained

by

However,

students

in

an

intelligence

examination

or

the degree of intelligence.

when
from

it

when

it

relates

relates
to

statistical

an

to

the

individual

point

of

view,

measurement
item
the

of

or event as
population

masses
a

is

not

separate entity.

For

figure

but

for

it

particular

in a given year is useless.

Statistical
are

applied

intelligence

statistical

statistical,

practical

efficiency,

be studied

for ranking

be

example,

may

not

cannot

like honesty,

For

understanding.

the limitations of statistics.

can

However,

many fields but statistical solutions are appropriate only when they

proper

analys i s

must understand

in

laws are true o n l y

universally true;

coin

tossing

on

an

average.

they are true only

experiment,

expect

The conclusions obtained

under certain

50/o

head

conditions.

and

50/o

statistically

For example,

tail,

only

if

the

experiment is carried out with a fair coin.

The

most

Misuse

of

significant
statistics

limitation
may

of statistics

arise

due

to

is

that

it

incomplete

can

be

best

information

used
or

by

experts.

inappropriate

application of the techniques.

www.itmuniversityonline.org

Page

10

Introduction to Q u a n t i t a t i v e T e c h n i q u e s

01.

Business

Mathematics a n d

eBook

Statistics

1 . 5 Functions of Variables
Functions are a convenient way of stating
in

finance

underlying

and

economics

are

studied

relationships among
by

constructing

variables.

mathematical

Many questions
models

of

the

phenomenon.

A function expresses the relationship of one variable or a group of variables with another
variable,

by

range.

representation

between

associating

"X",

relationship

every
of

this

representing

is

causal,

is

member

the

type,

"Y

cause,

termed

as

in

the

f(X)",

and

an

domain
where

"Y",

with
"f"

unique

represents

representing

independent

variable

its
and

member
the

is

as

the

relationship

effect.
Y

in

When

termed

the
as

3.

If

values

of

dependent variable.

For

example,

different
variable

variable

values
Y are

t a b u l a r and

of

is

variable

obtained.

By

related
X

are

to

substituted

obtaining

graphical form as in

Fig.

variable

these

and

in

the

values,

is

represented

function,

the

corresponding

function

can

be

SX

expressed

in

1.sa.

-1

-2

-2

-7

2
1

-2

-
-6

-
-12

Fig.

1.Sa:

Straight Line Equation

Functions of Many Variables

A function may be of many variables, where there may

be more than one independent

variable that determines the value of a dependent variable Y.


That is, Y = f(x1, x2

...

x,)

www.itmuniversityonline.org

Page

11

Introduction to Quantitative T e c h n i q u e s

01.

Business M a t h e m a t i c s and

eBook

Statistics

Linear Functions

It is represented a s :

aa +

a2x2 +

a1x1 +

variables.

...

+ anXn, where ao, a1, a2 . . . an are constants and x1, x2 ... Xn are

In a two-dimensional

space,

a linear function

represents a straight

line and

is

u s u a l l y written as, Y = a + mX.

Inverse Functions

It

is

function

functions,
be

that

expresses

variables

and

it is possible to express X in terms of Y,

uniquely

determined.

equal to f inverse of Y.

This

is

denoted

by

in

opposite

directions.

In

these

so that the given value of Y and X can


1

" X = f- (Y)",

The important aspect of an

which

can

inverse function

be

read

is that

as,

is

it gives back

the o r i g i n a l value of the variable.

For example,

there is a function,

y = f(x)

= x + 3,

if x =

2 then the value of "y" can

be

obtained as follows:
y = f(x = 2) = 2 + 3 = 5
1

The inverse function of the same will be,


x = r '( y = 5)

Exponential

s -

3 = 2. Thus, r'(y)

= f- (f(x = 2 ) )

if y = 5 then,

= 2.

Function

The exponential function

is defined

independent variable.

e has a fixed

function

determine

is

x = f- (y) = y - 3,

used

to

as y

e'.

Here,

"e" is the

base, whereas, x"

is an

value, approximately 2 . 7 1 8 2 8 1 8 2 8 . The exponential

the

relation,

such

that

constant

change

in

the

independent variable causes a proportionate c h a n g e in the dependent variable.

Logarithmic Function

It

is a function

(x=a' or x=e'),

(as y =

log, x or y =

In x)

that

is the

inverse of an

exponential

so that the independent variable appears in a logarithm.

is read as y is the logarithm of x to the base "b".

So, a log

If

function

y = log bx,

is simply the inverse function

of an exponential function.

www.itmuniversityonline.org

it

Page 12

Introduction to Q u a n t i t a t i v e T e c h n i q u e s

01.

B u s i n e s s Mathematics a n d

Some standard

eBook

Statistics

properties of logarithmic and exponential functions are:

Logarithmic Function

In e ;
log, a

log

Exponential Function

In e ' ;
exey

; 1

= ex+y

( e')'; e''

ax; x
0

log,(h x g)

Xe=

loq,

log, (h/g)

log

1.Sa:

log, h + log, g

loq,

ex/eY=exy

log, h - log, g

x ; logb x
logb a

Table

Standard

Properties of Logarithmic and

Exponential

Functions

1 . 6 Progression of Series
Two

special

progression

types
and

of

sequence

geometric

of

numbers

progression.

formulae in accounting and

finance.

These

used

in

mathematics

sequences

Sequence and

help

to

are

arithmetic

understand

series are also a p p l i ed

various

to commercial

problems.

Sequence

a, . . .

is an

ordered

is a sequence.

value

is called

the natural

the

collection

According

term

or

of numbers or objects.

to the definite

element of the

rule of law,
sequence,

...

nth

and

a1,

a2,

a3,

a, . . . ,

"a," has a definite value,


corresponding

to any

that

value of

number "n".

A finite sequence like a,, a,, a, . . . , a, is denoted


the

For example,

element of the sequence and

is denoted

by {ar},

For example, the sequence

i ;

1,

an

2 ... ,

{-n-}

by "{a,}", i ;

1,

2 . . . , n where "a," is

infinite sequence is given as a 1, a2, a3,

00

a, . . . an

or simply by {a.}.

is 1/2, 2/3, 3/4 . . .

n+l

www.itmuniversityonline.org

Page 13

Introduction to Quantitative T e c h n i q u e s

01.

Business M a t h e m a t i c s and

While a series

sum

of the

eBook

Statistics

is an expression of the type a1 + a , + a3 + a, + . . .

elements

of a

sequence

{a,},

number of elements, it is called a finite series;

1,

else,

... ,

00 .

If a

+ an +

series

. . . which

is the

contains a

finite

it is called an infinite series.

Arithmetic Progression

A sequence like a,, a,, a3 . . . , an is called an arithmetic progression, when (a, - a,)
(a3 - a,)

= ...

= (an - an-1)

abbreviated as A . P .

= d T h i s means, an arithmetic progression, which is

is a sequence in which each term is obtained

by a d d i n g a constant

"d" to the preceding term. d is the common difference of arithmetic progression.


Therefore, the n" term of an arithmetic progression is given by tn = a , +

the sum of n terms of an arithmetic progression is given by

Sn=

n (first term

Sn

=;

(n

- l)d and

(2a1 + ( n - l)d) or

+ last term)/2.

Example 0 1 :

If the fifth and twelfth term of an arithmetic progression is 14 and

35, respectively.

Find

the arithmetic progression.

Solution O 1 :

Let "a" be the first term and "d" be the common difference of arithmetic

progression.

Therefore, we can write:


For fifth term,

n = 5,

For twelfth term,

(2)

- ( 1 ) gives,

n =

7d

t s = a + 4d

1 2 , 112 = a +

14
lld

(1)
= 35

(2)

= 21

That is, d = 3

Putting the value of d in ( 1 ) gives:


a =

14 - (4

x 3)

= 2

Therefore, the required arithmetic progression is 2,

www.itmuniversityonline.org

5, 8,

11

...

Page 14

Introduction to Q u a n t i t a t i v e T e c h n i q u e s

01.

Business

Mathematics a n d

eBook

Statistics

Example 0 2 :

Divide 69

into three parts, which are in Arithmetic Progression

(A.P.),

such that the

product of the first two parts is 483.

Solution

02:

Let the second

part of A . P i s represented

with the differenced


(a - d )
=>

3a

+ a +

(a+

are

d)

(a

- d) and

(a

by a, then the first part and

third

part of A . P

+ d).

= 69

= 69

=> a = 23

The three parts are 23

- d,

23,

23

+ d

Since the product of first two parts is 483,


23(23

- d)

then:

= 483
483

=>

23 - d =

= 21
23

=>

23

- d

=> d =

= 21

23

- 21

= 2

Therefore, the three parts in A . P .

are 2 1 ,

23, and

25

Example 0 3 :

The first and the last term of an A . P .


terms is 7 1 7 1 .

Find

are -4 and

146,

respectively. The sum of the

the number of terms.

Solution 0 3 :

Let "n" be the number of terms in a series.


The first term of the series is a,
S, =
=>

=>

n (first term

7171

n(-4

n = 7171x2
142

+
+

-4

and the last term

is T, =

146.

last term)/2
146)/2

= 14342

=lOl

142

The number of terms in a series is

www.itmuniversityonline.org

101

Page

15

Introduction to Quantitative T e c h n i q u e s

01.

Business

M a t h e m a t i c s and

eBook

Statistics

Example 04:

Find

the last term of the A . P . ;

Solution

0.6,

1.2,

1.8 ...

to

13 terms.

04:

The first term

is a,

= O. 6

The number of terms is n =


The common difference=
The last term t13 = a,
= 0.6

13

1.2

- 0.6

= 0.6

12d

x 0.6

12

= 0 . 6 + 7 . 2
= 7.8

Geometric Progression

If each

term

multiplier,

in

the

a sequence
sequence

is obtained

is called

The constant multiplier is called

"r"

by

multiplying

a geometric

the common

the

progression,

the

common

ratio

0-

is given as, t,

If r =

If r

= arC

then,

sequence

Common

ratio

is

-6,

18,

a( 1 - rn )

-54

.
rs

Here,

term/first term

term/second term

the

ratio

is

ar"

is

as

if "a" is the first term


in

G.P.

and

the

nth

a( rn

is given as follows:

- 1)

...

constant;

1
...

9,

this

www.itmuniversityonline.org

2 7 , 81

X'

series

is

known

as

geometric

se r i e s .

Page

G.P.
and

term

r - 1

,
3

Third

... ,

is abbreviated

(-;) = -3

Another example for G. P.

Here, second

ar

by a constant

Sn

1 - r

in 2,

ar,

term

na

For example,

a,

The sum of "n" terms in G . P .

J.

1, then S, =

1, then

the

which

ratio. Therefore,
2

is

preceding

16

tn

Introduction to Q u a n t i t a t i v e T e c h n i q u e s

01.

Business

Mathematics a n d

eBook

Statistics

Example 0 5 :

Which term of the progression

Solution

The n" term of G. P.

8 ...

is 256?

2-

1,

r ; 4/2

; 2, and

t,

; 256

series ; t, ; ar""

n-1

n - 1 ;
Thus,

2, 4,

05:

In the given series, a ;

256

1,

n ;

9 implies that the ninth term of the geometric progression

is 256.

Example 0 6 :

Find the sum of n terms of the series 3,

33,

333 ... , n

Solution 0 6 :

The G . P .

series is 3,

Here, a ; 3 and

r ;

33,

333

... , n

11

rn - 1
The sum of n terms of G . P .

Sn

a(

r - 1

Example 0 7 :

Find three numbers in G . P .

Solution

whose sum

is 39 and

product is 7 2 9 .

07:

a
r

Let the three numbers be

a,

ar.

a
According

to the question,

=> a' ; 729

.-.

; 9

x a x ar;

729

9
So the numbers are

r ,

9, 9r

Again

+ 9 + 9 r ; 39

www.itmuniversityonline.org

Page

17

Introduction to Quantitative T e c h n i q u e s

01.

Business

M a t h e m a t i c s and

eBook

Statistics

.2.
+ 9r = 39 - 9

=>

.2.
+ 9r = 30

=>

=>

9 + 9r

=>

9r

c>

9r

=>

9r(r - 3)

=>

(r - 3)

= 30r

30r + 9 = O

27r - 3r +

- 3(r - 3)

(9r - 3)

r = 3,

or

= 0

= O

3
:.

9 = 0

r =

3
9

So, the numbers are

r,

9 , 9r = 3,

9,

27.

1 . 7 Calculus
In

the

present

management

techniques

to

marketing,

information technology,

fundamental
and

solve

business

scenario,

operations

in

problems.

calculus.

etc.

there

is

growing

Calculus-based
Limits,

t ec h n i q u e s

derivatives, and

Their theories

need

primarily

for

are

quantitative

widely

used

in

integration are the most

depend

on

the

idea

of limit

continuity of function.

Calculus
calculus

has

two

creates

variable.

On

main

branches:

changes

the

other

in

differential

function,

hand,

integral

with

ca l c u l u s
respect

ca l c u l u s

is

and
to

integral

unit

calculus.

change

associated

with

in

its

finding

Differential
independent

the

function

when the rate of c h a n g e is given.

Limits
Limits

give

us

the

also
"L",
as,

as f(x)

comes
then

it

nearer
can

limf(x) = L ,

of

dependent

function

"f(x)"

approach a particular value say "a".

variable tends to

is defined

value

2x.
to

be

If x is

0.

said

If x
that

w h e r e x --, a

given

values that

approaches
the
is

to

limit
be

value

of f(x)
read

are

as

approaches

f(x)
L.

x approaches

an

For example,

nearer to
and

when

0,

then

independent

a function

the

approaches

value of f(x)

some

This

is

a.

x approaches

If

number

symbolically

written
a

x-ee

value,

it

is written

as,

lim f(x) = L

,_

www.itmuniversityonline.org

f(x)

Page

18

larger

Introduction to Q u a n t i t a t i v e T e c h n i q u e s

01.

B u s i n e s s Mathematics a n d

eBook

Statistics

Rules of Limit of a Function:

The
is

l i m i t of a sum

equal

to

(or difference) of two functions with same independent variable

the

sum

(or

difference)

of

Mathematically for two functions f(x) and


lim

{f(x) g (x)}

x-a

lim f(x) lim g (x)


x-a

g(x)

limits

of

the

two

functions.

the above rule can be explained as:

x-a

The l i m i t of the product of two functions is equal to the product of their limits.
That is,

lim {f(x)

g (x)} = lim f(x)

x-a

the

The

x-a

limit of the

quotient

lim g (x)
x-a

of two functions

is equal

to

the quotient

of their

limits,

provided the limit of the divisor is not zero.

.
That

rs,

{ f(x)}

l i rn

!im

)
(

x-a

f(x)

I'

rm

lim g(x) t

x-+a

x-a

The l i m i t of a constant is equal to that constant.

The

limit of the

n"

power of any function

is equal to the

n"

power of the

l i m i t of

the function.

That is,

Jim {f(x)}"
x-a

= ( lim f(x))"

x-a

Example 0 8 :

Find

lim
M -+ - 2

Solution

Jim
lt-+

-2

4
,- x'
2X

)
3

08:

4-x'

2x2 + x3

=
)

Jim

[(2 - xX2 + x))

4-2

x'(2+x)

lim ((2 - x))


X--4-2

x2

((2 - (-2)))

(-2)'

www.itmuniversityonline.org

Page 19

Introduction to Quantitative T e c h n i q u e s

01.

Business

M a t h e m a t i c s and

eBook

Statistics

Example 0 9 :

Find

the value of lim

x-o

e' - e - ) .
x

Solution 0 9 :

-1-

(e'

l i rn

e-x

lJ

by a d d i n g

and

subtracting

1 in the numerator

X -+ 0

.
((e'
=l1m

- 1) - ce-

xO

-1))

1
= l i m (<e'
xO

> J - l i m (ce-

xO

>J

= 0 - 0 = 0

In certain cases,

l i m i t may have some forms as infinity;

form. The infinite form

Example

Find

.
lirn

x -+ <0

infinite

in the next example.

1- x

1- x

+
2

... +

1- x

J.

x
2

1- x

10:

L = lim [

Let,

is called

10:

the value of:

Solution

is explained

such a form

x -e ec

1- x

1- x

. . . + _x-J

3
2

1- x

1 - x

1
L =

(1 + 2 + 3 . . . + x)

lim
X .-) <0 1 - X

Since,

1, 2, 3 ... , x forms arithmetic progression series, the sum of first x natural

numbers is given as follows:

x(x + 1)

s,

1 + 2 + 3 ... + x =
2

Putting

L =

lim

this value in

lim

it becomes:

x(x + 1)

X -----Jo co 1 - X 2

L =

L,

x -e so (1 + x)(l - x)

x x(x + l)
2

www.itmuniversityonline.org

lim

,__, ( 1 - x)

x -"2

Page 20

Introduction to Q u a n t i t a t i v e T e c h n i q u e s

01.

B u s i n e s s Mathematics a n d

_!_ lim

.!. lim

'""' ( 1 - x)

_
l
_

'""'

eBook

Statistics

_!_ _ 1

.!.x_l_
2

= _ _!_

0 - 1

lim

'""' 1 - x

1- x

Example

...

= --

1- x

1- x

11:
2

.
Evaluate

(x

Sx + 6)

l l rn

X - 2

X -+ 2

Solution

11:

When x = 2 the function becomes undefined as 2 - 2 = 0. The division by zero is not


mathematically defined.

.
S o,I

(x

x-2

x -,i, 2

lim

.
1 rm

-Sx+6)

im

(x

,..,,

3)

(x-2)(x-3)
--

x - 2

x -+ 2

2 - 3

-1

Derivative

derivative

input.

In

is

defined

finance and

as,

the

economics,

rate

the

of

change

rate

in

of change

function

is

called

with

change

marginal

or

in

the

incremental

change.

For example, the rate of change of the total cost of capital gives the m a r g i n a l cost of the
capital.
in

the

change

This
value

is due to the
of a

function

per unit change in


is

useful

in y for a small change in

in

x is

the

studying
referred

new capital
the

raised.

behavior

of

The

rate of c h a n g e

function.

to as the derivative of y, with

Thus,

the

respect to

x.

The derivative of y = f(x)

with

respect to

x is u s u a l l y denoted

by

dx

of f(x)

or f'(x)

dy
dx

The derivative of a function f(x) at point a is defined as

lim f(x) - f(a)


--,.,

www.itmuniversityonline.org

= f'(a)

x - a

Page 2 1

or

Introduction to Quantitative T e c h n i q u e s

01.

Business M a t h e m a t i c s and

eBook

Statistics

There are two basic rules of differentiation:

Polynomial
o

Functions

Derivative of a constant

Let y =

N, where "N" is a constant value, then

For example, if y = 2, then

is

always

= 0 ,

= = 0

it means that the derivative of a constant

zero.

Derivative of a power function

Let y =

Nx', where N and "a" are constants, t h e n :

dy = N x
dx

= Nx

dx

= Nax-

dx

For example, if y = 2x

3,

then

dy

= (2)(3)x

1
-

= 6x

dx

Derivative of a sum

(or difference) of two or more functions

The relation is shown a s :


d
du
dv
-(UV)=-dx

dx

dx

Algebraic Functions
o

Derivative of product of two functions

Let, y = uv
Where,

f(x)

and

g(x)

are

differentiable

functions

of

x,

then:

dy
dv
du
- = U - + V dx

dx

dx

Derivative of a quotient of two functions


u

Let, y = - , v

v
Where, u = f(x) and v = g(x) are differentiable functions of x, t h e n :
du
dy

dv

v dx - u dx

==
dx

v'

Derivative of the

Let,

y =

number,

u",

where

nth

power of a function
=

f(x)

is a differentiable function

of x and

"n" is

t h e n :

www.itmuniversityonline.org

Page 22

any

Introduction to Q u a n t i t a t i v e T e c h n i q u e s

01.

B u s i n e s s Mathematics a n d

dy = un
dx

eBook

Statistics

= nun-l u = nf(x)n-l f(x)

dx

dx

dx

Higher Order Derivatives

Derivatives of a higher order can be taken by repeating

the process. The second and


3

third derivatives are written as

f"(x)or

d ' ; , f'"(x)

or

dx

If f(x)

= l n x , then f(x)

dx

The derivatives of different functions are listed

below:

1
-

If f(x)

= e,c,>, then f'(x)

If f(x)

In g ( x ) ,

= g'(x)e'C,)

then f'(x)

g'(x)
g(x)

Some standard formulae used

For Logarithmic and

in derivative are:

Exponential
For Trigonometric Functions

Functions

d
1
-lnx = dx
x

d
- s i n x = cosx
dx

d
1
-log
x =
dx
a
xlna

-COSX=-SlnX

d
dx

.i..ex ::: ex

d t a n x = sec' x

dx
d
-ax

d
=

ax I n a

-cot x

dx

-cosec x

secx tanx

dx
d

_.!.._ eax = aeax

- se c x

dx

dx
d
-cosecx = - cosecx cotx
dx
Table

1.7a:

www.itmuniversityonline.org

Standard

Formulae for Derivative

Page 23

Introduction to Quantitative T e c h n i q u e s

01.

Business

Example

M a t h e m a t i c s and

eBook

Statistics

12:

Find

the value of

Solution
2

+ 3xy + y = 0.

12:

Differentiating

[x' d
(y') +

with

v'

dy +2xy

respect to x :

d
(x'J] + [3x d
d
x (y) + 3y d
d
x (x)] + :

+3xdy + 3 y x l + d y

dx

(2yx

for the equation x y

+ 3xy + y = O

x y

2yx

dx

= O

= 0

dx

+ 3x + 1) dy + 2xy

+ 3y = 0

dx

dy

- (2xy

=
dx

2yx

+ 3y)

+ 3x + 1

Example 1 3 :

Find

d'y
the value of - when

Solution

Given,

13:

log [ ::

From Table

1. Sb,

Differentiating

dy

[x2J

y = log -

dx

log x

loge'

= 2 log

l o g . [ ) = log. h - log. g

both sides of y =

2 log

x - x loge

log. x'

x - x , with

= 2

log x - x

= clog. x ,

and

log. e'

= x log. e

respect to x:

= 2 - 1

dx

Again, differentiating

with

respect to x, we

have:

d y
dx

_.! ( d y )
dx

dx

_.! ( 2 x dx

)-

_.!(1)
dx

www.itmuniversityonline.org

-2x-

= -
2

Page 24

Introduction to Q u a n t i t a t i v e T e c h n i q u e s

01.

B u s i n e s s Mathematics a n d

eBook

Statistics

Integral Calculus

The derivative of a function f(x)

That is,

ddx f(x)

by f'(x)

= f' (x)

Then, the integration of f'(x)

f f'(x)dx

is denoted

is:

= f(x) + C , where, "C" is a constant of integration and

the function f(x)

is called

the integral.

Definite Integral

If (x)

is an integral of f(x), then

( b ) - (a)

is called the definite integral of f(x),


b

between the limits "a" and "b".

It is denoted

by the symbol

f(x)dx, where, a is the

lower limit and

Some

Properties of Integration

Integration of sum or difference of functions

b is the upper limit of integral.

{f(x) g(x) )dx =

f f(x)dx

f g(x)dx

Integration by substitution

f f(g(x))g'

(x)dx =

By substituting

f f(u)du

the value of g(x)

= u,

g'(x)dx=du

the

integral can

be easily

solved.
o

Integration by parts

f f(x)g(x)dx

= f(x)f g ( x )d x -

www.itmuniversityonline.org

f t'(x)(f

g(x)dx)dx

Page 25

Introduction to Quantitative T e c h n i q u e s

01.

Business M a t h e m a t i c s and

eBook

Statistics

Some standard formulae used

in integration are:

For Logarithmic and Exponential


For Trigonometric Functions
Functions

ff'

(x)dx = f(x) + C

xn+1

f x"dx

sin xdx = - cos x + C

[cos xdx , s i n x + C

=--+C
n + l

adx = a x

xn+l

f x-"dx

+ C

f tanxdx

= lsecxl + C

f secxdx

= ltanx + sec x i +

- n + l

exdx

ex +

Sin

d
x
X X = z -

sin2x
+ C
4

b'

f b" dx

Jtan

xdx

tanx- x + C

xdx

tanx + c

= In b + C

J sec
= In x + C

J cosec
J sec
Table

Example

Evaluate

, (x

xtanxdx

-cotx + C

secx + C

Formulae for Integration

3)
+

dx

+ 4x + 3

14:

3)

Standard

14:

Solution

1.7b:

xdx

(x +
x'+4x+3

dx

=J

3)
(x +
(x+l)(x+3)

www.itmuniversityonline.org

dx

1-d
x

J-

= log(x + 1)

+ C

x + l

Page 26

Introduction to Q u a n t i t a t i v e T e c h n i q u e s

01.

B u s i n e s s Mathematics a n d

Example

15:

Evaluate

eBook

Statistics

dx
1 - sinx

Solution

15:

f
=

x 1 + sin x dx
(Multiply and divide by (1

1 - sin x

l+sinxdx

l-sin

((1

f[

f [sec'

(1

(cos

x = (1

- sin

sin x
X

+ cos

+ sinx))

x))

1,

xrx

x + sec x tan x}:lx

1
sinx
(secx = -- and t a n x = --)
cosx

- sinx)(l

cos2

- sin' x)

=fl+sinxdx

cos

+ sin x) term)

1 + sin x

cosx

secx + tanx + C

www.itmuniversityonline.org

Page 27

Introduction to Quantitative T e c h n i q u e s

01.

Business

M a t h e m a t i c s and

eBook

Statistics

1 . 8 Chapter S u m m a r y

Mathematical and

statistical techniques facilitate

rational,

systematic, and

scientific

decisions.

Data

is statistical when

statistical when

it

relates to the measurement of masses.

it relates to an

Statistically obtained

individual

It is not

item or event as a separate entity.

conclusions are not universally true. They are only true u n d e r

certain conditions.

Statistics has many applications in the fields of business management, economics,


physical, and

natural sciences.

A function expresses the relationship of one variable or a group of variables with


another variable,

by associating

every member in the domain with a u n i q u e

member in the range.

An ordered

According
term

collection of numbers

like

a1, a , a3, a4 . . . , an
2

is a sequence.

to definite rule of law there is a definite value of a,, which

is called

the

or element of the sequence and corresponds to any value of the natural

number n.

A sequence like

a2 - a 1

a1, a2, a3, a4 . . . , an

a3 - a2

= ... =

an - an-1

is called an arithmetic progression,

d , where d

when

is the common difference of

arithmetic progression.

If each term

in a sequence is obtained

constant m u l t i p l i e r ,
abbreviated

the sequence is called

a geometric progression,

by a

which

is

as G. P.

Derivative is defined

as the rate of change in function with change in the i n p u t .

Thus, derivative of a function f(x)


gives f(x).

by multiplying the preceding term

is denoted

Therefore, derivative and

www.itmuniversityonline.org

by f'(x), w h i l e by

integration are reversible

integrating

f'(x)

processes.

Page 28

Matrices a n d
Its A p p l i c a t i o n

I T M

I n t r o d u c t i o n to Q u a n t i t a t i v e T e c h n i q u e s

02.

Matrices a n d

eBook

Its Application

2 . 1 Introduction
Knowledge

of

mathematics.
industrial

the total

matrices
It

is

has

used

management.

been

in
For

found

genetics,
example,

to

be

of

economics,
in

an

great

utility

sociology,

industrial

in

many

modern

scenario,

requirement of materials for a particular product and

psychology,

matrix

then,

branches

is

used

t
o

of

and
find

give a solution for

the best possible cost-effective option.

In

the

real

world,

problems

Mathematical operations and


using

the

matrix algebra.

can

be

more

results can

The

complex

and

be expressed

involve

more

in elegant and

use of matrices makes the solution

than

two

factors.

compact forms

of a system

of

by

linear

equations easier to find.

This

chapter

introduces

the

systematic

and

compact

arrangement

of

numerical

information in the form of matrices.

After reading this chapter, you will be able to:

Define

matrices

Identify types of matrices

Describe the basic operations of matrices

E x p l a i n main operations of matrices: Transpose and

State properties of determinant

State applications of determinant

Compute Inverse of a Matrix

www. itmu niversityonline.o rg

Determinant

Page 30

Introduction to Quantitative T e c h n i q u e s

02.

M a t r i c e s and

eBook

Its A p p l i c a t i o n

2 . 2 Definition of Matrices
A matrix can

be defined

arrangement.
formation

along

presentation

as an ordered

matrix
"m"

is

rows

system

and

is called an "rn

by

"n"

set of numbers

of

"m

columns

n"

in

a predefined

numbers

bounded

by

arranged
a

bracket

two-dimensional
in

[].

rectangular

This

type

of

n" matrix, which is written as "rnx n" matrix.

a,2
A =

Here, the numbers in a matrix


element of subscript
compact

form,

2, . . . m and j
etc.

The

indicates a

the above

1,

2, ... n.

index

notation

shows

placed.

T h is

the

to

and

notation

is

amn

and

are called

the

second

represented

by

its entries or elements. The first

element

Am x n

indicates a column.

or A

by capital

[a,1];

where,

letters

like A,

is

designate

the

lower

used

in

ii'"

index

element

indicates

"Tensor

is

"a",

the

Analysis".

where

column

the

where

Sometimes,

super

A =

[8

1,

B,

C,

the

(upper)

element

braces

{}

parentheses () are also used to denote matrices.

For example,

In

s m a l l letters represent elements of the matrix.

used

row

matrix

row

. . . ,

Matrices as a whole, are denoted

while corresponding

other

a11,a12

3]

9 3]
Alternatively, it can be written as

www.itmuniversityonline.org

A=[;

Page 3 1

is

and

I n t r o d u c t i o n to Q u a n t i t a t i v e T e c h n i q u e s

02.

Matrices a n d

Example

eBook

Its A p p l i c a t i o n

01:

Construct a 3 x 4 matrix, whose elements are a,

= 2i

- j.

Solution O 1 :

The general 3 x 4

We have, a,1
a
a

11

12

matrix is g i v e n

2i

- j,

1,

2,

by

A=[::

::

a31

a32

1,

3, 4

3 and j

2,

= 2(1) - 1 = 1
= 2(1) - 2 = 0

a13

=2(1)-3=-1

a14

= 2(1) - 4 = -2

a21 = 2(2) - 1 = 3
a22

a23

= 2(2) - 3 = 1

2(2) - 2

a24 = 2(2) - 4 = 0

Similarly,

Therefore,

31

= 5, a

32

= 4, a

the required

33

= 3, a

matrix is:

= 2.

34

A=[

-l

-{]

2 . 3 Basic Types of Matrices


There are a number of different types of matrices.
the

matrix

classifies

them.

Classification

The

also depends

number of columns and


on

the entries.

rows

The following

in

are

the different kinds of matrices:

Row Matrix or Vector

A matrix
matrix
can

For

having

only one

is of the orderl x n .

be expressed

example,

[2

as:

row and

number of columns

It consists of a single row,

A = [a

11

6]

any

and

[3

12

13

4]

is called

a row

with elements along

matrix.

n columns.

It

14]

are

row

matrices

of

order

1x3

and

respectively.

www. itmu niversityonline.o rg

A row

Page 32

1x4,

Introduction to Quantitative T e c h n i q u e s

02.

M a t r i c e s and

Column

eBook

Its A p p l i c a t i o n

Matrix or Vector

A matrix having

any n u m b e r of rows and

order

matrix

of the

mx 1

and

only one column

consisting

of

single

is called

column,

a column matrix.

with

elements

It is

along

rows.

a11

a21

For example,

A column
as:

{a11

it can

matrix

be expressed as

A =

is usually expressed

a 21

by writing

the elements horizontally,

within

braces

amd

5
2
For example,

is a column matrix of order 4 x 1

4
6

Square

A matrix
mxn

Matrix or Vector

having

an equal

matrix, for which

number of rows and

m =

n,

that is,

columns

number of r o w s =

is

known as a square matrix.

number of c o l u m n s ,

An

is a square

matrix.

For example,

A=

Zero

A =

[!

is a square matrix of order 2 x 2

41

-2

- 7

is a square matrix of order 3 x 3

Matrix or Null

Matrix

If all the elements of a matrix are zero,

then such a matrix

is called

a zero matrix or n u l l

matrix.

For example,

A =

is a zero or n u l l matrix of order 3 x 2

www.itmuniversityonline.org

Page 33

I n t r o d u c t i o n to Q u a n t i t a t i v e T e c h n i q u e s

02.

Matrices a n d

Diagonal

In

Matrix

square

diagonal

eBook

Its A p p l i c a t i o n

matrix,

matrix.

This

[a,j].

matrix

If

all

the

consists

non-diagonal

of top-left to

elements

bottom-right

are

zero,

then

or top-right

to

it

is

bottom

left diagonals.

T h i s matrix is written as

o
ol
3
0

Scalar Matrix

If in a diagonal matrix,
equal,

then

all

non-diagonal elements are zero and

it is a scalar matrix. That

For example,

is,

11

= a

22

= . . . = am,

] i s a scalar matrix of order 3 x 3

all d i a g o n a l elements are

= b

and a,1

= 0, for all i

j.

1{
The

matrices

and

1:1

]are

scalar

matrices

of

order

one

and

2x2

and

3x3,

11

respectively.

Identity

square

Matrix or Unit Matrix

matrix,

in

elements are zero,

is termed

is a scalar quantity)

For example,

I,

upper

is called

= [

Upper Triangular

A square

which

matrix,

triangular

triangular matrix,

all

as an

diagonal
identity

matrix.

are

A scalar matrix
is denoted

all

with

non-diagonal

1 (where k

k =

by "I,".

Matrix

all of whose elements


matrix.
if a,1

A = [

elements

a u n i t or identity matrix and

Alternatively,
O for all

a13

For example,

the

:i:

a23

below the
a

square

leading
matrix

diagonal are zero,


A

[a,j]

is

is called

called

an

i > j.

l
is an upper t r i a n g u l a r matrix.

a,,

www. itmu niversityonline.o rg

Page 34

an

upper

Introduction to Quantitative T e c h n i q u e s

02.

M a t r i c e s and

Lower Triangular

A square

eBook

Its A p p l i c a t i o n

matrix,

Matrix

all

of whose elements above the

lower t r i a n g u l a r matrix.

Square

matrix

[a,i]

leading

is called

diagonal

are zero,

is called

lower t r i a n g u l a r matrix,

if a,1

= O f o r a l l i < j .

0
For example,

-5

] is a lower triangular matrix.

2.4 Basic Operations of Matrices


There

are

number

of

operations

that

can

be

applied

to

modify

matrices.

The

basic

operations are:

Addition of matrices

Subtraction of matrices

Multiplication of a matrix by a scalar

Multiplication of matrices

Addition of Matrices

If "A" and

"B" are two

matrices of the same order,

the matrix where each element

is the sum

mathematical terms,

[a,i]

for all

The

and

of the corresponding

B =

[b,1 1,

then A +

B",

is defined

elements of A and

B =

[a,1]

[b,i]

number

of

B.

[a,1 +

as
In

b,11,

i and j.

definition

That is,
Then,

when A =

then their s u m , "A

of

addition

of

matrices

can

if there are three matrices, A +

be

extended

to

finite

matrices.

B + C.

A + B + C = ( A + B ) + C

Familiar

[a11 +

b1Jl

[a, +
1

b,1 + c,i]

properties

commutative,

if,

Then, A +

[a,1 +

B =
=

of
=

[C11]

numbers
[a11]

and

extend
B

[b11]

to

these

are

operations

matrices

of

the

of

matrices.

same

order,

Addition
say,

b;j]

[b,1 + a,1]

= B + A

www.itmuniversityonline.org

Page 35

is

mxn.

I n t r o d u c t i o n to Q u a n t i t a t i v e T e c h n i q u e s

02.

Matrices a n d

For example,

eBook

Its Application

A = [:

and

B = [!

A+B=[:

]+[!

]=[:::

B + A = [ !

]+[:

]-[!::

:]=[190

:]-[ 1 9 0

!]
!]

. . A + B = B + A

Suppose for any three matrices A =


Then,

(A+

B) + C = [a,J + b1Jl

B = [b1Jl, and C=

[C1Jl, each of order m x n :

+ [c,j]

[a1J + b1J + C1J]

[a1J]

= A +

[a1Jl,

+ [ b1J + C1J]
(B

+ C)

From the above explanation, the addition is associative, that is, the matrix sum does not
depend on the order of the matrix.

Zero is the "additive identity" for matrix addition.

is the matrix of order m x n and O is an m x n zero matrix, then A +

Example

Add

A =

O = O + A.

01:

the following:

3
-2

6
5

-3

OJ
3

, B =

l-2
7

Solution O 1 :

Adding two matrices, A and

-2)
A + B =

6 + 1

0+4]

5 + 2

3+3

-3+(-4)

2 + 1

- 2 + 7
[3+(
3 + 4

A+B=l

_;7

www. itmu niversityonline.o rg

Page 36

If A

Introduction to Quantitative T e c h n i q u e s

02.

M a t r i c e s and

eBook

Its A p p l i c a t i o n

Subtraction of Matrices

If A =

[a,i]

and

B =

are two matrices of the same order,

m x n , then the difference of

D, where, D = [d1il-

A - B =

In other words,
That is,

[b,il

D = A - B = A +

(-1)

the sum of the matrix A and

B.

matrix - B .

Example 0 2 :

-1

I f A = 4

[1

2]

5 , B = S

- 1

Then, verify A +

(B - C)

- 3]

(A+

[4
, and

C ;

B)

- C

Solution 0 2 :

A +

(B

- C)

[;

3+(-3)
4 + 5

[
2+0

A + (B - C) = [

(A+

B)

:1

- 1

- C =
[[

www.itmuniversityonline.org

Page 37

I n t r o d u c t i o n to Q u a n t i t a t i v e T e c h n i q u e s

02.

Matrices a n d

eBook

Its Application

- 1 + 2

[[3 + 1
4 +S
2 + 1

1
2 + (-3)]]
[4
5+2
- 0

2 + 0
0+(-1)

3 + 1

3
-2

1 - 1
-1-2]

[4-4

9 - 0

2 - 3

7 - 2

3 - 1

-1-(-2)

4-3

0
(A+ B)- C

- 1

= [

-t] ..................................

(2)

Therefore, from equation ( 1 ) and

(2), A +

(B - C)

= (A+

B)

- C

Multiplication of a Matrix by a Scalar

If a

matrix

which

is

multiplied

is a real

number.

by

scalar

Therefore,

( n u l l matrix), the following

quantity,

kA =

[kaq],

then

each

element

is

multiplied

For conformable matrices A,

B,

by

C, and

laws of ordinary algebra are satisfied.

Commutative

Associative

Distributive

Identity

Law

Law

Law

Law

A + B = B + A

Example

Find x and

A +

(B+C)

= (A+B)

+ C

k(A + B) =

kA +

kB

A + Z = A

03:

v,

if,

x + y = [

and

x - y = [

Solution 0 3 :

We have,

x + y = [

and

x - y = [

A d d i n g two matrices
1;
2x = [

X = [

(Divide both the side by 2)

www. itmu niversityonline.o rg

"k",

Page 38

Introduction to Quantitative T e c h n i q u e s

02.

M a t r i c e s and

eBook

Its A p p l i c a t i o n

Putting the value of x in the first matrix

] + y = [;

y = [;

]-[

= [

] = [

= [

Multiplication of Matrices

Multiplication

of two

matrices

is defined

only

is the same as the number of rows of the

if the

number of columns

of the

left matrix

right matrix.

The multiplication of matrices follows the given steps


Step

1:

compatibility

Step

2:

operations of multiplication

Let there
B =

[b,1]

of B.
is,

be two matrices, A and


is defined

Therefore,

only

the

if the

number of columns of A are

matrices A and

if A is of the order m x p and

m x n matrix;

B. The product AB of the two matrices A = [a

where,

C =

[c,i]

B are

compatible,

B i s of the order

for i =

pxn,

equal to

with

the

respect

then the

to

j]

and

number of rows

product AB.

product A B =

C,

That

which

is

1, 2, 3, . . . m and j = 1, 2, 3, . . . n, and c,1 is given by:


p

c;i

a,1b1i

+ a; 2 b 2 i + . . . + a;pbpi

L a;kbki

k=l
So, to construct the element

row

of

with

the

c,,

of the matrix product AB,

multiply the elements of the i'"

of

the

i'"

running

the

left

right finger to

run

down the i" column of the matrix

corresponding

elements

column

of

B.

The

sum

of

these

products gives the elements of AB.

The

product can

A and,
Then,

be easily computed

simultaneously,
multiply

the

using

the

elements

in

by

the

corresponding

index finger across the

position

and

add

the

;'"

row of

successive

products.

For example,

if A = [a

11

a21

a,

a22

13]

a23

and
2x3

B =

b11

b,2

b21

b22

[b

www.itmuniversityonline.org

b
31

32

B.

3x2

Page 39

I n t r o d u c t i o n to Q u a n t i t a t i v e T e c h n i q u e s

eBook

02. Matrices a n d Its A p p l i c a t i o n

b12
b22

l
w i l l be a 2 x 2 matrix

b32

Let,

A B = [c,,
C21

c12]
C22

2x2

Where,
c11 =

(1'' row of AJ

x (1'' column of BJ

= a11b11 + a12b21 + a13b31

c12 =

( 1 " row of A)

x (2"' column of BJ

= au b12 + a12b22 + a13b32

c21 =

(2"' row of A)

x ( 1 " column of B)

c22 =

(2"'

x (2"' c o l u m n of B)

row of A)

= a 21 b 11 + a 22 b21 + a 23 b 31
= a 2 1 b 1 2 + a 22 b 22 + a

Matrix multiplication also satisfies the following

23b32

properties, where A,

B,

and

Care

matrices.

Associative

Distributive

Identity

Multiplicative

Law

Law

Law

Inverse

A (BC)

In general,

A ( B + C)

(AB)C

matrix multiplication

= AB

+ AC

A I =

IA

AA-

is not commutative, that is, A B *

= A- A = I

BA.

2
For example,

if A =

!] ::

www. itmu niversityonline.o rg

and

B = [-\

-o

-1

Page 40

Introduction to Quantitative T e c h n i q u e s

02.

M a t r i c e s and

eBook

Its A p p l i c a t i o n

[o

[o

21[-:1]

[1

AB=

21[:o:J

[1

3][-:1]

[2

3][o:J

[2

41[-:1]

4][o:J

[Oxl+lx(-1)+2x2
AB=

Ox(-2)+1x0+2x(-1)]

lx1+2x(-1)+3x2

lx(-2)+2x0+3x(-1)

2x1+3x(-1)+4x2

2x(-2)+3x0+4x(-1)

[3
=

- 2]
-5

-8

Another example for matrix multiplication

If,

A =

- 2

1
AB=

[3

1]
and

B =

114

2]

-2

-4

[4

2]

-4

3 x 4 + l x 3 + l x 2

3x6+lxO+lx(-4)

3 x 2 + l x l + l x 3

2x4+(-2)x3+4x2

2x6+(-2)x0+4x(-4)

2x2+(-2)xl+4x3

Sx6+2x0+7x(-4)

S x 2 + 2 x 1 + 7 x 3

[
S x 4 + 2 x 3 + 7 x 2

1 2 + 3 + 2

18+0+(-4)

6 + 1 + 3 ]

8+(-6)+8

12+0+(-16)

4+(-2)+12

30+0+(-28)

10+2+21

[17

14

10]

10

-4

14

40

33

[
20+6+14

www.itmuniversityonline.org

Page 4 1

I n t r o d u c t i o n to Q u a n t i t a t i v e T e c h n i q u e s

02.

Matrices a n d

Example

Its Application

eBook

04:

- 4
Find the value of x and

y,

if

Solution 04:

First, find the product of the

4
[-

2
3
- 3

left matrices.

312]
[- 4 x 2 + 2 x x + 3 x 5 ]
5 X =
5 x 2 + 3 x x + S x 5
1

2 x 2 + (-3) x x + 1 x 5

8+2x+15]
=

10+3x+25
[-

4 - 3x + 5

Equate the resultant matrix with the right matrix.

i: :: : !] = [;s]
[

4 - 3x + 5

-8+2x+15= 9
=> 2x = 9 - 7
=>

x = 1

4-3x+5= y
=> 4 - 3 x 1 + 5

[Substituting the value of x]

=> 9 - 3 = y
=> y = 6

:. x = 1 and

y = 6

www. itmu niversityonline.o rg

Page 42

Introduction to Quantitative T e c h n i q u e s

02.

M a t r i c e s and

eBook

Its A p p l i c a t i o n

Example 0 5 :

florist

wants

combination
flower used

of

to

roses

First bouquet

of

price

carnations

of
(CJ,

two

types

and

lilies

of

bouquets.

(LJ.

The

Each

numbers

bouquet

of each

type

SJ

each

rose

is

4,

each

carnation

is

2.50,

and

each

lily

is

5.

Use

a
of

matrix

multiplication to determine how much shall each bouquet cost.

Solution 0 5 :

The matrix for the cost of each flower

4 ]
Cost=

is

[6

Second b o u q u e t

cost

(RJ,

the

in the two bouquets are shown in the following matrix.


R

The

decide

Rose(RJ

2.5

Carnation(CJ

Lily (LJ

The flowers cost matrix is of ( 3 x t )

The matrix for the n u m b e r of flowers in each bouquet


R

First bouquet
Second b o u q u e t

[6

SJ

The above matrix is of ( 2 x 3 J

The cost for the two bouquets is calculated

4
First bouquet

Second b o u q u e t [ a

4J

2. 5

Second b o u q u e t

First bouquet
Second b o u q u e t

Rose (RJ
t(CJ

arna

ion

Lily (L)

First bouquet

by multiplication of matrices as follows:

[6x4 + 7 x 2 . 5 + SxSJ
8x4 + 6 x 2 . 5 + 4x5

[66.SJ
67

Thus, the cost of the first bouquet is 66. 5 and that of the second

www.itmuniversityonline.org

bouquet is 67.

Page 43

I n t r o d u c t i o n to Q u a n t i t a t i v e T e c h n i q u e s

02.

Matrices a n d

eBook

Its Application

2 . 5 M a i n Operations of M a t r i c e s :
A matrix obtained
matrix" and

Suppose,

Transpose of a Matrix

by the interchange of rows and columns is known as a "transpose of a

is denoted

by A'.

A=[::

::

::]

a31

a32

a 33

After interchanging the rows and columns,

it is represented as:

A'=[::
a13

For example,

Therefore,

A'

A=[

= [

Some Special

Matrices:

Symmetric Matrix

The

square

Therefore,
is,

[ a,il

matrix

A is a "symmetric matrix" if

interchanging

transpose

is the

same

matrix.

columns does not change the matrix form, that

[a,,].

For example,

A =

6]

rows and

its

[5
and

A'=

3 6]

are symmetric matrices.

Skew Symmetric Matrix

The matrix A is a "skew


is

the

[a,1]

same

+ [a1

,J

as

the

(j,

symmetric" if [a,
1]
i)'"

element

with

= -[a1,]
the

sign

where,

the (i, j)

changed.

From

1"

element of A

the

definition,

= O. The diagonal elements of skew symmetric matrix are zero.

- 4 3]
For example,

A = [

-3

www. itmu niversityonline.o rg

-5

is a skew symmetric matrix.

Page 44

Introduction to Quantitative T e c h n i q u e s

02.

M a t r i c e s and

eBook

Its A p p l i c a t i o n

2 . 6 M a i n Operations of M a t r i c e s :
The

value

then

of a

matrix

is

represented

its determinant is denoted

using

a determinant.

by "det A" or "IAI" and

If A

is

square

matrix,

is written as:

eaa

21

Also,

by

Determinant

det of A is

defined

represented

by the symbol "t, ",

Note that the determinant can only

be

for a square matrix.

Determinant of Second Order

If there are two equations in two variables, X and Y

a,x

+ b1Y;

(1)

a,x

+ b,Y;

(2)

Then,

the

:. det A ;

coefficient

matrix

(matrix

formed

by

the coefficient

of

variables)

is

given

by,

IAI;

The det A is called

"second order determinant."

Determinant of Third

Order

Suppose there are three equations having three u n k n o w n variables,

X, Y, and

Z, as:

a1X+b1Y+c1Z=0

a2X + b2 Y + c2Z ; 0
a3X + b

Y + c Z ; 0
3

From the above equations, the determinant of the third order can

a1

11 ; a,

a3

The

c1

c2

b3

C3

value

of

be written as:

the

determinant

of

the

third

order

can

be

found

by

expanding

determinant along the first row.

b2

c2

- b, a2

c2

a2

b2

a3

b3

+ C1

t,., = a1
b3

C3

a3

C3

www.itmuniversityonline.org

Page 45

the

I n t r o d u c t i o n to Q u a n t i t a t i v e T e c h n i q u e s

02.

Matrices a n d

eBook

Its A p p l i c a t i o n

Where, A 1, A2, and A3 are known as the cofactors of a . ,

Determinants are of the following

b 1, and

c1,

respectively.

types:

Ortho Symmetric Determinants

If every element of the


from

the

diagonal

leading

are

equal,

diagonal
then

is the same and

the

determinant

the e q u i d i s t a n t elements
is

said

to

be

an

"ortho

symmetric determinant."

For example,

Skew Symmetric Determinants

If the elements of a leading


to

its

conjugate

with

the

diagonal are all zero and

sign

changed,

the

every other element

determinant

is

said

to

be

is equal
a

"skew

symmetric determinant."

For example,

Singular and

Square

-h

-g

- f

Non-singular Matrices

matrix

A is

non-singular when

said

IAI

to

be

a "singular

matrix" when

IAJ

O and

it

is

known

as

;< 0 .

3
For example,

is singular as

= 6 - 6 = 0

In addition,

if

[!

is

non-singular as

I =

15 - 12 = 3 ,

which

equal t o o .

www. itmu niversityonline.o rg

Page 46

is

not

Introduction to Quantitative T e c h n i q u e s

02.

M a t r i c e s and

eBook

Its A p p l i c a t i o n

2 . 7 Properties of Determinant

If

rows

(or

columns)

are

interchanged

into

columns

(or

rows),

the

value

of

determinant remains the same.

For example,

a1

b1

c1

ll = a2

b2

c2

a3

b3

C3

After expanding along the first row:


ll = a 1 ( b

= a b2c
1

- b

) - b1(a

-a

) + c1(a

-a

- a b 3 c 2 - b1a2c
+ b a c2 + c a2b
1
3
1
3
1
3

= a1b2c3

a 1 b 3 c 2 + c1a2b3

b1a2c3 + b 1 a 3 c 2

c a

c1a3b2

b2

= ai(b2c3 - b 3 c 2 ) - a z ( b 1 c 3 - b 3 c , ) + a 3 ( b 1 c 2 - b 2 c 1 )

Interchanging

ll ' =

a1

a2

a3

b1

b
3

C1

C2

C3

Therefore,

rows into columns:

=a1(b

-b

)-a

(b1c

-b

c,J+a

(b1c

-b

c1)

ll = ll'

If two rows or columns are identical, the value of the determinant is zero.

For example,

ll = 2

After expanding along the first row

ll

= 2(18 - 2 0 ) - 3(12 - 4)

= -4 - 24

If

two

rows

+ 28

(or

+ 4(10 - 3)

= 2 x (-2)- 3(8)

+ 4(7)

= 0

columns)

of

determinant

are

interchanged,

the

sign

of

determinant changes as w e l l .

a1

a2

a3

If ll = b1

b2

b3

C1

C2

C3

= a1 (b 2 c 3

www.itmuniversityonline.org

b3c2)

a2 (b1 c3

b 3 c , l + a3 (b, c2

b2c1)

Page 47

the

I n t r o d u c t i o n to Q u a n t i t a t i v e T e c h n i q u e s

eBook

02. Matrices a n d Its A p p l i c a t i o n

Interchanging

t,'

b1

b2

b3

a2

a
3

C2

C3

Ct

/', ' = b 1 a 2 c
/', ' = - b

2a1c3

If each

-b1a

b1 (a2c

second

rows

b2

b3

t,' = a1

a2

a3

Ct

C2

C3

- a c2) - b2 (a1 c
- a ci) + b3 (a1 c2 - a c1)
3
3
3
2

-b2a1c

+b

c1 +b

a1c

-b

- b 3 C 2 ) + 2 (b1 C3

element

For example,

of a

row

t,

= 3

(or column)

1(12 - 2 0 ) - 2(18 - 0) + 4(12 - 0)

After m u l t i p l y i n g

/',!

2 x 5

4 x 5

10

= 3

10

20

the second

c1

1a3

c2

- b 3 c i ) - a3 (b1 C2 - b2c1)
of a determinant

number then the determinant is a l so multiplied

" =

+b3a1c2 +b1a2c3 -b3a2c1 +b2a3c1 -b

/',' = -1 (b2C3

the first and

b1

column

is

m u l t i p l i ed

by

the

by that number.

-8 - 36 + 48

by 5 which w i l l give

= 1(60 - 1 0 0 ) - 10(18 - 0) + 4(60 - 0)


= 1(-40) - 1 0 ( 1 8 ) + 4(60)
= -40 - 1 8 0 + 240 = 20
Also

5 x t, = 5 x 4 = 20

Hence,

:::

5 x

www. itmu niversityonline.o rg

Page 48

same

Introduction to Quantitative T e c h n i q u e s

02.

M a t r i c e s and

2.8

eBook

Its A p p l i c a t i o n

Applications of Determinant

F i n d i n g the Area of a Triangle

One of the main


vertices at

(x,,

uses of determinants is to test for invertibility.


(x2, vs),

vi),

and

(x,, v s ) .

will

be easy to compute the area

the

triangle

is

not

using

right-angled

Consider a triangle with

If the triangle is a right-angled

the formula

triangle,

then

; x

the

base

area

can

height.

also

be

triangle, then

However,

found

it

when

using

determinant.

If the

vertices are

(x.,

y,),

(x y,),

and

(x,,

y,),

then

the area

of the triangle

is given

by:

+ x

Yt

1]

Y2

Condition of Collinearity of Three

Points

If the area of a triangle is equal to zero, then the three points would

not form a triangle.

Therefore, the three points are collinear.

Let A(x 1,

y,),

B(x2,

y 2), and

C(x3,

y 3 ),

be three

points on a plane.

by these three points has an area equal to zero, then points A,

That is,

=>

x,

Y1

; x2

y2

X3

Y3

x,

Yt

x,

Y2

X3

Y,

If the triangle formed

B, and

C a r e collinear.

www.itmuniversityonline.org

Page 49

I n t r o d u c t i o n to Q u a n t i t a t i v e T e c h n i q u e s

02.

Matrices a n d

Example

For

the

eBook

Its A p p l i c a t i o n

06:

given

vertices

(-1,

-3),

(2,

4),

and

(3,

-1),

check

if

the

three

points

are

collinear.

Solution 0 6 :

If the area

of the triangle

formed

by the three

zero, then the three points are said

vertices

(-1,

-3),

(2,

4),

and

(3,

-1)

is

to be collinear.

The area of the given t r i a n g l e ; [ :

X3

YJ

;[-i:4:J

; .!. [ - 1 4
2

-1

1 - ( - 3 ) 2 1 + 1 2
1

4 ]
-1

; ; [-1(4 + 1 ) - (-3)(2 - 3) + 1(-2 - 1 2 ) ]

; ;[-5-3-14]

1
;

[ - 2 2 ] ; - 1 1 ,. 0
2

Therefore,

Example

the points are

not collinear.

07:

If the area of a triangle is 35 sq.

units with vertices

(2, - 6), (5, 4), and

(k, 4 ) , find

k.

Solution 0 7 :

Let

the

vertices

of

the

triangle ABC is 35 sq.

2
Therefore,

1
- 5

-6

triangle

be

(2,-6),(5,4),

and

(k,4).

Since

the

area

of

units:

1 ; 35

2
k

1
[2(4 - 4 ) - ( - 6 ) ( 5 - k) + 1(20 - 4 k ) ] ; 35
2
[6(5 - k) + 1(20 - 4k)]; 70
30 - 6k + 20 - 4k ; 70

www. itmu niversityonline.o rg

Page 50

the

Introduction to Quantitative T e c h n i q u e s

02.

M a t r i c e s and

eBook

Its A p p l i c a t i o n

- lOk = 70 - SO

lOk = S O 70
k =

12ork=-2

Cramer's Rule
Determinants are useful
rule in solving

in

finding

linear systems

the

inverse of a n o n - s i n g u l a r matrix.

is described

a,x + b,y

d,

(1)

a,x + b2y

= d2

(2)

Multiplying

(1)

by

b, and

(2)

by b, and

+ b1b,y = b2d1

(3)

a,b,x

+ b,b,y = b,d,

(4)

(4)

(a b

) x = (b

b 2 d 1 - b1d2
x =

a1b2 - a2b1

Now,

from

( 3 ) , you

d1

b1

d2

b2
b1

a2

b2

by m u l t i p l y i n g

(1)

= - (say)

by a, and

(2)

= a,d,

(5)

a,a,x + a,b,y

= a,d,

(6)

(4)

(a b1 - a1b )y
2

Thus,

This is called

( 3 ) , you

(a d1 - a 1 d

1
x = - and

from

you g e t :

2)

a,a,x + a,b1y

Now subtract

new equations,

D1

c--=-=-c
a1

the

get:

b d

For two equations consider the system:

subtracting

a,b,x

Now subtract

below.

Use of Cramer's

by a, and

subtracting,

you g e t :

get:

2
y = -

"Cramer's rule."

www.itmuniversityonline.org

Page

51

I n t r o d u c t i o n to Q u a n t i t a t i v e T e c h n i q u e s

eBook

02. Matrices a n d Its Application

The values of D,

0 1, and

D = determinant of A =

D,

is obtained

02 can be obtained as follows:

a,

bi

a2

b2

, that is, the coefficient of x and

y.

by replacing a,
a2

Dz is obtained

by replacing b,
b2

by

d,
d2

For example, consider the two equations

3x + 7y = 12
2x - 3y = 9

Let D be the determinant of the coefficient matrix.

-3

Let D,

-9 - 14

12

-3

o, =

matrix,

-99

be the determinant of the coefficient matrix,

12

where the column for the variable x

by the constants from the right hand side of the equation.

-36 - 63

has been replaced

D2

-23

be the determinant of the coeffi cient

has been replaced

Let D,

where the column for the variable y

by the constants from the right hand side of the equation.

The determinant of the coefficient matrix is not 0.

99

Then,

=D= -

o,

23

Then,

= D= -

D2

99

3
23

23

-3

23

www. itmu niversityonline.o rg

Page 52

Introduction to Quantitative T e c h n i q u e s

02.

M a t r i c e s and

eBook

Its A p p l i c a t i o n

Example 0 8 :

Solve the following

equations using

Cramer's rule.

2x + 3y = 13
x + 7y = 23

Solution 08:

The g i v e n equations are:

2x + 3y = 13

x +

7y

Here,

23

= 1 4 - 3 = 1 1 at 0

D =

The given system of equation

D1

13

23

= 9 1 - 69 = 22

2
D2

has a u n i q u e solution.

13

46 - 13

33

23
By Cramer's rule:

D1

y =

22

11

!2_

= 33 = 3

11

Cramer's Rule for Three

Equations

For three equations consider the system:

a1x + b1y + c1z = d 1

........................

(1)

= d,

......................

(2)

= d3

........................

(3)

a,x +
a3X

b,y

+ c,z

+ b3y +

Similar

to

C3Z

the

variables can

Cramer's

be obtained

d1

b1

C1

d2

b2

C2

D1

d3

b3

C3

a1

b1

CJ

a2

b2

C2

a,

b3

C3

rule

of

two

equations

with

two

variables,

the

value

for

as follows.

x -- -

www.itmuniversityonline.org

Page

53

three

I n t r o d u c t i o n to Q u a n t i t a t i v e T e c h n i q u e s

02.

Matrices a n d

S i m il a rl y ,

y =

eBook

Its Application

o,

a1

d1

C1

a,

b,

d,

a2

d2

C2

a2

b2

d2

a3

d3

C3

a3

b3

d3

a,

b,

c,

D
3
z=- =
D

and
a,

b,

c,

a,

b,

c,

a,

b,

c,

a3

b3

C3

a3

b3

C3

For example, considering the simultaneous equations


3x - 2y + 4 z = 5
x + y + 3z = 2

- x + 2y- z = 1

By Cramer's rule:

-2

- 1

D =

4j
3

1
= 321

- (-2)

- 1

+ 4

- 1 - 1

1
-

= 3 ( - 1 - 6 ) - ( - 2 ) ( - 1 - (-3)) + 4(2 - (-1))


=-21+4+12
=

o,

-5

-2

-1

= 51
2

_ (-2) 2

-1

3
-

= 5 (-1-6)-(-2)(-2-3)+4 (4-1)
= -33

-1

- 1

o,

- 5

= 3
1

-1

- 1

+4
-1

-1

= 3(-2 - 3 ) - 5 [ - 1 - (-3)] + 4 [1- (-2)]


=-15-10+12
= -13

-2

- 1

5
2

D3 =

1
- (-2)

2 = 31
1

www. itmu niversityonline.o rg

-1

2
5
1 +

-1

Page 54

Introduction to Quantitative T e c h n i q u e s

02.

M a t r i c e s and

eBook

Its A p p l i c a t i o n

= 3 ( 1 - 4) + 2 [ 1 - (-2)] + 5[2 - (-1)]


=-9+6+15

= 12

x = D1

D
33

= - 13

- 5 '
13
Y = S

X =

y = D2

= - 33

and

= D3

- 5 '

12
-5

12

and

Z = -

5,

Example 9 :
Solve the following
2x1-x2+3x

X2

- X3

system of equation

by using

Cramer's rule.

= 9

-1

X1 + X 2 - X 3

= 0

Solution 9 :
The given equation
2X1-X2+3X3

Ox

+ x

- x
3

X1 + X2 - X3

D =

Here,

D =

is rewritten as:

= 9

-1

= 0

-1

-1

-1

2(-1+1)-(-1)(0+1)+3(0-1)

= -2

''

3
And

I_\

l
=

9 (-1

= 0

-/
1

1)

+ 1 - 3

- 11
-1

- (=

1)(1

+ 0)

+ 3(-

1 - 0)

-2

www.itmuniversityonline.org

Page

SS

I n t r o d u c t i o n to Q u a n t i t a t i v e T e c h n i q u e s

02.

Matrices a n d

= 2 (1

eBook

Its A p p l i c a t i o n

- 0) - 9(0 +

1)

+ 3(0 +

1)

= 2 - 9 + 3 = -4

03

1]

- i

2 (0 + 1 ) - (-1)(0 + 1) + 9(0 - 1 )

= 2 + 1 - 9
= -6

By Cramer's rule:

o,
X1

-2

;::: - : - :; ! ,

-2

2 . 9 Inverse of a Matrix
If A and
called
an

B a r e two square matrices of the same order,

the

inverse of A and

inverse of a matrix,

an element are given


Minor of an

It

is defined

1)

The m i n o r of an element a,1

is for a matrix of order,

nxn,

the

minors

of

the

defined

by deleting

So, determinant of a matrix of order 3 x 3

a,,

I,

then,

by "A-1." To understand
cofactor of

elements

is

a11,

the 1th

by omitting

is represented

row and
by

column

M12

= (a21a33

M n =

(a22a33

(a21a32

a 12

a13

a 21

a22

a23

a31

a32

a33

and

a,,

a22

a23

a21

a23

a32

a33

a31

a33

are

and

- a23a32)

- a23a3i)

- a22a31)

www. itmu niversityonline.o rg

1)

row and i'h column of I A I .

, respectively.

in

M,1. The minor of

as the determinant of the (n

a 11

aaa.

the

a32

Mu

B is

below:

sub-matrix of IAI, formed

Therefore,

is denoted

as the value of the determinant obtained

and

Element

element a,

(n

inverse of B,

BA

the concept of minor of an element, determinant, and

which the element exists.


an

A is the

such that, AB =

Page

56

Introduction to Quantitative T e c h n i q u e s

02.

M a t r i c e s and

Cofactor of an

Element

Cofactor of an element a,

The cofactor of element

If

A =

all

a12

a13

21

eBook

Its A p p l i c a t i o n

22

is denoted

a11

is

by C11 and

is given

by:

C1;

= (-1jiM;;

1 1
= (- 1) M l l

ell

l
then the cofactor values of the elements are obtained as follows:

23

[
a31

a33

C 11

C21

= -1 (2'1Jca12a33 - a13a32)

C
31

-1<

a32

-1

)(a 22a33

- a23a32)

(3+1)(

)
a12a23

C12

-1<1+2J(a21a33

- a23a31)

C13

Czz

-1 (2+2\a11a33 - a 1 3 a 3 1 )

C23

(3+2)(
32

- a13a22

= -1

Table 2 . 9a:

For example,

suppose,

A=[;]=

The minor of the element a11

And

- a13a21

33

-1 (2+3Jcalla32 - a12a31)

= -1

(3+3\

a11a22 - a12a21

Cofactor Values of Elements

1]

-1

)
a11a23

-1 (1+3l(a21a32 - a 2 2 a 3 1 )

3 is,

M11

= 2

x 7 - 5 x 6 = -16

its cofactor is
1Mll

Cll

= (-1)1+

= (-1)2 x (-16) = - 1 6

The minor of the element a23 = 5 is,

And

its cofactor is

C23

Adjoint of a Square

= ( - 1 )2 + 3 M 2 3

M23 =

= (-1)

If

A =

12

a13

a21

a2 2

a 23

a31

a32

a33

18

x (18) = -18

Matrix

Let the determinant of a square matrix A be

all

3 x 6 - 0 x 4

l
, then

IAI =

IAI.

a11

a12

a13

21

22

a23

C11
If the matrix formed

by the cofactors of the elements

www.itmuniversityonline.org

in A is

C21
[
C31

Page

57

I n t r o d u c t i o n to Q u a n t i t a t i v e T e c h n i q u e s

eBook

02. Matrices a n d Its A p p l i c a t i o n

Then, the transpose of the matrix of cofactors is given

by

C11

C21

C12

C22

C13

C23

T h i s is called

the adjoint of the matrix A and

The property of adjoint matrix

A(Adj A)

= IAI

=>

A(Adj

is written as Adj A.

is:

A) = I

IAI
1
AA= I

And,

Then,

A-

Steps to

(Adj A)

Find the Inverse Matrix

Step

1:

find

the value of the determinant of g i v e n square matrix (say A ) .

Step

2:

construct a matrix of cofactors,

that is,

replace the values with cofactor values.

Step 3 :

take a transpose of the matrix of cofactors,

Step 4 :

compute the inverse matrix using

to obtain adjoint of given matrix.

For example,

IA I =

if

x s - c2)

C-3)

- 1

A = [-

the formula,

A-

l!I (Adj A ) .

] , then the inverse of A can be obtained as follows.

c-1)

-1s

+ 2

-13

The cofactor of the elements in the given matrix is obtained a s :


1

A11

= (-1)1 +

A12

A21

=(-1)2

A22

(-1) +

(-1)

2
+

x 5 = 5

x (-1)

x2=-2

x (-3) = -3

The cofactor matrix

-2

-3

= [

On transposition of cofactor matrix,

Adj

A = [ A11
A12

A 2 1 ] = [5

-2]

A22

- 3

www. itmu niversityonline.o rg

the adjoint of matrix A is obtained

as follows.

Page 58

Introduction to Quantitative T e c h n i q u e s

02.

M a t r i c e s and

A-

eBook

Its A p p l i c a t i o n

_!_ (Adj A)
JAJ

[5

= -13

-2J

-3

Therefore, the inverse of the given matrix

is

_ 3 [

- 2]
-3

Verification:

AA- 1 -

[-3

- -13

AA-

-1

2] 5 - 2
5

-3

[- 3 x 5 + 2 x 1

= - 13

[- 15 + 2

= -13

- 5 + 5

- 3 x -2 + 2 x -3J

- 1 x 5+ 5 x 1

[- 13

= -13

- 1 x -2 + 5 x -3

6 - 6]
2-15

-13

= [ ] = I

Since A A-

3
= I,

then the inverse of matrix

A = [-

-1

Example

is

qrven

as:

A-1

= -

1 [5
1 3

2]
_

10:

Find A-

2] .

for matrix

A=

-11

5
3

Solution

10:

The elements of the cofactor matrix are:

C11

= (-1)

= (-1)

12

+
1

[5 x 7 - 6 x (-3)] = (35 + 18) = 53

(3 x 7 - 6 x 1) = - ( 2 1 - 6) = - 1 5

+
1

C13

= (-1)

C21

= (-1)

C22

= (-1)

C23

=(-1)

[3 x (-3) - 5 x 1] = (-9 - 5) = -14

[7x4-(-3)x(-1)]=-(28-3)=-25

+
2

[2 x 7 - (-1) x 1] = 14 + 1 = 15
3

(2x-3-4xl)=-(-6-4)=10

www.itmuniversityonline.org

Page

59

I n t r o d u c t i o n to Q u a n t i t a t i v e T e c h n i q u e s

02.

Matrices a n d

eBook

Its A p p l i c a t i o n

(-1)

[4 x 6 - (-l)x 5]

29

C32

3 2
(-1) + [2 x 6 - 3 x (-1)]

-15

C33

= (-1)3 +

31

(2 x 5 - 3 x 4) = -2

Matrix of cofactor =

53

-15

- 25

15

-14]
10

[
29

On transposition,

IAI

Adj

-15

-2

53

- 25

29

-15

15

-15

[
-14

10

-2

A=

2[35 - ( - 1 8 ) ] - 4 ( 2 1 - 6 ) - 1(-9 -

SJ=

106 - 60 + 14

60

..A-'=i!i(AdjA)

=> A-

= _!__

53

-25

-15

15

60 [- 1 4

After m u l t i p l y i n g

=, AA - I = _!__

2 9 ]
-15

10

-2

the above matrices

2]

106 - 60 + 14

- 50 + 60 - 1 0

159-75-84

-75+75+60

87 - 75 - 12

53+45-98

-25-45+70

29+45-14

58 - 60 +

60 [

6
1
0

60

60

0 ]
0

60

OJ

[0

= I

www. itmu niversityonline.o rg

Page 60

Introduction to Quantitative T e c h n i q u e s

02.

M a t r i c e s and

eBook

Its A p p l i c a t i o n

2 . 1 0 Chapter Summary

system

of

numbers

columns and

bound

arranged

by

the

in

bracket

rectangular

[]

is

called

formation

an

by

along

matrix.

m
It

rows
is

and

written

n
as

m x n matrix.

a11

a12

a1n

a21

a22

a2n

am!

am2

amn

A =

A matrix obtained

by

mxn

interchanging

rows and

columns is known as a transpose of a

matrix.

The

square

matrix

is

symmetric

matrix

if

its

transpose

is

interchange of rows and columns does not change the matrix form,
[a,1]

Basic operations are addition of matrices,


by a scalar, and

Multiplication
matrix

of two

A determinant

If the elements

subtraction of matrices,

that is,

multiplication of

multiplication of matrices.

matrices

is

possible

only

if the

is a value of matrix.

of any

row

It is denoted

number of columns of the

or column

of a determinant are each

multiplied

by the

by that number.

Cramer's rule for two equations:


a,x + b,y

= d,

a,x + b,y

= d2

Therefore,

d1

b1

a1

d1

d2

b2

D
a2
1
= - and y = -

d2

=
a1

b1

a2

b2

D2
=

a1

b1

a2

b2

1
To find

left

by det A or IAI.

same number, the determinant is also m u l t i p l i ed

The

is the same as the number of rows in the right matrix.

same.

[a11].

a matrix

the

the inverse of a matrix, you can use

www.itmuniversityonline.org

A-

= l!I (Adj A).

Page 6 1

M e a s u r e s of
Central Tendency

I T M

Introduction to Q u a n t i t a t i v e T e c h n i q u e s

03.

eBook

Measures of Central Tendency

3 . 1 Introduction
A measure
average

of central

value

or

central

average is used and

For

example,

the

tendency

total

region.

This

then that can


Thus,

chapter

the

value

implemented

production

difficult to remember but


known,

is

analysis

that

of

sugar

from

the

data

entire

factories

production

as a representative of the total

different

by
set

life of every

20

if the value of the average

be used

on

describes

in the day-to-day

value of central tendency

focuses

of statistical

calculating
of

data.

a single

The

individual.

in

particular

of sugar for the

region
region

you

will

is

helps to represent a set of data.

methods

of

calculating

average

or

different

types

be able to:

Define the measures of central tendency

Calculate arithmetic, geometric,

Calculate median and

Identify the relation

and

harmonic mean

mode

between mean,

www. itmu niversityonl i ne.org

is

production of sugar in the

measures of central tendency.

After reading this chapter,

word

median, and

mode

Page 63

of

I n t r o d u c t i o n to Q u a n t i t a t i v e T e c h n i q u e s

03.

Measures

3.2

eBook

of Central Tendency

Central Tendency or Average

Central tendency
typical

value

better way,

is a single value which

around

which

other

is used

values

to represent an entire set of data.

cluster.

To

the concept of averages are discussed

understand

central

It is a

tendency

in

in this chapter.

Some important definitions of average are:


According

to A.

L.

Bowley,

Averages are

statistical

constants which

enable

us to

comprehend in a single effort the significance of the whole.

According
values to

to A.

E. Waugh,

represent them

An

average

in

some

is a single

way,

a value

the whole group of which it is part, as typical

According

to

Clark

and

Sekkade,

An

value
which

selected

from

is supposed

a group

to

stand

of
for

of all values in a group.

average

is

an

attempt

to

find

one

single

figure to describe the whole of figures.

From

the

dataset
value.

3.3

above

that

definitions,

describes

the

it

can

be

tendency

concluded
of

that

group

For this reason, an average is referred

of

average

values

is

to

the

central

concentrate

value

on

of a

single

to as a measure of central tendency.

Objectives of Averaging

Some of the important objectives of averaging are listed

below.

To find out a single value that describes the characteristic of a whole group

A measure of central
value.

This value

be used

is

it

a value

enables us to get an

to represent thousands,

For example,
country.

tendency

idea

mean

solves

this

represents the

of the entire data.

m i l l i o n s , and

is difficult to calculate and

Average

which

mass

data

Therefore,

in

one

single

one value can

billions of values.

remember the incomes of every i n d i v i d u a l

problem

by

giving

one

single

figure

in

the

in a

form

of

average income.

To facilitate comparison

By

reducing

of data

can

the entire data


be

done

easily.

www.itmuniversityonline.org

into
For

one

single value,

example,

the comparison

manager

can

between

compare

the

various

production

Page 64

sets
of a

Introduction to Q u a n t i t a t i v e T e c h n i q u e s

03.

eBook

Measures of Central Tendency

company for different


is improving

To aid

periods.

or deteriorating.

This comparison

helps to

identify whether the

production

Accordingly, the manager can take corrective measures.

in decision making

Averages act as a b e n c h m a r k or standard


production

manager may

production

targets

averages

act

as

rely

for the

on

the

individual

benchmarks

for

for managerial control and

average
and

productivity

of an

the organization,

performance

decision-making.

employee

as a whole.

appraisals

and

to

set

future

Therefore,

decision-making

these
in

the

future.

3.4 Characteristics of a Good Average


An

ideal average should

The

average

observation

satisfy the following characteristics:

should

be

should

based

affect

on

the

all

the

observations.

average,

only

then

The
will

absence
it

be

of

called

single

true

representative of the entire distribution.

Average
small or
make

Average

It

should

affected

in the

much

by

extreme

observations.

An

extremely

series of observations may distort the average and

be

rigidly

defined

through

some

algebraic

people compute the average of the same data,


be

suitable

collected

then

be

large value

should

has

not

can

it useless.

different

should

that

data

average

for further algebraic treatment.

on

the

average

should

be

sales

able

to

of

various

form

formula,

that

if

they get the same value.

For example,

firms

base

so

to

of a

if a

researcher

particular

calculate

the

industry,
combined

average sales of all the firms.

It should
of

have stability

population

research

average
each

should

scholar

population,

each

income

other.

sampling

in

get

collects
sample

value

The

sampling.

approximately
the

ten

difference

fluctuation.

income

consists

of

It

of

between

can

be

the

data

100

samples

fluctuation are less are considered

www. itmu niversityonl i ne.org

Different samples collected


same
of

adults

should
the

be

ten

of

average.

ages.

approximately

that

the

the same

For

of

averages

samples

Individually,
same

samples

set

example,

homogenous

mixed

averages

concluded

from

or

is

whose

of
the

close

known

to
as

sampling

effective.

Page 65

I n t r o d u c t i o n to Q u a n t i t a t i v e T e c h n i q u e s

03.

Measures

eBook

of Central Tendency

3 . 5 Various M e a s u r e s of Central Tendency


The different types of measures of central tendency used

Mean

Median

Mode

Mean can

be further classified

business and

industries are

into

Arithmetic Mean
o

Simple Arithmetic Mean

Weighted Arithmetic Mean

Geometric Mean

Harmonic Mean

Graphical
Fig.

in

representation of Classification of Measures of Central Tendency is depicted

3.Sa.

Measures of
Central Tendency

Arithmetic Mean

Mean

Median

Geometric Mean

Harmonic Mean

Mode

Simple
Arithmetic Mean

Weighted
Arithmetic Mean

Fig. 3 . S a :

Classification of Measures of Central Tendency

www.itmuniversityonline.org

Page 66

in

Introduction to Q u a n t i t a t i v e T e c h n i q u e s

03.

eBook

Measures of Central Tendency

3 . 6 Arithmetic M e a n
For a given set of observations, the Arithmetic Mean
the observations d i v i d e d

(A.M.)

by the number of observations.

It

is obtained

as the sum of all

is denoted

by " X " and

read

as x-bar.

S i m p l e Arithmetic Mean

S i m p l e arithmetic mean is calculated differently for ungrouped and grouped data.

For Ungrouped

Data

Ungrouped

is a

data

observations.

simple

series

Arithmetic mean

of discrete

for ungrouped

data.

data

It

can

does

not

include

be calculated

frequency

by a p p l y i n g

for

direct

and short-cut methods.

Direct Method

In direct method, mean is calculated

by d i v i d i n g the total value of the observations

by the total number of observations.

If the variable "X" takes "n" values

of the given values is obtained

x 1 , x 2 , x3 . . . X n , then the arithmetic mean,

X,

by using the formula:

Where,
n = Number of observations
n

L x,

= The s u m of v a l u e s of a l l observations

I :::1

Short-cut Method

In the short-cut method, A . M .


arbitrary basis, using

can be calculated

by taking an assumed

mean on an

the following formula:

"" d
X=A+-L,n

Where,

= (X - A)

A = Assumed mean

www. itmu niversityonl i ne.org

Page 67

I n t r o d u c t i o n to Q u a n t i t a t i v e T e c h n i q u e s

03.

Measures

eBook

of Central Tendency

Example 0 1 :

The

monthly

savings

of

employees

in

an

office

are

mentioned

below.

Calculate

5000

4000

the

average savings per employee per month.

Saving

2000

5000

1000

2500

6000

3000

2000

1000

(In")
Table 3 . 6 a :

Monthly Savings of Employees

Solution O 1 :
By

Direct Method

The average monthly savings of employees in an office

is calculated

using

the following

formula:
)( =

X1

+ X2 + X3

+ , .. +

Xn

2000 + 5000 + 1000 + 2500 + 6000 + 3000 + 2 0 0 0 + 5000 + 4000 + 1000


=10

= 31500

= 3150

10
The average saving

Using
the

the

above

observations

per employee per month

method,
are

one

vast,

can

then

calculate
the

i s 3150.

the

average

frequency

for

limited

distribution

is

values.

more

However,

helpful

to

manager.

www.itmuniversityonline.org

Page 68

if

the

Introduction to Q u a n t i t a t i v e T e c h n i q u e s

03.

eBook

Measures of Central Tendency

By Short-cut Method

Calculation of average monthly savings is calculated as follows:

d= x - A (A = 3000)

X = x
2000

-1000

5000

2000

1000

-2000

2500

-500

6000

3000

3000

2000

-1000

5000

2000

4000

1000

1000

-2000

Id=

1500

Table 3 . 6 b :

Ld

X = A +

Monthly Savings of Employees

1500

= 3000 +

= 3150

10

The average saving

per employee per month

For Grouped

Data (Discrete Series)

Grouped

data

has

grouped

data

can

frequency
include

arithmetic mean can

discrete,

be calculated

both the methods are given

distribution
as

i s 3150.

for

every

well

as,

observation.
continuous

through direct and

indirect

Unlike

series.

ungrouped

In

methods;

grouped

data,
data,

the formulae of

below.

Direct Method

In

case

of

simple

frequency

"f;" times, the arithmetic mean

distribution,

is given

assuming

the

observation

" x , " occurs

by:

)( = f1X1 + f2X2 + f3X3 + . . . + fnXn


f1

+ f2 + . . , + fn

Where,
x

M id- p oi nt of

ith

class in continuous series or the

ith

observation

in discrete

series

www. itmu niversityonl i ne.org

Page 69

I n t r o d u c t i o n to Q u a n t i t a t i v e T e c h n i q u e s

03.

eBook

M e a s u r e s of Central Tendency

f;

Frequency of i'" class

N = Total frequency

Short-cut Method
n

I;f,d,
1;;

Where,
d

_ X; - A
i

. _
I

1, 2 . . .

X = Mean
A = Assumed

mean

h = Size of the equal class intervals


N = Total number of observations

Note:

Short-cut

method

frequencies

are

for grouped

large

and

it

and

ungrouped

becomes

data

difficult

to

is a p p l i e d
calculate

only when the values and


them

through

arithmetic

means.

Example 0 2 :

The monthly electricity consumption of households residing


provided
all

in the table below.

in a residential complex is

calculate the average monthly electricity consumption for

households.
No. of
Energy
Households
Consumption
Frequency

(X)
(f)

Table 3 . 6 c :

Frequency

150

25

200

21

250

30

300

42

350

so

400

12

Distribution of Electricity Consumption of Households

www.itmuniversityonline.org

Page 70

Introduction to Q u a n t i t a t i v e T e c h n i q u e s

03.

eBook

Measures of Central Tendency

Solution

02:

By Direct Method

Following

table is computed

to calculate mean.
No. of

Energy
Households
Consumption

fiXi

Frequency

(x,)
(f,)

150

25

3750

200

21

4200

250

30

7500

300

42

12600

350

50

17500

400

12

4800

Table 3 . 6 d :

Frequency

180

Ix,=

50350

Distribution of Electricity Consumption of Households

The mean electricity consumption at the residential complex is:

x = I f;x, =
N

50350

279. 72

180

By Short-cut Method

The arithmetic mean can

= A+

be calculated

using:

If,d,

N
Where,
A = Assumed
d;

= x, - A

N = Total

mean
Deviation of

;th

observations from the assumed

mean "A"

number of observations

www. itmu niversityonl i ne.org

Page 7 1

I n t r o d u c t i o n to Q u a n t i t a t i v e T e c h n i q u e s

03.

eBook

M e a s u r e s of Central Tendency

The previous example can be so lved

using the short-cut method as follows:

Assumed mean "A" = 250


No. of
Energy Consumption

Households

(x;)

Frequency

d;

X;

250

fidi

(f;)

150

25

-100

-2500

200

21

-so

-1050

250

30

300

42

so

2100

350

so

100

5000

400

12

150

1800

I;t;d;
.

Table 3 . 6 e :

Frequency

= 5350

D1stribut1on of Electricity Consumption at a Residential


Complex

X = A + Lf;d;

= 250 +

s so =

250 + 2 9 . 7 2

279.72

180

The mean electricity consumption at the residential complex is 2 7 9 . 72

For Grouped

Data (Continuous Series)

Direct Method

In a continuous series,

where the data

arithmetic mean can be computed


Step 0 1 :

Step

02:

by applying

in

the form

of class

intervals,

the

the following steps:

Obtain the mid-point of each class interval and denote it by " rn , ",
Multiply

these

mid-points

interval and obtain the total

Step 0 3 :

is given

the

respective

frequency

of

each

L f;m;

Divide the total obtained

www.itmuniversityonline.org

by

by the sum of frequency.

Page 72

class

Introduction to Q u a n t i t a t i v e T e c h n i q u e s

03.

eBook

Measures of Central Tendency

Example 0 3 :

For the following

data,

obtain the arithmetic mean

0 - 10

Marks

by direct method.

20 - 30

10 - 20

30 - 40

40 - 50

No. of

15

25

45

35

Students
Table 3.6f:

Solution

Frequency Distribution of Marks of Students

03

Marks

Mid-point

No. of Students

(X)

(m.)

(f;)

fimi

0 - 10

20

100

10 - 20

15

25

375

20 - 30

25

30

750

3 0 - 40

35

175

so

45

10

450

40 -

N
Table 3 . 6 g :

I: f,m,

1aso

If;m;

90

1850

Frequency Distribution of Marks of Students

20_5

90

Short-cut Method

In a continuous series, the arithmetic mean may


following

be computed

steps:

Step 0 1 :

Take an assumed

Step 0 2 :

From

mean.

the m i d - p o i n t of each class interval, deduct the assumed

and divide the difference by size of class intervals.


given

by applying the

The deviation

mean

is calculated as

below:

K - A
d,

'

"h" is class interval

Step 0 3 :

find

'
M u l t i p l y the respective frequencies of each class by these deviations and

the total

Step 04:

L f;d;.

Apply the formula:

www. itmu niversityonl i ne.org

Page 73

I n t r o d u c t i o n to Q u a n t i t a t i v e T e c h n i q u e s

03.

Measures

eBook

of Central Tendency

Consider Table 3.6f and obtain the arithmetic mean


Assumed

by short-cut method.

mean "A" = 25

Mid-points

Marks

No. of Students

d,
I

-A

fidi

(f;)

(x;)

0 - 10

20

-2

-40

10 - 20

15

25

-1

-25

20 - 30

25

30

30 - 40

35

40 - 50

45

10

20

N
.

Table 3 . 6 h :

If;d;

90

-40

Frequency D1stnbut1on of Marks of Students

Z:f,d,

-40

X = A +

Therefore,

= 25+

10 = 2 0 . 5 6

90

Properties of Arithmetic Mean

The mathematical

properties of the arithmetic mean are:

The sum of the deviations of the

items obtained

by subtracting

mean from the value of observations is always zero, that is,

the arithmetic

L (x

- X)

It is clear from the following example:

Z: x

x - X

-10

10

-5

15

20

25

10

75

L (x-

X)

= 0

Table 3 . 6 , :

Here

'

"x

75

_
L,
_

= - =

Ungrouped

Data

15

www.itmuniversityonline.org

Page 74

Introduction to Q u a n t i t a t i v e T e c h n i q u e s

03.

eBook

Measures of Central Tendency

The sum of the squared deviations of the observations from the arithmetic mean is
m i n i m u m , compared

If

each

item

in

substitutions w i l l

X =

Lx

or

nX

to any other central tendency.

the

series

is

replaced

by

the

mean,

be equal to the sum of the i n d i v i d u a l

then

the

sum

of

these

item.

Lx

For example,

if

Lx

= 100

and

the arithmetic mean

by 20,

you get the same total 20

When

the

related

as X 12

number

groups

is

of

items

given,

the

is

20.

If each

item

is

replaced

+ 20 + 20 + 20 + 20 = 1 0 0 .

and

the

arithmetic

combined

average

mean

of

of these

two

or

groups

more

can

be

than

two

calculated

N1X1 + N,X,

N1 + N2

Where,

X1, X, = Arithmetic mean for two groups


N

1,

= S a m p l e size of the g r o u p s

X12 = Combined average of the two means

Weighted Arithmetic Mean

The weighted
value,

with

mean

is calculated

respect

to

the

by

total

taking
value.

into account
The

term

the

relative

weight

importance of each

stands

for

the

relative

importance of the different observations.


It is calculated

using

the formula:

Lwixi
Xw

=
=
i
l

Lwi

Where,

X. = Weighted
x, =
w,

;th

arithmetic mean

value of variable X

= Weig ht assigned

to the

www. itmu niversityonl i ne.org

;th

variable value

x,

Page 75

I n t r o d u c t i o n to Q u a n t i t a t i v e T e c h n i q u e s

03.

Measures

eBook

of Central Tendency

In the case of frequency distribution

if f,, f, . . . f,

are the frequencies of the variable


n

I:w,(f,x,)
values x.,

x2 . . . Xn, then weighted

arithmetic mean

Xw =

=
L.,

w,

Example 04:

The data

related to the

bonus of people is given

in the following

table.

Find the weighted

arithmetic mean.

Bonus(')

so

60

70

80

90

100

110

(x,)
No. of People

(w,)
Table 3 . 6 j :

Frequency

Distribution of Bonus Paid to People

Solution 0 4 :

The average bonus paid

per member is:

Bonus(')

so

60

70

80

90

100

110

350

560

540

200

110

(x,)
No. of People

(w,)

so

WiXi

180
.

Table 3 . 6 k :

Frequency D1stnbut1on of Bonus Paid to

People

'
_

I: w , x ,

I=

I:

w
,

1990
25

79. 60

www.itmuniversityonline.org

Page 76

Introduction to Q u a n t i t a t i v e T e c h n i q u e s

03.

eBook

Measures of Central Tendency

3 . 7 Geometric Mean
Geometric
time.

It

Mean

(G.M.)

plays a vital

is used

role

If

the

number

of

managerial

"n" items. So,

root of the product of

t h e i r geometric mean

in

to evaluate the average

is given

by:

observations

decision-making

if

X1,

X2

G.M. = x .x
1

is

more

Xn

rate of change over a period


and

can

are the given

be defined

as the

of

n'"

"n" observations then

x,

than

three,

then

to

simplify

calculations,

logarithms are used.

og

G M
.
.

G.M.

_ logx 1 + logx 2 + . . . + l o g
N

= antilog(Ilg

X;

Xn

In a discrete series, G . M .

In a continuous series,

.
(I log x;
= antllog
N

G M . = anti log [If; lg m ; ]

where "m," is the mid-point of i'" class

interval.

Discrete Series

Example 0 5 :

The d a i l y production ( i n tonnes) of four companies is given below.

Calculate the average

daily production of the given data using the geometric mean formula.

Daily
Production

500

250

1462

1275

(in tonnes)

(X)
Table 3 . 7 a :

Daily Production

www. itmu niversityonl i ne.org

(in tonnes) of Four Companies

Page 77

I n t r o d u c t i o n to Q u a n t i t a t i v e T e c h n i q u e s

03.

Measures

eBook

of Central Tendency

Solution O 5 :

.
G.M.

Find

= antlloq

[2::log
N

the logarithm of the g i v e n values and

add

them to get:

Production
log x,
(x,)

500

2.6990

250

2.3979

1462

3.1650

1275

3.1055
Ilog x,

Table 3 . 7 b :

G.M.

= anti log(

Daily Production

11.3674

(in tonnes) of Four Companies

2:: l o g x
N
')

= antilog(11.!674)

= anti log

2. 8418

= 694. 7

Note:

Use scientific calculator to compute log and

www.itmuniversityonline.org

antilog of given values.

Page 78

Introduction to Q u a n t i t a t i v e T e c h n i q u e s

03.

eBook

Measures of Central Tendency

Continuous Series

Example 0 6 :
From

the following d i s t r i b u t i o n , find

the average increment ( i n

percentage) of the

employees.

Increment

0 - 10

10 - 20

20 - 30

30 - 40

40 - 50

15

25

to
No.Of
Employees
.

Table 3.7c:

Solution

Increment ( i n percentage) of the Employees

06:
Mid-point

Increment O/o

f;

log mi

fi

x log mi

m;

00 - 10

0.6990

3.4950

10 - 20

15

1.1761

8.2327

20 - 30

25

15

1.3979

20.9685

30 - 40

35

25

1.5441

38.6025

40 - 50

45

1.6532

13.2256

Table 3 . 7 d :

G.M.

Lf

= antilog

log m J
N
'

60

L f,

log m,

= 84.5243

Increment of the Employees

= antilog

(84.5243)
60

= antilog

www. itmu niversityonl i ne.org

1.40874 = 2 5 . 6 4

Page 79

I n t r o d u c t i o n to Q u a n t i t a t i v e T e c h n i q u e s

03.

Measures

eBook

of Central Tendency

3.8 Harmonic Mean


Harmonic

Mean

the

reciprocal

X2 . . .

Xn

(H.M.)

of the

is

based

on

arithmetic

the

mean

reciprocals of numbers averaged.


of the

is a given set o f " n " observations,

reciprocal

observations,

H.M.

individual

= -1-1---1

n
=--

- + - + ..... +

I]_

X1

is defined

observations.

as

If x,,

then the harmonic mean is given as follows:

n
In ungrouped

of

It

X2

X;

Xn

N
In grouped

observations or discrete series,

H.M.

, N ; Lf,

=
1
I;(f, x - )

x,
N

In

continuous

series,

H.M.

= --

I(f,

where "mi" is

the

m i d - p o i n t of

ith

class

interval

x ,)

a n d N = I; f ;

Harmonic Mean for Individual Observations


For example,

if the following data

x
1/X

is given, then

H.M.

be:

10

13

20

28

25

63

0.100

0.077

0.050

0.036

0.040

0.1250

0.0159

Table 3.Sa:

Therefore,

harmonic mean will

= -=

0.4439

Ungrouped

Data

= 15.77

I ,Yx ;

www.itmuniversityonline.org

Page 80

Introduction to Q u a n t i t a t i v e T e c h n i q u e s

03.

eBook

Measures of Central Tendency

Harmonic Mean for Discrete Observations


Example 0 7 :

Calculate

harmonic mean for the following:

:
:

Table 3 . S b :

Discrete Series

Solution 0 7 :

If the following

data

is given,

then the harmonic mean will be:

X;

10

15

20

30

35

40

f;

20

30

25

40

so

SS

2.0

2.0

1.25

1.33

1.43

1.375

f;/x;

Table

N
=
L

/
f
_
; X
_
; =

H. M.

3.Sc:

Discrete Series

220
9.385

23.441

3.9 M e d i a n
Median

is the middle value of a series,

the arithmetic mean,


a

positional

median

average.

In

the

is

not

case

when arranged

based

of

on every observation

median,

series are less than the median value and

in any order of m a g n i t u d e .

50%

of

50% are above

the

in

the

series.

observations

It

Unlike

is called

of the

entire

it.

For Individual Observations

For

individual

observations,

the

median

may

be

computed

by

applying

the

following

steps:
Step

1:

Step 2:
Step

Arrange the data

in ascending

or descending

Identify the m i d d l e value of the dataset.


In

3:

a group

and d i v i d e by 2.

of "N"

It can

odd

number

be expressed

order of magnitude.

This is the m ed i a n value.

of values,

add

to

the

total

number of values

in the form of a formula as:


1)

Median

Step 4 :

Observation

located

at

In a group of " N " even

(N;

'" position.

number of values,

the median

is the arithmetic

mean of

the two values in the middle.

www. itmu niversityonl i ne.org

Page 8 1

I n t r o d u c t i o n to Q u a n t i t a t i v e T e c h n i q u e s

03.

Measures

eBook

of Central Tendency

th

th

Median=

A.M.

of two values located

at

()

( +

and

1)

position.

Example 08:
Find

the median of the following

data:

2, 6, 9, 3,

7, 4, and

12.

order:

2, 3, 4, 6,

7, 9, and

12.

Solution 0 8 :
Arrange the series in ascending
N

= 7

Median

= Observation

located at

(N;

)th

position

7; 1
=

)"'

The observation

position

4th

located

at the

4th

position

is 6,

so the median of the given dataset is 6.

Example 09:
The
and

income

24300.

(in

Find

of

six

employees

the median

is

given

as:

25000,

20000,

20600,

27000,

25400,

25000, 25400, and

27000.

income.

Solution 0 9 :
Arrange the series

in ascending

order:

20000, 20600, 24300,

Since the

number of observations is 6,

by taking

the arithmetic mean of two m i d d l e position values.

Median= A.M.

of two values located at

A.M.

of

(24300; 25000)

3rd

and

which

()th

is an even

and

number,

( + 1)"'

the median

is obtained

position

position values

4th

= 24650

For a Discrete Series

After arranging

the data

in

ascending

or descending

order, find

the cumulative frequency

(c.f.), then a p p l y :
N

Median

= Value corresponding

to c. f.

> -

- 2

www.itmuniversityonline.org

Page 82

Introduction to Q u a n t i t a t i v e T e c h n i q u e s

03.

eBook

Measures of Central Tendency

Example

10:

Find the median of the following

data:

Income

10000

15000

25000

35000

40000

45000

15

25

30

15

10

(In')
No. of
People
Table 3.9a:

Solution

Distribution of Income of People

10:

The given data

is already in ascending

order.

Income

10000

15000

25000

35000

40000

45000

15

25

30

15

10

40

70

76

91

101

(In')
No. of People
Cumulative Frequency

15
.

Table 3 . 9 b :

Median=

D1stnbut1on of Income of People

to c.f. just greater than

N = 1 0 1 =

Here,

c.f.

Value corresponding

5 0 _ 5
2

= 7 0 ;,

50.5

The value corresponding


Therefore,

median

to c.f.

(=

70)

is 25000

= 25000

For a Continuous Series

In a continuous series, the following

median.

In a continuous series,

calculate median

L +

is used

to

to determine the exact value of the

locate the median class.

The formula to

is:

%-f
Median

formula is used

Where,
L =

Lower limit of the median class

f0 = Cumulative frequency of the class preceding


fm
i =

the median class

Frequency of the median class

Class interval of the median class

www. itmu niversityonl i ne.org

Page 83

I n t r o d u c t i o n to Q u a n t i t a t i v e T e c h n i q u e s

03.

Measures

Example

Find

eBook

of Central Tendency

11:

the median salary of the employees of a factory.


Salary

No. of

(Int)

Employees

10 - 20

10

20 - 30

15

30 - 40

20

so

30

40 Table 3.9c:

Solution

Distribution of Salary of Employees

11:
Salary

No. of

(Int)

Employees

10 - 20

10

10

20 - 30

15

25

30 - 40

20

45

so

30

75

c.r.

40 Table 3 . 9 d :

Distribution of Salary of Employees

N = 7 5 = 3 7 . 5

Here,

Median class = Class corresponding

to c. f.

N
75
,, _ = - = 3 7 . 5

2
c.f.

;, 3 7 . 5

Therefore,

Median

is 45, which corresponds to the class interval 30 - 40


median class is 30 - 40

= L +

%
-f0
2

x i

fm

= 30 + 3 7 . 5 - 25

10

20
= 36.25
Therefore,

the median salary of the employees of a factory is t36.25

Advantages of Median

The median

is simple to understand and

It is not affected

by extreme values and

easy to calculate.
it can

be computed

by using

open-ended

class intervals too.

www.itmuniversityonline.org

Page 84

Introduction to Q u a n t i t a t i v e T e c h n i q u e s

03.

eBook

Measures of Central Tendency

Disadvantages of Median

In the case of an even


may not necessarily

n u m b e r of observations for ungrouped

be the value contained

To calculate the median,


descending

it

data,

the median

in the dataset.

is necessary to arrange the data

in ascending

or

order of magnitude.

It is difficult to compute the median for a dataset containing

large amounts of

data.

3 . 1 0 Mode
Mode

is

often

therefore,

said

the

to

value

be

the

that

value

occurs

which

most

has occurred

discrete

be

observation

mode

can

most

frequently.

mode is an observation that


series,

occurs

located

In

maximum

through

often

an

in

the

ungrouped

data.

The

discrete

mode

series,

is,
the

number of times, while in a grouped

highlighting

the

maximum

frequency

in the series.

For Discrete Series

The following
76, 65,

The

78,

79,

weight

mode

data shows the weight of 15 workers:


80 and

value

here

is

78

78.

60,

78, 67,

80,

57,

79,

60,

78,

59,

75.

occurs

In

maximum

discrete

number

ungrouped

of

times,

serie s,

the

that

is,

mode

can

times.
be

Hence,

determined

the
by

inspection o n l y .

For Continuous Series

When
the

the data

following

frequency.

is grouped

formula

to

in

find

a continuous frequency
the

mode,

It is given by the following

which

is

distribution,

located

in

the

a manager can
class

with

the

highest

formula:

11,

L +

::::
0

x
f,, 1

MO

::::

+ 62

f1

L +

fo

.
x

2f

Where,

L =

Lower limit of the modal class

f1 =

Frequency of the modal class

fa=

Frequency of the class preceding

www. itmu niversityonl i ne.org

apply

the modal class

Page 85

I n t r o d u c t i o n to Q u a n t i t a t i v e T e c h n i q u e s

03.

eBook

M e a s u r e s of Central Tendency

f2 = Frequency of the class succeeding the modal class


61
i=

= f1 - fo, 62

= f1 - f2

Class interval of the modal class

Example

12:

The monthly sa v i n g s of 100 employees per month are given.

Saving

Table 3 . l O a :

Solution

per Month

Find the mode.

No. of

(In')

Employees

2000 - 3000

20

3000 - 4000

18

4000 - 5000

30

5000 - 6000

12

6000 - 7000

20

Distribution of Savings per Month of Employees

12:

Since the m ax i mu m frequency is 30, which corresponds to the modal class 4000-5000.

Therefore,

L +

.6.

.6.1 + l:i. 2

Here,

6,

= f

-f, = 3 0 - 1 8 = 12

= f, -f,

12
+

M o = 4000 +
12

= 4000

= 4000

= 3 0 - 1 2 = 18

x 1000
18

12
+ 30 x 1000

+ 400 = 4400

www.itmuniversityonline.org

Page 86

Introduction to Q u a n t i t a t i v e T e c h n i q u e s

03.

eBook

Measures of Central Tendency

Example

13:

Find the value of mode from the data given

below:

Weight

93-97

98-102

103-107

108-112

113-117

118-122

123-127

128-132

12

17

14

(In kg)
No. of
People

Table 3. l O b :

Solution

Distribution of Weight (in kg) of people

13:

First, convert the classes into class boundaries.


Lower limit of class b o u n d a r y = Lower l i m i t of the corresponding c l a s s ('ower l i m i t of second clas - upper limit of first class)

Upper limit of class b o u n d a r y = Upper l i m i t of the corresponding class+


cower limit of second clas - upper limit of first class)

Weight

No. of
Class Boundaries
People

(In kg)

93 - 97

92.5-97.5

98 - 102

97.5 - 102.5

103 - 107

102.5 - 107.5

12

108 - 1 1 2

107.5 - 1 1 2 . 5

17

113 - 117

112.5 - 117.5

14

1 1 8 - 122

117.5 - 122.5

123-127

122.5-127.5

128-132

127.5-132.5

Table 3 . l O c :

D1stnbut1on of Weight (on

kg) of people

Here, the maximum frequency is 1 7 . The corresponding class 1 0 7 . 5 - 1 1 2 . 5 is the


mod a I class.

111

Therefore,

M0

= L +

til + b..2

Here,

11

= f

ti,

f, - f,

L +

f0 = 17 - 12 = 5

17 - 14 = 3

61

l:i.1 + /::J.2

www. itmu niversityonl i ne.org

Page 87

I n t r o d u c t i o n to Q u a n t i t a t i v e T e c h n i q u e s

03.

Measures

eBook

of Central Tendency

5
=

107.5+--x5
5 + 3
25

= 107.5+

= 1 0 7 . 5 + 3 . 1 2 5 = 110.625

Advantages of Mode

Mode

can

be

used

to

describe

qualitative

phenomenon,

for

example,

when

manager wants to compare consumer preferences for different types of products.

The value of the mode can

It is not affected

be determined

graphically.

by extreme values.

Disadvantages of Mode
The

value of the

mode

is

not

two or more modes, making

Relation between
A distribution
distribution.
asymmetrical.
following

in

Mean,

which

When

relation

on

these
to

mean,

Solution

values
Karl

median are 27,

=,, 2 7

- 2Mean

Mean

series.

Some datasets contain

Pearson,

and
not
if

median

coincide

equal,

the

the

3Median

called

distribution

distribution

median, and

is

is

a
is

symmetrical
known

asymmetrical,

then

mode.

- 2Mean

21.6,

respectively.

Find the mean of the distribution.

14:
- 2Mean

..

are

h o l d s between the mean,

Mode = 3 M e d i a n
3 x 21.6

=,, 2Mean

of the

14:

The mode and

item

Median

mode,

Mode

Example

every

it difficult to interpret the result.

Mode, and

the

According

based

= 64.8

- 27

18. 9

www.itmuniversityonline.org

Page 88

as
the

Introduction to Q u a n t i t a t i v e T e c h n i q u e s

03.

eBook

Measures of Central Tendency

3 . 1 1 Chapter S u m m a r y

One

of

the

most

important

objectives

of

statistical

value that describes the entire data. This is called

An

ideal average should

The

important

mean,

harmonic

different
grouped

series

of

mean,
of

mathematical
median,

and

observations

mode.

which

are

to

get

one

single

the central value or an average.

and

averages

is

calculate.

are

arithmetic

These averages
broadly

divided

can

mean,
be

into

geometric

calculated

ungrouped

for
and

data.

Arithmetic mean
and

types

be easy to understand

analysis

ungrouped

Weighted

can

by direct and

short-cut

methods for both,

grouped

data.

average

each value with

Geometric mean

H a r m o n i c mean

Median

is the

be solved

is

calculated

by

taking

into

account

the

relative

importance

of

respect to the total value.


is the average rate of c h a n g e over a period of time.
is based

middle

on the reciprocals of numbers averaged.

value of a series arranged

either in

ascending

or descending

order.

Mode

is

value

in

the

data

that

the observation corresponding

The

relation

between

mean,

www. itmu niversityonl i ne.org

occurs

most

frequently

for ungrouped

to the maximum frequency for grouped

mode and

median

is,

Mode=

3Median

data

data.

- 2Mean.

Page 89

or

is

M e a s u r e s of D i s p e r s i o n

I T M

Introduction to Q u a n t i t a t i v e T e c h n i q u e s

04.

M e a s u r e s of Dispersion

eBook

4 . 1 Introduction
The measures of central tendency studied

in the earlier chapter provide values to which

observations are assumed to concentrate on a s i n g l e point.

However, central tendency is

unable to point the degree of variability among observations. Thus,


to

study

the degree of dispersion

distribution of observations.

that

provides

Different data

may

it becomes essential

higher understanding
have the same mean

to

the

pattern

of

but with different

variability.

Measures
the

of dispersion

expected

value.

convey

The

the

degree

measures

of

to

which

dispersion

values

that

are

in

a distribution differ

used

most

often

from

are inter

quartile range and standard d e v i a t io n .

After reading t h is chapter, you w i l l

be able to:

Explain the measures of dispersion

Define range, quartile deviation, absolute mean deviation,

standard

deviation,

and

coefficient of variation

Calculate

range,

q u a rt i l e

deviation,

absolute

mean

deviation,

standard

deviation,

and coefficient of variation

List the merits and demerits of important measures of dispersion

www. itmu niversityonl i ne.org

Page 9 1

I n t r o d u c t i o n to Q u a n t i t a t i v e T e c h n i q u e s

04.

Measures of D i s p e r s i o n

4 . 2 M e a n i n g and
Dispersion,
observed

eBook

Definition

which means scatter,

values

vary

from

the

spread,

central

or variation,

value.

It

measures the extent to which the

helps

to

compare

two

or

more

sets

of

observations in terms of variability.

Some common definitions of dispersion are:


According

to

A.

L.

Bowley,

Dispersion

is

the

measure

of

the

variation

of

the

items.

In another definition,

according

to

B. C.

Brooks,

Dispersion

or spread

of the scatter or variation of the variables about a central

4.3

which data
To

is most useful.

test

the

average

in different ways and

in different scenarios to determine

Some important aspects of measures of dispersion are:

reliability

of the

average:

it

helps

to

determine the extent to

represents the characteristic of a statistical data.

the average is h i g h l y
No variation

value.

Significance of Measures of Dispersion

Measures of dispersion are used

is the degree

reliable and

if the variation

of

data,

with

dispersion or variation
has

respect

would

mean

to

It

correlation,

or

helps to compare two or


consistency.

greater

as compared

less degree of dispersion.

It is useful for further statistical analysis.


as

homogeneity

it

lack of consistency or uniformity,

serves

is small,

indicates uniformity of data.

series

to data that

If the variation

an

is large, an average is unrealistic.

To compare the variability of two or more sets of data:


more

which

basis

to

regression,

www.itmuniversityonline.org

calculate

and

testing

other

statistical

measures

which

are

used

of hypothesis.

Page 92

in

Introduction to Q u a n t i t a t i v e T e c h n i q u e s

04.

M e a s u r e s of Dispersion

eBook

4.4 Types of Dispersion


Measures of dispersion can
of dispersion and

be

broadly classified

into two categories,

absolute measures

relative measure of dispersion.

Measures of
Dispersion

Absolute

Relative

Measures

Measures

Quartile

Mean

Standard

Deviation

Deviation

Deviation

Coefficient

Coefficient

Coefficient

of Range

of Quartile

of Mean

Deviation

Deviation

Fig. 4.4a:

Absolute

of Variation

Types of Dispersion

Measures of Dispersion

Absolute dispersion
are

Coefficient

given.

Thus,

is expressed

they

are

in the same u n i t in which the original data series

suitable

to

compare

the

variability

of

two

or

more

distributions where units of measurement are the same.

Relative

Measures of Dispersion

They are expressed

in the form of ratios or percentages, or coefficient of absolute

measures of dispersion. Thus,

Methods of Studying

relative measures of dispersion are free from

units.

Dispersion

There are various methods to study dispersion.

Range

Quartile deviations

Absolute mean deviation

Standard deviation

Coefficient of variation

www. itmu niversityonl i ne.org

The most important methods are:

Page 93

I n t r o d u c t i o n to Q u a n t i t a t i v e T e c h n i q u e s

04.

Measures of D i s p e r s i o n

Range and
of

the

and

quartile deviations are

variable

standard

4.4.1

eBook

at

particular

positional

positions

deviation are called

in

measures,

the

since they

distribution.

The

depend

absolute

on the values

mean

deviation

mathematical measures of deviation.

Range

Range

is

between

the

simplest

the

value

method

of

the

studying

smallest

d i s t r i b u t i o n . " Mathematically,

Range=

of

it can

dispersion.

and

the

It

is

largest

be represented

defined

as

observation

"The

difference

included

in

the

as:

L - S

Where,
L =

Largest observation

S =

Smallest observation

If the

units

measure

to

of various data
a

distributions
measure
called

manager

given

that

is

in

for

sets are

different,

comparison.

different

independent

units
of

the

of

then

However,

the
to

measurement,

units

of

range

calculated

compare
a

the

manager

measurement.

This

is

not

variability

can

use

relative

useful
of

the

relative

measure

is

the coefficient of range.

Coefficient of range

5
L L+S

The coefficient of range

is the

ratio of the difference

between

two extreme observations

of the distribution to t h e i r sum.

Example 0 1 :

The daily wages of the workers, as given, are:

Daily Wages of the


Workers

50 - 100

100

- 150

150

- 200

200

- 250

(inf)

No. of Workers

Table 4 . 4 . l a :

www.itmuniversityonline.org

10

18

15

21

Distribution of Daily Wages of the Workers

Page 94

Introduction to Q u a n t i t a t i v e T e c h n i q u e s

04.

M e a s u r e s of Dispersion

eBook

Calculate the range and coefficient of range for the given data.

Solution

Range=

01:
L -

= 250 -

50

Range = 200

.
L - S
250 - 50
200
Coefficient of range= L +
=
+
=
= 0.67
5
250
50
300

Example 0 2 :

The following table gives the age distribution of a group of 95 i n d i v i d u a l s :

Age

6 - 10

11 - 15

16 - 20

21 - 25

26 - 30

36 - 4 0

31 - 35

(In Years)
No. of

15

35

18

Individuals
Table 4.4. l b :

Distribution of Age of Individuals

Calculate the range and the coefficient of range for the given data.

Solution 0 2 :

This

is

an

respective

inclusive
class);

series

for

the

(value

equal

calculation

of

to

the

range,

upper
it

must

limit
be

of

the

class

converted

into

is

i n c l u d ed

a continuous

series, as shown in the following table.

Inclusive

Continuous

Classes

Classes

Frequency

6 - 10

5.5 - 10.5

1 1 - 15

10.5 - 1 5 . 5

16 - 20

15.5-20.5

15

21 - 25

20.5 - 25.5

35

26 - 30

25.5 - 30.5

18

31 - 35

30.5 - 35.5

36 - 40

36.5 - 40.5

Table 4.4.lc:

www. itmu niversityonl i ne.org

5
.

in

D1strobut1on of Age of Ind1v1duals

Page 95

I n t r o d u c t i o n to Q u a n t i t a t i v e T e c h n i q u e s

04.

Measures of D i s p e r s i o n

L = 40. 5,
Range=

eBook

S = 5. 5
L - S

= 4 0 . 5 - 5 . 5 = 3 5
Range=

35
5

L-

Coefficient of range=

= 40.5-5.5 =0.76
40.5+5.5

L + S

Applications of Range

It

is

used

in

industries

manufactured

It is useful

The

to

prepare

control

charts

for

statistical

quality

control

of

products.

in studying

meteorological

variation

in stock prices.

department

also

uses

range

to

determine

the

difference

between temperatures.

It

is

also

used

in

day-to-day

departmental store, determining

life.

For

example,

determining

daily

sales

in

monthly wages of workers in a factory, etc.

Merits of Range

It is the simplest method

It can

It is r i g i d l y defined, that is,

It is widely used

be used

to

understand

when the data

and the easiest to compute.

is of a qualitative nature.

it is subject to fluctuations, from sample-to-sample.

for quality control measures and

stock market fluctuations.

Demerits of Range

It is not based

It

is

highly

on every observation of d i s t r i b u t i o n .

affected

by

fluctuations

of sa m p l i n g .

It

does

not

give

any

information

about the character of distribution.

It cannot be used

4.4.2 Quartile

in case of open-ended cases, that is, open class intervals.

Deviation

Range as a measure of dispersion


the

observations

observation

is

are

within

discarded

the
the

has many

range.
limited

limitations.

Therefore,
range

when

It fails to explain
the

established

dispersion
is

more

how

of

the

scattered
extreme

instructive

and

representative of the entire data.

This

range,

range.

To

which
obtain

approximately the

includes the m i d d l e
this

range,

same size,

www.itmuniversityonline.org

the

that

is,

50/o of the distribution,


observations

are

is called

divided

into

the inter-quartile
four

groups

of

one fourth of the observations or 25% of the total

Page 96

Introduction to Q u a n t i t a t i v e T e c h n i q u e s

04.

Measures

of Dispersion

eBook

distribution. Thus, the distribution of data


second
the

quartile

median

obtained

of

after

quarter of

the

(Q,),

and

third

distribution.
discarding

This

one

observation

or

higher

quartile

inter-quartile

quarter

at

has three quartiles, first or lower q u a rt i l e (Q,),

the

of the

upper

end

(Q,).

The

range

is

observation
of the

second

quartile

calculated
at

the

from

observations

lower end

distribution.

That

represents

and

means

another

the

inter

quartile range represents the difference between the third quartile and the first quartile.

Three quartiles and

the

inter-quartile

range can

be

represented

graphically

as

shown

in

Fig. 4.4.2a.

Interquartile

Range

1"'

Lowest

2ow1

Quartile

Fig. 4.4.2a:

Quartile deviation
median

(Q.D.)

Deviation

Items

3,-,j Quartile

Quartile

Highest

Q,

Q,

Q,

the

of the d i s t r i b u t i o n .

by dividing

the inter-quartile range by 2.

Q 3 ; Q1

average
In

Range

Q1

is obtained

(Q.D.)

gives

Observation

Three Quartiles and Inter-quartile

Inter-quartile Range = Q3

Quartile

of

(median)

Observation

Quartile Deviation

of Items

amount

symmetrical

by

which

the

distribution,

two

the

quartiles

differ

two quartiles

Q,

from

and

Q,

the
are

equidistant from the m e d i a n .


Median - Q1 = Q, - Median

This

difference

exactly
small

of

variation

denotes
called

50/o

large

can

be

taken

as

the observations.
in

the

central

variation.

A small

50%

The

measure

of the

relative

of

dispersion.

q u a rt i l e

deviation

observations,

measure

The

median

denotes

while

corresponding

high
to

high

Q.D.

covers

uniformity

quartile

q u a rt i l e

deviation

deviation

the coefficient of quartile deviation.

www. itmu niversityonl i ne.org

or

Page 97

is

I n t r o d u c t i o n to Q u a n t i t a t i v e T e c h n i q u e s

04.

Measures of D i s p e r s i o n

eBook

Q, - Q l
=

Coefficient of Q . D .

Q, +

Quartiles for Ungrouped

To find

o,

Data

quartiles, first arrange the values in ascending

compute the values of the upper and

to

Upper quartile

to

above

= Q, = value corresponding

formula

is

applicable

then

lower quartiles.

Lower quartile = Q1 = value corresponding

The

order of their m a g n i t u d e and

if "N"

is

an

( N ; l rh

item

odd

(N4+ l) rh item

number

but

if "N"

is an

even

number,

say,
N =

Q1 =

10,

then:

N)'h
(4

value corresponding to

item

1)th

= value corresponding

to

= value corresponding

to

(2. s)"' item

= value of 2"'

Similarly,

the

+ 0 . 5 (value of 3'' item - value of 2"' item)

item

same

item

procedure

is

used

to

compute

Q 3,

if

the

value

of

is

an

number.

Quartiles for Grouped Continuous Data

The following

formulae are used

to compute quartiles for grouped

continuous data.

N
- - c.f.

Lower quartile = Q1 = L1 + 4

xi

3N

- - c.f.
Upper quartile

Q3 = L

x i

Where,
L1

Lower limit of the

respective quartile class, first or third

N = Total frequency
f = Simple frequency of the quartile class

www.itmuniversityonline.org

Page 98

even

Introduction to Q u a n t i t a t i v e T e c h n i q u e s

04.

Measures

i =
c. f.

of Dispersion

eBook

Class interval
= Cumulative frequency of class preceding

to the respective quartile class

Example 0 3 :

Calculate the quartile deviation


data

related

to the wages

and

the coefficient of quartile deviation

for the following

(tn e) of workers.

Wages

Less than 45

45 - 47

48 - 50

51

- 53

Over 53

18

(f)

14

No. of Workers
.

Table 4.4.2a:

Solution

03:

Following

table is used

62

99

D1strobut1on of Wages

(m f) of Workers

to compute quartiles.
Wages

No. of Workers

(f)

(f)

c.f.

Less than 45

14

14

45 - 47

62

76

48 - 50

99

175

51

- 53

18

193

Over 53

200

Table 4.4.2b:

.r'!_ = 200

Distribution of Wages ( i n ? ) of Workers

= 50

The c.f.

immediately greater t h a n

50

is

76;

therefore,

Q,

lies

in

the class 45

- 47

(first

quartile class).
N
- - c.f.
Lower quartile = Q1 =

4
L1 +-f-xi

45

50 - 14

62
= 46.16

3N = 3 x 2 0 0 =
150

4
The

4
c.f.

immediately

greater

than

150

is

175;

therefore,

Q3

lies

in

the

class

48

(third quartile class).

www. itmu niversityonl i ne.org

Page 99

50

I n t r o d u c t i o n to Q u a n t i t a t i v e T e c h n i q u e s

04.

eBook

Measures of D i s p e r s i o n

3N
- - c.f.

Upper quartile ; Q

L1 +

3;

x i

; 48 + 1 5 0 - 76

99
; 49.49

49.49 - 4 6 . 1 6
Thus,

Q.D.

1.67

Q, - Q ,
Coefficient of Q. D.

Merits of Quartile

It is considered to

It

also

be

not affected

It is useful

3.33

= 0.034

95.65

Deviation

can

49.49 - 4 6 . 1 6
49.49 + 4 6 . 1 6

Q3 + Q,

be a better measure of dispersion,

computed

for frequency

distribution

as compared

with

to range.

open-ended

classes.

It

is

by extreme observations.

in studying

the middle 50%

items in the series.

Demerits of Quartile Deviation

It is not applicable for any further mathematical computation.

It is affected

A variation

It is not based

considerably

in sampling

affects the value of quartile deviation.

on all observations.

4.4.3 Absolute

Mean

Absolute

deviation

mean

by fluctuations of s a m p l i n g .

Deviation

or

mean

deviation

(M.D.)

a b so l u t e deviation of a set of observations from


or

mode.

Mean

deviation

is

also

known

as

can

be

defined

its central value,

average

as

the

mean

such as mean,

deviation,

as

it

is

the

of

median,
average

difference between observations and central tendencies of the data set.

Mean

If

Deviation for Ungrouped

x 1 , x 2 , x3

. . . , Xn

are

an average "A" is given

M.D.

"n" given

Data

observations,

then

the

absolute

mean

deviation

by:

= .! L I X - A l = .! L I d I
n

www.itmuniversityonline.org

Page

100

about

Introduction to Q u a n t i t a t i v e T e c h n i q u e s

04.

Measures

eBook

of Dispersion

Where,

d ; x - A
x ; Value of observations
n ; Total n u m b e r of observations

A ; Central tendency of a data set

Note:

The

symbol

Ix

-Al

indicates

the

absolute

values

obtained

by

ignoring

the

sign

of

deviation.

The

relative

deviation.

measure

corresponding

to

mean

deviation

is

called

the

coefficient of mean

If the mean deviation is taken from the m e d i a n , t h e n :

Coefficient of mean deviation

(about mean)

M.D.
Mean

Coefficient of mean deviation

(about median)

MdD.
Me

Coefficient of mean deviation

(about mode)

,an

M.D.
Mode

Example 04:

The income of a group of five members is given

below.

Find

the mean deviation from the

median.

Income

3200

(t')
Table 4 . 4 . J a :

3400

3600

Income of a Group of Five

3800

3000

Page

101

Members

Solution 04:

First, the observations are arranged

in ascending order.

1
Median;

(n;

)'h

value corresponding

to

; value corresponding

to

; value corresponding

to 3'' item

www. itmu niversityonl i ne.org

l)th
T
(

item

item

I n t r o d u c t i o n to Q u a n t i t a t i v e T e c h n i q u e s

04.

Measures of D i s p e r s i o n

The

value

3400.

corresponding

The

mean

eBook

to

deviation

the

from

3''

item

is

the m e d i a n

3400,
A =

so

the

median

of

is calculated

3400,

the

as

given

shown

data

in

is

Table

4.4.Jb.

Income
I

Ix

34001

c,>
3000

1-4001

= 400

3200

1-2001

= 200

3400

I O I =

3600

12001

= 200

3800

14001

= 400

::I

Table 4.4.Jb:

Mean deviation =

.!. L I

= 1200

Income of a Group of Five

Members

Where,

Deviation of observations from median ignoring signs

io ;

12
Therefore, mean deviation =

240

This means that the average deviation of the i n d i v i d u a l incomes from the median income
is 240.

Example O S :

Calculate the mean deviation from median for a discrete series.

Table 4.4.Jc:

www.itmuniversityonline.org

Discrete Series

Page

102

Introduction to Q u a n t i t a t i v e T e c h n i q u e s

04.

Measures

Solution

Following

of Dispersion

eBook

05:

table

is

used

to

compute

mean

deviation

from

median

for

given

discrete

series.

c.f.

10

11

12

15

12

12

18

33

13

12

45

12

14

48

48

Table 4.4.3d:

= Lfl

M D
.

!'!.

I = L

48

Lf

I
'

= 36

Discrete Series

x - median

24

c.f.

just

greater

than

Therefore, the m e d i a n =

D.

= L fl

I = L f[

Lf

I=

24

a grouped

or

is

33,

and

the

value

of

"X"

corresponding

to

33

is

12.

12

36

0. 75

48

Mean Deviation for Grouped

In

The

M.

fl d i

[ d ]

continuous

Data

series,

first,

the

midpoint

of the

various

classes

is obtained

and the deviations of these points from the central tendency are computed.

The

mathematical

representation

or

formula

of

mean

deviation:

L f[

x - A

= Lf.

www. itmu niversityonl i ne.org

Page

103

where

I n t r o d u c t i o n to Q u a n t i t a t i v e T e c h n i q u e s

04.

Measures of D i s p e r s i o n

eBook

Example 0 6 :

Calculate the mean deviation

(about mean) for the following

Class Interval

data:

4 - 6

6 - 8

8 - 10

10 - 12

Frequency

Table 4.4.3e:

Continuous Data

Solution O 6 :

Mean deviation

{about mean)

L ti

x - X i

Now, arithmetic mean

Take assumed

" f(x

is given

Itd

X = A + N
-

by:

mean A = 7

-A)

X = A + L..,

=
N

Mid-

Frequency
(x

Class
value

(x)

- 7)

f(x - 7)

I =

Ix

7.21

(f)

4 - 6

-2

-6

2.2

6.6

6 - 8

0.2

0.8

8 - 10

1.8

3.6

11

3.8

3.8

10 - 12

Ifd

10

If!

x - X

= 2

14.8
Table 4.4.3f:

Mean deviation

(about mean)

.1:_

L ti

Continuous Data

148
x- X i =

= 1.48

10

Merits of Mean Deviation

It is less affected

by the value of extreme observation.

It is based

It is simple to understand and easy to compute.

It is rarely used

on every

item of the data.

in sociological studies.

www.itmuniversityonline.org

Page

104

Introduction to Q u a n t i t a t i v e T e c h n i q u e s

04.

Measures

of Dispersion

Demerits of Mean

eBook

Deviation

If mean deviation

is computed

from

the mode,

it is

not scientific because the value

of mode cannot always be determined.

It

does

not

take

mathematically

into

account

the

algebraic

Mean deviations cannot be calculated

It is affected

Standard

deviations,

which

can

cr (sigma)

for open-ended

classes.

be defined

and

as the

square

deviations from

was

first

root

of variance

arithmetic mean.

suggested

by

Karl

where

variance

Standard

Pearson

as

Computation of Standard

measure

For ungrouped data

For grouped data

Standard

Standard

Deviation

for two types of data:

(discrete series)

(continuous series)

Deviation for Ungrouped

deviation for ungrouped

Direct method

Short-cut method

Data

data can

For Direct Method, the formula is:

cr =

be computed

J L (x

by two methods:

- X)

Where,
a = Standard

x =

deviation

Observations

N = Total

number of observations

www. itmu niversityonl i ne.org

is the

deviation

dispersion.

It is calculated

are

Deviation

deviation

by

the

by the fluctuations of sample size.

average or mean of the squared


denoted

of

illogical.

4.4.4 Standard

signs

Page

105

is
of

I n t r o d u c t i o n to Q u a n t i t a t i v e T e c h n i q u e s

04.

Measures of D i s p e r s i o n

eBook

S t ep s for Calculation

Step 0 1 :

calculate the arithmetic mean of the data

(X)

Step

02:

take the deviations of the observation from the mean

Step

03:

square these deviations and obtain the total

Step

04:

d i v i d e it by N to obtain standard deviation

I<x-

(x - X)

X)

Example 0 7 :

Find the standard deviation of 4, 9,

11,

12,

17,

5, 8,

12, and

14.

Solution O 7 :

92
X=-=10.222
9

(x - X)2

11

12

17

12

14

38.7

1.49

0.60

3.16

45.9

27.3

4.94

3.16

14.3

I x = 92

I(x-X)2

= 139.55

Table 4.4.4a:

cr =

J I ( x - X)

J!

Discrete

Data

x 1 3 9 . 5 5 = 3.94

Short-cut Method

When mean

is not a whole number,

take the deviations


overcome

such

from

in a fractional value,

the mean and

difficulty,

short-cut

or

then,

obtain

assumed

it becomes a tedious process to

the square of these deviations.

mean

method

is

used

to

calculate

standard deviation. The short-cut method for calculating standard deviation is:

Where,

d = x - A = Deviation from assumed mean


A = Assumed
1d
1d

mean

= S u m of deviations
2

= Sum of square of deviations

N = Number of observations

www.itmuniversityonline.org

Page

To

106

Introduction to Q u a n t i t a t i v e T e c h n i q u e s

04.

Measures

Standard

Standard

of Dispersion

eBook

Deviations for Grouped Continuous

deviations

for

grouped

continuous

Data

data

can

be

measured

with

the

following

methods:

Direct method

Short-cut method

Step deviation method

For Direct Method, the following

steps can

be used

Step 0 1 :

Find

Step 0 2 :

Calculate the arithmetic mean using

to find

standard

deviation:

the mid-points of various class intervals

the formula:

Ifx

= -N

Step 0 3 :

Find

Step 04:

Square the deviations and

class and

the deviations "d" of the m i d - p o i n t s from

obtain

L fd'

, use

m u l t i p l y them

the formula:

For Short-cut Method, the following

by the respective frequencies of each

J I, fd

formula

the arithmetic mean

is used

J I, f(x - X)

to compute standard deviation:

Where,
d = x - A
A = assumed

mean

Example 08:

Find the standard

House

deviation

if house prices are, as given

below:

Prices

( In Thousands)

40 - 60

60 - 80

80 - 100

100 - 120

120 - 140

15

(ll')

Frequencies

Table 4.4.4b:

www. itmu niversityonl i ne.org

Continuous Data

Page

107

I n t r o d u c t i o n to Q u a n t i t a t i v e T e c h n i q u e s

04.

eBook

Measures of D i s p e r s i o n

Solution 0 8 :

= ::[fx = 3500
N

87_5

40

House Prices

Mid-

Frequenc

(In Thousands)

point

= (x -

fx

fd

87.5)
(x)

(')
40 - 60

50

250

1406.25

7031.25

60 - 80

70

630

306.25

2756.25

80 - 100

15

90

1350

6.25

93.75

100 - 120

110

880

506.25

4050

120 - 140

130

390

1806.25

5 4 1 8 . 75

::[ fx

Hd'=

3500
19350

Table 4.4.4c:

= J1

I fd'

= J19350 = 21.99
40

Step Deviation

Step

deviation

deviations

Continuous Data

Method

method

taken

from

is

used

assumed

to

make

mean

the

are

calculation

divided

by

easy

and

a common

simple.
factor

To

simplify

which

is

usually

the size of class interval to get the step deviation " d ' ''.

::[fd'
I td' 2
cr = ,1='-c--c-- - (--)
N

xi

Where,

( x - A)
i

i = the common factor or class interval


A= ass
u m e d mean

www.itmuniversityonline.org

Page

it,

108

Introduction to Q u a n t i t a t i v e T e c h n i q u e s

04.

Measures

of Dispersion

Combined Standard

It

is

possible to

the following

eBook

Deviation

compute the combined

standard

deviation

of two

or

more groups,

using

formula:

N1(crf + d f ) + N 2 ( cr + d )

"

-.,-
12 -

N1 + N2

Where,

cr.,

combined

standard deviation

cr,

standard

deviation of first group

cr,

standard

deviation of second

d1

1x

Where

1 -

12

1 ,

X 12 -

d,

Ix,

group

x.,I

N 1 X 1 + N2X2
N1 + N2

The above formula can

be extended

to find out the combined

standard

deviation of three

or more groups.

Example 0 9 :

The

number of workers employed,

deviations

in

each

section

of

the

the

mean

factory

wages

are

(in )

given.

per month,

The

deviation of all the workers are taken together as shown

mean

and

wages

in the following

Workers

Mean Wages

Standard

Employed

(Int)

Deviation

the
and

standard
standard

table.

Section

50

1113

60

60

1120

70

1115

80

90
.

Table 4.4.4d:

Solution

Sect,on-w,se

D1stribut1on of Wages

09:

The following

formula

= (50 x 1 1 1 3 )

is used

+ (60

to compute combined

x 1120)

+ (90

x 1115)

standard

223200 =

deviation of three groups.

1116

123

50 + 60 + 90

200

N 1 (0; + d ; ) + N , ( 0 ; + d ; ) + N , ( 0 ; + d ; )
CT 123

N1 + N, + N,

www. itmu niversityonl i ne.org

Page

109

I n t r o d u c t i o n to Q u a n t i t a t i v e T e c h n i q u e s

04.

eBook

Measures of D i s p e r s i o n

d.

= I X. - X 1 2 3 l = l l l 1 3 - 1 1 1 6 l = 3

d,

d,

= Ix, -

F," -

x,,,I = 1 1 1 2 0 - 1 1 1 6 I = 4

x;;;-1

N1
cr 123

o;

= 1111s-1116 I = 1

+ d ; ) + N, (

o;

o;

+ d;) + N 3 (

+ d;)

=
N1 + N , + N 3

50(60
cr 123

2
2
2
2
2
+ 3 ) + 60(70
+ 4 ) + 90(80
+ 1 )

=
s o + 6 0 + 90

1051500
200

"123

= 72.51

4.4.5 Coefficient of Variation

The

relative

measure of dispersion,

of variation and

based

on

standard

deviation

is called

the coefficient

is given as:

Coefficient of Variation

"

x
This

is generally expressed

Coefficient of Variation

in a percentage form, that i s :

" x 100

x
The coefficient of variation measures the spread
used

to

compare

the

two or more datasets.

variability,

homogeneity,

of data as a proportion of its mean.


stability,

uniformity,

and

The dataset for which the coefficient of variation

consistency

is
of

is greater is said

to be less consistent or less homogeneous.

www.itmuniversityonline.org

It

Page

110

Introduction to Q u a n t i t a t i v e T e c h n i q u e s

04.

Measures

Example

of Dispersion

eBook

10:

Wages paid

( i n " > to the workers in two factories, A and

Daily Wages

No. of Workers

(Inf)

in Factory A

in the table below:

No. of Workers
in

Factory

- 130

15

25

130

- 140

30

40

140

- 150

44

60

150

- 160

60

35

160

- 170

30

12

170

- 180

14

15

Table 4 . 4 . S a :

which factory

are given

120

180 - 190

Find

B,

Distribution of Workers According to Wage Group

pays a higher average wage and

which

factory

has a consistent wage

structure?

Solution

To

find

mean

10:

out
is

which

factory

calculated.

calculated.

For

The following

pays
the

higher

consistency

table

is

used

to

average
of

wage

wage

than

structure,

the

others,

coefficient

the
of

compute the average wages and

arithmetic

variation

coefficient of

variations for the given data.

Assumed

mean A =

1 5 5 and

www. itmu niversityonl i ne.org

h =

is

10.

Page

111

I n t r o d u c t i o n to Q u a n t i t a t i v e T e c h n i q u e s

04.

Measures of D i s p e r s i o n

eBook

MidWages

(x - A )
point

fA

fAd

d=

(t)

fAd

fad

fa

fad

h
(x)

120 - 1 3 0

125

15

-3

-45

135

25

-75

225

130 - 140

135

30

-2

-60

120

40

-80

160

140 - 1 5 0

145

44

-1

-44

44

60

-60

60

150 - 1 6 0

155

60

35

160 - 1 7 0

165

30

30

30

12

12

12

170 - 1 8 0

175

14

28

56

15

30

60

180 - 1 9 0

185

21

63

15

45

}J.d'

Ne=

-70

= 448

192

}JAd

NA=

200

Table 4.4.Sb:

}Jd

:i:fd

-158

562

Distribution of Workers According to Wage Group

For factory A:

XA = A + L/Ad x h = 155 + -

x 10

151. 5

200

NA

2
LfAd

70

-[LfAdJ

NA

h =

NA

448 - ( - 7 0 ) '
200

10

200

= 1 . 4 5 5 x 10

Coefficient of Variation for factory A =

"A

14. 55

100

XA

14 55
-

x 100

9.60

151.50
For factory B:

Xa = A + :d x h

1 5 5 + (-/
x 10
9528)

146. 77

cra

Lfad2 - ( L f a d J 2 x h =

Na
cra

1.499

192

Na

10 =

562 - ( - 1 5 8 )

10

192

14.99

www.itmuniversityonline.org

Page

112

Introduction to Q u a n t i t a t i v e T e c h n i q u e s

04.

Measures

eBook

of Dispersion

Coefficient of Variation for factory

B =

x 100

""

Xa

14.99

xlOO

146.77
= 10.22

Since the arithmetic mean


wage t h a n factory

B.

is higher for factory A,

The coefficient of variation

that is factory A pays a h i g h e r average

is less for factory A;

therefore, factory A

has a more consistent wage system than that of factory B.

Merits of Standard

Deviation

It is based on every observation of the d i s t r i b u t i o n .

It

can

be

groups.

used

To

variation

to

compare

It

It

gives

the

combined

variability

of two

or

standard

deviation

more distributions,

economists

or

two

the

or

more

coefficient

of

Deviation

For example,

the general

nature of extracting

readily comprehensible for a non-mathematical

greater

of

to be the most appropriate.

is difficult to compute.

root is not

the

is considered

Demerits of Standard

calculate

weight

to

extreme

businesspersons,

who

values
are

and

more

this

person.

has

interested

the square

not
in

found

the

favor

results

of

with

modal

class.

Difference between

In

mean

However,
The

sum

Mean

deviation,
in

the

standard

of the

Deviation and Standard

deviations

deviation,

squares of the

are

the

taken

from

deviations

deviation

Deviation

any

are

of items

of

taken

from

the

central

from

tendencies.

arithmetic

the arithmetic

mean.

mean

is the

least.

In

mean

standard

deviation,

algebraic

signs

deviation, s i gns are taken

www. itmu niversityonl i ne.org

are

ignored;

whereas,

in

the

calculation

into account.

Page

113

of

Introduction to Quantitative T e c h n i q u e s

05.

eBook

Correlation

Correlation
relation

can

be

between

computed

any

two

only

for

variables.

numerical

This

data.

linear

It

measures

relation

can

be

the

degree

represented

of

linear

using

the

equation of a straight line,


Y = a +

bX

Where,
Y = the dependent variable
a = the intercept on Y-axis
b = the slope
X = the independent variable

ro
9

'
'

x
0

Fig. 5 . 2 a :

,o

'
Linear Correlation

5 . 3 Significance of Correlation
Study
the

of correlation

various

methods for forecasting

a p p l i ca b i l i t y owing

has

Most

factors

other,

such

demand,
different

and

applications
future

to the following
in
as

in

real-life

planning

by

executives.

income
factors.

and

between

expenses,

Such

It

It

is

used

has g a i n ed

as

one

of

enormous

reasons:

business environments exhibit


relationship

scenarios.

variables

etc.

relationships

some

All

are

like

these

identified

kind
price

factors
and

of relationship with
and

supply,

influence

measured

supply

one

with

each

or

the

and

more

help

of

correlation.

Once the
value

of

income

relationship
another

is

between the two factors is established,

variable.

established,

the

For

example,

value

of

one

once

the

factor

it is easier to find

relationship

may

be

between

derived

with

price

the

Page

and

help

another factor.

www.itmuniversityonline.org

the

119

of

I n t r o d u c t i o n to Q u a n t i t a t i v e T e c h n i q u e s

05.

eBook

Correlation

Correlation
estimating

tool

is

future

of absolute
projections,

importance

for

economists.

such as potential demand

They

use

the

of a commodity,

tool

for

effect of

price on sales, impact of advertisement in sales, etc.

The

tool

also

helps

business domain,

the

such

executives

as geographical

are crucial to decision-making

T h o u g h correlation
own

limitations,

is a widely used

as

well.

of

The

firm

to

locations,

identify
costs,

critical

profits,

factors

etc.

in

Such

the

factors

in business.

and

preferred

correct

use

of

tool

in statistical

correlation

tool

inferences,
depends

it has

on

its

correct

interpretations of the data derived from the tool.

5 . 4 Types of Correlation

Correlation
value

can

of one

be classified

variable

into

changes

various

due

to

types

depending

variations

in

the

on

value

the

pattern

of another

in

which

variable.

the
The

types depend on the relationship between two or more variables or the impact of change
in one factor on the other factors. The different types of correlation are as follows.

Positive and

Depending

Negative Correlation

on

variable's

change

in

direction,

correlation

negative correlation.

With

same direction,

there is a positive correlation

then

reference to each other,

is

classified

as

positive

if both the variables fluctuate


between them.

or

in the

Some examples of

positive correlation are:

Increase in advertisement expenses increases the sale of a product

Increase in

price of luxury goods such as, designer clothes or Rolex watches leads

to increase in demand of the said goods

Fig.

www. itmu niversityonline. erg

5.4a:

l l

Positive Correlation

Page

120

Introduction to Quantitative T e c h n i q u e s

05.

eBook

Correlation

However,
value

when the decrease

of a different

fluctuate

in

variable,

the value of a variable is

that

is,

in the opposite direction,

with

reference

then there

to

identified

each

by an

other,

if

increase

both

is a negative correlation

the

in the

variables

between

them.

Some examples of negative correlation are:

Decrease in the price of petrol leads to increase in sale of cars

Decrease

in

the aviation

the

price of air tickets

witnesses an

number of air travel

in

industry

l
Fig.

Simple,

increased

Multiple, and

In simple correlation,

S.4b:

Negative Correlation

Partial Correlation

relation

between two variables is studied.

Some examples of

simple correlation are:

Price and

Age and

demand

of a product

income of the consumer

However,

when the

relationship

multiple

correlation.

It

between two or more variables is s t u d i ed ,

studies the

influence on a group

of factors.

it is known as

Some examples of

m u l t i p l e correlation are:

Effect of age of consumer,

income of consumer, and

price of the commodity on the

demand of the particular commodity

Effect

of

obtained

Partial

marks

obtained

Zoology,

Botany,

and

Physics

on

the

overall

marks

in an examination

correlation

studies

other variables u n c h a n g e d .

in

Production

of a

the

impact of one

variable

on

the

other,

keeping

effects of

Some examples of partial correlation are:

commodity

and

price

of

raw

material,

keeping

labor,

machinery,

keeping

geography,

and capital unchanged

Demand

of a commodity and

income of the

consumer,

and consumer preference constant

www.itmuniversityonline.org

Page

121

price,

I n t r o d u c t i o n to Q u a n t i t a t i v e T e c h n i q u e s

05.

eBook

Correlation

Linear and

Non-linear Correlation

The correlation
change

in

the

is said
other

to

be linear in

factor

in

the

nature,

same

ratio.

relationship between them,

it is represented

When any two variables do

not share

equation

then

of a

straight

line,

if the

ratio of change in one factor

When

any

two

variables

share

leads to
a

linear

are not represented

by an

by a straight line.

a linear

there exists

relationship and
a

non-linear

or

curvilinear

correlation

between them.

In

the

case

of

variables may

non-linear

or

curvilinear

correlation,

be in the form of a curve on the g r a p h .

the

With

relation

between

any

two

the change in one variable in

a non-linear correlation, the other variable will not c h a n g e with a constant ratio.

N o n - l i n e a r correlation

is represented

in

Fig.

S.4c.

1800

1600

I
1400

I
1200

I
1000

I
800

600

/
400

/
200

'

10

'
Fig. S . 4 c :

In

most

cases,

complex and
correlation

the

techniques

difficult and

hence,

of

Non-linear Correlation

interpreting

non-linear

a general assumption

type

of

correlation

is made that there

are

is a non-linear

between such variables.

Spurious Correlation

One

may

misinterpret

relationship

between

the cause and

effect

the

existence

of correlation

any two variables.


relationship

www. itmu niversityonline. erg

However,

between

as

the

existence of cause

correlation

does not

two or more variables.

and

necessarily

If there

effect
imply

is a cause and

Page

122

Introduction to Quantitative T e c h n i q u e s

05.

eBook

Correlation

effect

relation

them

is certain.

Sometimes,
a

high

termed

For

any

two

variables,

two entirely unrelated

degree

of

correlation

then

variables with

between

them.

the

existence

no cause and

Such

of

correlation

effect

correlation

is

relation

between

may show

coincidence

and

is

as spurious or nonsense correlation.

example,

their

between

income.

numerical

there

is

no

However,

value,

which

cause

if

we

is

effect

calculate

due

cautious while interpreting

and

to

relation

its

between

correlation

change

only.

the

height

coefficient

Therefore,

it

is

r,

it

of

may

advisable

people
give
to

and

some

be

very

the correlation coefficient r.

5 . 5 Method of F i n d i n g Correlation u s i n g Scatter Diagram


He r e , the
using

the

method

explains

scatter diagram.

the scatter diagram


the

Y-axis.

The

can

how

to

identify

the

correlation

For a set of values taken

be

plotted

coordinate

points

by taking
can

be

one

by

between

the two

variable on

identified

as

(X,

If the coordinate points in a scatter diagram are closely


there
it

can

is a strong
be

correlation

interpreted

value of r will

that

between

there

is

them.
strong

positive

Based

on

variables

say,

the

X and

another on
clutter

it can

points are

correlation

be interpreted

in

upward

between

them

and

+1

'
y

3".i .. -

.. -"'
. ,:,.
. ., .
., r
:JJtM

,. .
. . , ,.,

Fig.

www.itmuniversityonline.org

5.Sa:

Strong

and

Positive Correlation

Page

that

direction,

be close to + 1.

-1

Y,

be interpreted.

placed,

If the dense

variables,

two

the X-axis and

Y).

direction of points, correlation between the two variables can

any

123

the

I n t r o d u c t i o n to Q u a n t i t a t i v e T e c h n i q u e s

05.

If

eBook

Correlation

the

direction

is

upward

and

the

points

are

dispersed

from

each

other,

it

can

be

interpreted that there is a weak positive correlation between them and the positive value
of r will be nearer to zero.

-1

+1

y ,


:
.
.
.



.
. . .

..

x
-

Fig. S . S b :

If

all

the

upward

points

in

scatter

direction as shown

Weak Positive Correlation

diagram

in the

Fig.

lie

on

S.Sc,

the

it can

same
be

straight

interpreted

line

and

move

in

the

that there is a perfect

positive correlation between them and the value of r is exactly equal to + 1.

-1

+1

v :

.,.
,

,,..

Fig.

If all the coordinate points

S.Sc:

Perfect Positive Correlation

in a scatter diagram are closely

in downward direction, then there is a strong

placed, and

negative correlation

if the

points are

between them and the

value of r w i l l be close to - 1 .

www. itmu niversityonline. erg

Page

124

Introduction to Quantitative T e c h n i q u e s

05.

eBook

Correlation

+I

-.

!.-)

.....

. .,-:,
,
.........
:.
. ,
.

....

r.-.i1...

..,_,..,._.,

...

,.,.

. . ..

Fig.

However,
that
and

there

if the direction
is

a weak

S.Sd:

Strong

is downward

negative

Negative Correlation

and

the

correlation

points are dispersed,

between

them

and

the

it can

value

be interpreted

of

r is

negative

is close to zero.

+l

...

.. . . .

.,. .

. ..

..

::

..

Fig.

S.Se:

Weak

If all the points in a scatter diagram


in

the

Fig.

5.Sf,

it

can

be

between the two variables and

www.itmuniversityonline.org

Negative Correlation

lie on the same line

interpreted

that

there

is

in downward
a

perfect

direction as shown

negative

correlation

the value of r is exactl y equal to - 1 .

Page

125

I n t r o d u c t i o n to Q u a n t i t a t i v e T e c h n i q u e s

05.

eBook

Correlation

-1

+1

'

......

.....

.....
x

Fig. S.Sf:

If

the

points

in

direction at all,

scatter

it can

be

Perfect

diagram

are

interpreted

Negative Correlation

randomly

that,

there

is

spread

on

the

no correlation

graph

between

and

have

them

and

value of r is equal to zero.

-1

+l

'
y ,


.
.
. .
... :
.
.
: .. . .
. . . ..
. . .

. . .: . . .
. .
. .
. . .
. . . . a. : . .





. . .
.
.
. .
..
\
. .
.
.
.. .
.,
.

Fig.

5.Sg:


. I

No Correlation

Advantages of scatter diagram

It is a non-mathematical and

basic method of finding correlation.

It is quick,

It provides an approximate correlation

simple, and easy to understand.


between any two variables.

Disadvantages of scatter diagram

It gives only a rough

idea about the correlation.

It fails to give the exact degree of correlation

www. itmu niversityonline. erg

between two or more variables.

Page

126

no
the

Introduction to Quantitative T e c h n i q u e s

05.

eBook

Correlation

Example 0 1 :

T h e following

data

gives

the

age

and

weight

of

the

players

in

state

football

team.

Prepare a scatter diagram for the given data.

Table 5 . S a :

Age

Weight

(In years)

(In kg)

29

60

31

60

27

65

30

70

28

62

35

63

28

64

29

74

32

73

30

69

Distribution of Age and Weight of 10

Players

Solution 0 1 :

Considering

age along

X-axis and

weight along

Y-axis,

the following

scatter diagram

can

be prepared.

so-

"'

-"

c:

60-

.c

40-

"'

;;;

;;:

20-

15

10

20

25

40

30

35

Age (In years)

Fig. 5 . S h :

Thus,

it

spread

may

be

noted

on the graph,

between

the

age

and

that

Scatter Diagram for Age and Weight

the

points

representing

the

nor do they follow a straight line.


weight

of

the

state

football

pairs of value are

not

randomly

This indicates that the correlation

team

is

non-linear

or

curvilinear

nature.

www.itmuniversityonline.org

Page

127

in

I n t r o d u c t i o n to Q u a n t i t a t i v e T e c h n i q u e s

05.

eBook

Correlation

5 . 6 Karl Pearson's Coefficient of Correlation


Correlation
variables.

analysis

used

to

identify

This degree of association

of correlation" and

The

is

most

widely

developed

by

is denoted

used

British

the

degree

is measured

of

association

between

by a numerical value called

any

two

"coefficient

by "r".

mathematical
biometrician,

formula
Karl

to

find

Pearson

the

(1867

coefficient

1936)

of

and

is

correlation
known

was

as

Karl

Pearson's coefficient of correlation method.

Cov(X, Y)
r=--

T h i s is the formula for Karl

Pearson's coefficient of correlation.

Note:

1.

Coefficient of correlation

2.

If r = + 1,

then there is

3.

If r = - 1 ,

then there is a perfect

4.

If r = 0,

Example

A sales
sales

+ 1, that is, - 1 ,; r ,; + 1.

perfect positive correlation.


negative correlation.

then there is no correlation.

02:

manager

observed

executives.

executives

and

figures

the

for

following

r lies between - 1 and

He

their
last

data and

that

wanted

to

some

analyze

performance.

quarter

are

fresh

The

given.

graduates

the

relation

experience
Calculate

between

of

the

perform

sales

better
the

than

experience

executives

coefficient

of

experienced

and

of

sales

their

sales

correlation

from

interpret the value.

www. itmu niversityonline. erg

Page

128

the

Introduction to Quantitative T e c h n i q u e s

05.

eBook

Correlation

Name of the
Years of

Sales in

Experience

Lakh

Sales
Executive

Pradeep

Aka sh

Nisha

Yogesh

Rahul

Sajnay

0.25

Abha

0.50

Nair

David

2
5

Yasin
.

Table 5 . 6 a :

1.

Distribution of the

so

Experience and their Sales Figures of Sales


Executives

Solution 0 2 :

The

years

of

experience

is

denoted

coefficient of correlation the following

Name of

Years of

Sales in

the Sales

Experience

Lakh

Executive

(x)

(y)

by

the

variable

and

sales

by

Y.

To

find

the

table is constructed:

{x - x)

(v- v)

(x-xf

Pradeep

1.6250

-0.4500

Aka sh

-1.3750

Nisha

Yogesh

Rahul

(v-vf

(x-xXv-v)

2.6406

0.2025

-0.7313

-0.4500

1.8906

0.2025

0.6188

-0.3750

-2.4500

0.1406

6.0025

0. 9188

3.6250

2.5500

13.1406

6.5025

9.2438

4.6250

2.5500

21.3906

6.5025

1 1 . 7938

Sajnay

0.25

-3.1250

-1.4500

9. 7656

2.1025

4.5313

Abha

0.50

-2.8750

-0.4500

8.2656

0.2025

1.2938

Nair

-2.3750

-1.4500

5.6406

2.1025

3.4438

David

1.5

-1.3750

-1.9500

1.8906

3.8025

2.6813

Yasin

1.6250

3.5500

2.6406

12.6025

5. 7688

33.75

34.50

67.4063

40.2250

39.5625

Total

Table 5 . 6 b :

Distribution of the Experience and

their Sales Figures of Sales

Executives

www.itmuniversityonline.org

Page

129

I n t r o d u c t i o n to Q u a n t i t a t i v e T e c h n i q u e s

05.

eBook

Correlation

From the table:

Lx=33.75
X

LX

LY=34.50

= 33.75

= 3.375

10

LY

v =

and

= 34. 50

3.45

10

Now, calculate the values

ofL

(x - x )2 ,

L (y

- YJ

and

L (x

xXv - v).

From the above table,

L (x

- x)2

= 67.4063

L(Y-YJ

= 40.2250

L (x

xXv-

v)= 39.5625

Place these values in

Karl

Pearson's coefficient of correlation formula:

L(x-xXy-Y)

r =

--.=="'===-==-===-

JL (x

XJL

(y- YJ

39.5625
=

=0.7598
,/ 6 7 . 4 0 6 3

40.2250

r=0.7598

This

is

positive

experience

of

correlation

executives

with

and

their

reasonably
sales

high

figures

for

degree
the

of association

last

quarter.

It

between
indicates

the
that

h i g h e r the experience of an executive, the better their performance is.

Properties of Coefficient of Correlation

The

coefficient

of

correlation of (X, Y)

correlation

is

symmetrical

= correlation of (Y,

The coefficient of correl

If the variables X and

ation

with

respect

to

and

that

X).

is independent of change of origin and

scale.

Y are independent then coefficient of correlation

www. itmu niversityonline. erg

Y,

r =

Page

0.

130

is,

Introduction to Quantitative T e c h n i q u e s

05.

eBook

Correlation

Limitations of Correlation Coefficient

A high

value of r does

between

them.

It

not always i m p l y that

may

also

imply

that

both the factors have

both

the

factors

are

h i g h correlation

dependent

on

third

factor.

In

some

instances,

the

two

factors

t h o u g h there may not be any

If

the

data

misleading

At

times,

and

not

It

is

exhibit

high

correlation

basis for the relationship

in

nature,

advisable

to

correlation

ascertain

coefficient

between them.

coefficient

logical

even

may

relationship

provide
between

prior to the study.

it

hence,

logical

homogeneous

information.

the factors,

is

may

can

be

difficult

to

interpret

the

significance

of

correlation

coefficient

it may be misunderstood.

Applications of Correlation

Correlation
indicative
analysis
related

has

and

wide

not

range

of application

exhaustive.

is

is applicable to any field,

to each other.
the

The

about

common

logic w h i l e identifying

is some

not

possible

relationship

possible

Whether

there

is

Whether the

return

relationship

The

type

of

give

an

The

applications

exhaustive

list.

are

only

Correlation

in

list of a p p l i c a t i o n s

situation

under

be

is to stimulate the

study.

One

must

use

the relationships.

regression analysis can

be used

between them.

to identify:
relationship

between

the

Price-Earnings

(P/E)

ratio

of

its profit.

of a stock is

increasing

or decrease in the returns of BSE

fields.

between any two variables,

any

company's stock and

to

provide the

to investigate the cause and effect relation

Correlation analysis is used

various

where any two variables under consideration could

intention to

thinking

If there

It

in

relation

that

exists

(Sensex)
between

or decreasing

with

respect

to

increase

or N i ft y .
profit

and

expenditure

of

R&D,

sales

revenue, etc.

Whether the productivity of employees increase with

Whether there
and

relationship

between

the

money

in training

spent on

hours.

employee welfare

satisfaction.

Whether there
and gold

is any

increase

is

any

relation

between

price, wholesale price and

The type of relation that BSE

www.itmuniversityonline.org

the

return

on

stock

and

inflation,

inflation

consumer price index.

(Sensex) and

gold

prices share between them.

Page

131

I n t r o d u c t i o n to Q u a n t i t a t i v e T e c h n i q u e s

05.

eBook

Correlation

Whether the dividend

per share increases or decreases with

respect to

increase or

decrease in earnings per share.

Whether

there

is

any

relationship

between

the

metal

expansion

rate

and

temperature.

The type

of

relation

that

is there

between

an

MBA

exam

score

and

performance

score on the job.

The relation

The

between the yield

relation

between

the

of a crop and the amount of fertilizers u se d .

change

in

bank

interest

rates

and

the

prices

of

real

estate.

Correlation
in

is also applied

accordance with

in stock market to analyze the change in the return of a stock

the change

in

the market

returns.

The

investor collects data on the

market and stock returns. An example is shown in Table 5.6c.

Period

Market Return

1.10%

-4.40/o

0.10%

-6.60/o

7.10%

6.00/o

3.00%

-0.50/o

3.70%

1 1 . 50/o

-1.80%

1 . 6 1 /o

0.60%

-2.30/o

-2.90%

15.40/o

-15.11%

- 1 4 . 90/o

12.56%

13.10/o

10
Table 5.6c:

To

analyze

market
has to
using

the

return,

change

in

correlation

Stock Return

the

Data of Market and Stock

return

analysis

identify the relationship

of a stock,

can

between

be

used

the

in
by

return

Return

accordance
the
on

with

investor.

stock and

the

change

For this,
the

the

market

Page

the

investor

return,

Karl Pearson's coefficient of correlation.

www. itmu niversityonline. erg

in

132

by

Introduction to Quantitative T e c h n i q u e s

05.

eBook

Correlation

5 . 7 Chapter S u m m a r y

The

correlation

termed

between

any

two

variables

Coefficient of correlation

With

reference to each

r lies between

other,

if any

by

reference

to

each

other,

if

- 1 and

+ l , that is,

-1

two variables

then there exists a positive correlation


With

measured

If

the

direction

direction,

any

two

the

densely

plotted

represents a strong

it represents a weak positive correlation


closely

If

the

it

placed

points

in

coordinate

represents a strong

widely

dispersed

scatter

points

in

If

all

the

direction,

points
then

in

there

scatter

the

+l.

ct irection,

same

in

fluctuate

in

the

opposite

scatter

diagram

is

in

upward

between the two variables.


are

widely

spread

from

each

between the two variables.


scatter

diagram

are

in

downward

between the two variables.

diagram

are

directed

downwards,

it

between the two variables.

diagram

exists

diagram

scatter

represents a weak negative correlation

in

negative correlation

points

in

r s

between them.

positive correlation

other,
the

directed

points

upward

direction,

it

of

fluctuate

variables

If the

If

value,

between them.

direction, then there exists a negative correlation

numerical

as "coefficient of correlation".

is

lie

on

perfectly

the

positive

same

line

correlation

and

in

an

between

upward
the

two

variables.

If

all

the

direction,

points
then

in

there

scatter
exists

diagram

perfectly

lie

on

the

negative

same

line

correlation

and

in

downward

between

the

two

variables.

If the

points are

randomly

there is no corre lation

The

existence

relation

The

of

spread

on

the graph and

have

no

ct irection at all, then

between the two variables.

correlation

does

not

necessarily

imply

the

cause

and

between any two variables.

Karl Pearson's formula to calculate coefficient of correlation r is given a s :

L(x- xXy- v)
r -,...aaa..-...;,_....,_

L(x-xfL(y-vf

www.itmuniversityonline.org

Page

133

effect

Rank Correlation
and

Regression Analysis

I T M

Introduction to Q u a n t i t a t i v e T e c h n i q u e s

06.

Rank

Correlation

and

eBook

R e g r e s s i o n Analysis

6 . 1 Introduction
This chapter

introduces the concept of measuring

variables through
regression,

and

rank correlation.

coefficient

between these variables.


its properties, and

Rank

correlation

Spearman,

in

factors

that

factors.

To

frequently
can only

For

satisfaction
scale.

This

ranking

year

of

Spearman's

It

in

is

useful

nature

correlation,

between

two or more

as

you will see

the

degree

of

relationship

how to fit a regression

line,

this concept.

by

British

when
there

ranks

explains

multiple

calculating

is

are

psychologist,

no

are

correlation

quantitative

assigned

variables that

the

not

to

Charles

the

measure

factors.

Edward
between

for

these

Researchers

measurable quantitatively

but

on the basis of some characteristics.

are

does

given

research

to
not

be

projects

measured,

where

the

consumer

data

is

preference

collected

according

measure the characteristic quantitatively

characteristics.

correlation

rank

which

developed

qualitative

many

levels

of the

coefficient

in

scale

the

across

was

1904.

qualitative

be ranked

instance,

determination

regression analysis,

coefficient

calculate
come

In

relationship

introduces the concept of regression,

solve problems to understand

the

are

of

It also

the

to

In

identify

correlation

is

the

such

the

most

situations,

relation

you

cannot

between

appropriate

but

two

measure

to

or
to

employee
an

ordinal

it facilitates the

use

Karl

variables.
identify

Pearson's
Therefore,

the

relation

between two qualitative variables.

In

conclusion,

between

rank correlation

paired observations,

After reading t h i s chapter,

is

a technique

with

you

will

that

measures

strength

of association

respect to their ranks.

be able to:

Define Spearman's rank correlation

Illustrate Spearman's rank correlation coefficient

Define regression analysis

Discuss the applications of regression analysis

State the different types of regression

Discuss the different methods of obtaining

State the properties of regression coefficients

Define the coefficient of determination and

Explain multiple regression

www.itmuniversityonline.org

the

regression

lines

its properties

Page

135

I n t r o d u c t i o n to Q u a n t i t a t i v e T e c h n i q u e s

06.

R a n k Correlation

and

eBook

Regression A n a l y s i s

6 . 2 Spearman's Rank Correlation


Definition

The

strength

of

association

rank correlation coefficient.

between

pairs

It is denoted

of

by

ranked

data

"p" (rho)

is

computed

by

Spearman's

given as follow:

6Ld,
p=1---

n (n

2
-

1)

Where,

p =

Rank coefficient of correlation

n = N u m b e r of items or i n d i v i d u a l s being
d = R,
R1

R2 =

ranked

Difference of ranks between

paired

items of two series

Rank of items with

respect to the first variable

R2 =

Rank of items with

respect to the second

Rank

correlation

whether

ranks

correlation

method
are

is

based

given

is categorized

or

are

into three

Case 1 :

when

ranks are given

Case 2 :

when

ranks are not given

Case 3 :

repeated

Case

1:

on

the

to

variable

ranks

be

or order of

assigned,

the

the

observations.

process

of

Based

identifying

on

rank

possibilities:

ranks

when ranks are given

Example 0 1 :

company

training,
months,

gave

they
they

took
were

rank correlation

month-long
a

test

rated

between

and
with

training
were

to

10

ranked,

respect

to

sales
based

their

on

sales

t h e i r performances during

representatives.
their

the

performances.

performance.

training

At

and

Find

the

their sales

end
After

performance

Sales
2

10

Representatives
Rank Obtained

in

10

10

Training
Rank Based on Sales
Performance
Table 6 . 2 a :

Ranks of Performance of 10 Sales Representatives

www.itmuniversityonline.org

eight

Spearman's

after e i g h t months.

of the

Page

136

Introduction to Q u a n t i t a t i v e T e c h n i q u e s

06.

Rank

Correlation

Solution

Let

and

01:

us test whether there

obtained

eBook

R e g r e s s i o n Analysis

in

the

training.

is any

relation

Since

the

between

given

data

the
is

performance

in

rank

order,

rating
we

and

need

the
to

rating

use

the

formula of rank correlation coefficient.

Rank

Rank Based

Sales

Obtained in

on Sales

Representatives

Training

Performance

(R1)

(R2)

d = R 1 - R 2

d'

-1

-2

-2

10

10

-2

10

1
Id'=

Table 6 . 2 b :

Ranks of Performance of

24

10 Sales Representatives

6 Z: d
Rank correlation coefficient:

p =

1 - 'c' -n(n

- 1)

Here, the sum of square of the deviations of paired

Therefore,

ranks is obtained a s :

:1:d

24

6
24
p - 1x
10 x 99

= 1 - 144
990
= 1 - 0 . 1 5

0.85

This shows that there is a h i g h degree of association


training and

(85%)

between the performance

the sales performance of the representatives.

www.itmuniversityonline.org

Page

137

in

I n t r o d u c t i o n to Q u a n t i t a t i v e T e c h n i q u e s

06.

R a n k Correlation

Case

2:

and

eBook

Regression A n a l y s i s

when ranks are not given

Example 0 2 :

Find

out

Spearman's

rank

correlation

between

internal

and

of postgraduate students' in a college from the data given

external

assessment

marks

below.

Internal Assessment

External Assessment

Marks

Marks

Name of the
Student
(Out of 100

(Out of 1 0 0 )

52

so

62

71

74

75

45

43

51

59

59

65

48

51

35

29

61

36

Table 6 . 2 c :

Internal and

External Assessment Marks of Postgraduate Students

Solution O 2:

In

this

example,

ranks

ascending or descending

The

internal

magnitude.
62, and

That

is,

not

given.

Therefore,

rank

data
1 to

values

the

will

highest

can

be

followed

to

be

assigned

value

The rank 9 is to be assigned

approach

first

assign

ranks,

either

in

an

order of m a g n i t u d e .

assessment

so o n .

similar

are

74;

ranks

rank

2 to

in
the

descending
second

to the lowest data value,

assign

ranks

to

data

order

highest

of

value

that is, 35.

values

in

the

external

assessment.

Following

table

is

used

to

obtain

the

rank

correlation

coefficient

between

internal

external assessment marks of postgraduate students.

www.itmuniversityonline.org

Page

138

and

Introduction to Q u a n t i t a t i v e T e c h n i q u e s

06.

Rank C o r r e l a t i o n a n d

eBook

Re g re s s io n Analysis

Internal

External

Name
Assessment

Rank

Assessment

Rank

Marks (Out

R1

Marks

R2

of the

d = R 1 - R 2

d'

Student
of 1 0 0 )

(Out of 100 )

52

so

-1

62

71

74

75

45

43

51

59

59

65

48

51

35

29

61

36

-5

25

Total
Table 6 . 2 d :

Internal and

36

External Assessment

Marks of Postgraduate Students

After assigning the ranks, the squared difference of the ranks are obtained.
2

Here,

ld

= 36
2

Rank correlation coefficient,

6 I; d
p = 1--
n(n2 - 1)

1 - 6 x 36
9 x 80

1 - 216
720

This is a fairly

1 - 0.3

0.7

h i g h degree of association so there is a positive relationship between the

internal and external assessment marks of postgraduate students.

Case 3 :

when

W h i l e assigning
rank can

ranks are repeated

ranks to data values,

be assigned

to

if any of the data values are repeated,

similar observations.

Suppose the

two observations at

8'" place are the same, the rank assigned to each of these two should
7
R a n k =

an average

be:

= = 7 . S

www.itmuniversityonline.org

1"

Page 139

and

I n t r o d u c t i o n to Q u a n t i t a t i v e T e c h n i q u e s

06.

R a n k Correlation

In

case of repeated

regular

formula

and

ranks,

of

eBook

Regression A n a l y s i s

for example,

Spearman's

obtain the correct formula,

rank

the term

if the

value

correlation

m(m' - l)

is

repeated

coefficient

is added

for "rn" times,

needs

to }:d

to

be

for eac h

then

the

adjusted.

To

repeated

value.

12

Example 0 3 :

Calculate

the

rank

correlation

coefficient

between

the

variables

and

for

the

data

given:

65

66

67

67

68

69

70

72

67

66

65

68

72

72

69

71

Table 6 . 2 e :

Observations of the Variables X and Y

Solution O 3 :

Here,

let

series

X,

highest

us

assign

we assign

value

that

ranks
rank

is

70,

to

the

observations

1 to

the

highest

and

continuing

in

descending

value that
in

the

is

same

order

of

magnitude.

72,

assign

rank 2 to

way,

assign

rank

4 to

the

For

second

data

value

68.

It

is

to

be

noted

that,

rank to both, which

5+ 6

the

value

67

is

repeated

is the average of the

twice,

therefore,

next two ranks,

5 and

we

assign

common

6.

11

Rank of 6 7 ; -2-;

5.5

2;
Continuing

with the next rank, we assign

For series Y, the highest value 72


and

rank 7 to 66 and

8 to 65.

repeats twice, so we assign the average of ranks

2 to both the values.


1

Rankof72;

; ; 1 . 5

2
Continuing

Following
X and

with the

table

2
next rank, we assign

is used

to calculate the

rank 3 to 7 1 ,

rank 4 to 69, and

so on.

rank correlation coefficient between the variables

Y.

www.itmuniversityonline.org

Page

140

Introduction to Q u a n t i t a t i v e T e c h n i q u e s

06.

Rank

Correlation

and

eBook

R e g r e s s i o n Analysis

Rank R1

Rank R2

d = R1

d.

- R2

65

67

66

66

67

5.5

65

-2.5

6.25

67

5.5

68

0.5

0.25

68

72

1.5

2.5

6.25

69

72

1.5

1.5

2.25

70

69

-2

72

71

-2

4
Id2 = 2 7 . 0 0

Table 6.2f:

Observations of the Variables X and Y

After assigning the ranks, a new adjusted

correlation.

So,

in this case,

the term

formula w i l l

m('

- l)

be used

to find

has to be added

rank coefficient of

twice,

for adjusting

2
ranks of 67 and

72.

Rank correlation coefficient for repeated

[Ld 2

ranks

is:

+ m 1 ( m , 2 - 1 ) + m2(m/ - 1 ) )
12

12

p = 1 - ------------
2

n(n

1)

Where,
rn,

Number of times 67

repeated

rn, =

N u m b e r of times 72

repeated

For series X :

Since the value 67

m 1 ( m , 2 - 1 ) = 2(22
12

repeated

-1)

= _1_

12

two times,

to

hence by taking

m i = 2, we have

to add

}:d2.

For series Y :

Since the value 72

repeated

two times hence

by taking

rn, = 2, we

have to add

2
m1(m1
12

-1)

2(22

-1)

12

_1_

to

}:d2.

www.itmuniversityonline.org

Page 1 4 1

I n t r o d u c t i o n to Q u a n t i t a t i v e T e c h n i q u e s

06.

R a n k Correlation

By substituting

and

eBook

Regression A n a l y s i s

the values

in the formula, we get:


2

(L

6
Rank correlation coefficient

d' + 2(2

1J + 2(2

12
p = 1 - ----- -(- ,_
-_
)
8

1J)

12
_

6(2 7 + _!. + .!.)


= 1-

8 x63
= 1 - 168

504
= 1-0.3333
= 0.6667

Here

p = 0 . 6 6 6 7 i m p l i e s that, there is a h i g h degree positive association

between X and

Y.

6 . 3 Regression Analysis
Regression
define
and

the

plays an

important

relationship

education,

cost

role

between
of

in

two

the field

or

investment

more
and

of management.
related

profit,

variables,

demand

It

is a tool

such

and

as

that

level

supply,

helps to

of

income

income

and

expenditure, etc.

According

to

establish

the

functional

the

definition

nature

of

by

Ya

Lun

relationship

Chow,

Regression

between

variables,

analysis
that

attempts

is,

to

study

to
the

relationship between the variables and thereby provide a mechanism

for prediction or forecasting.

6.4 Applications of Regression Analysis

Regression analysis studies the relationship between two or more variables.

It enables a person

to find

the value of a dependent variable from

an

independent

variable.

It

is

useful

for

the

calculation

2"

determination "r

or "R

coefficient

of

correlation

"r"

and

coefficient

of

".

It estimates the statistical


consumption,

of

curve

between

the variables

like demand,

supply,

price, etc.

www.itmuniversityonline.org

Page

142

cost,

Introduction to Q u a n t i t a t i v e T e c h n i q u e s

06.

Rank

Correlation

Using

and

regression analysis,
price and

eBook

R e g r e s s i o n Analysis

it

is possible to

identify the relation

Gold

Wholesale price and consumer price index

Amount of fertilizers and

Amount of rainfall and

Income and

Volume of production and

Profit and

Income and

its sales

increase or decrease in crop

crop

production

production

expenditure
percentage of defects

expenditure on advertisement
insurance of a policy

Distinguish between Correlation and

Distinguish

between:

between correlation and

holder

Regression

regression

is given

in the following

Correlation

It is the relationship

table.

Regression

between two or

It is a mathematical measure,

which

shows the average relationship

more variables

between two variables


It measures the degree of relationship

It is the measure of the nature of the

between two variables

relationship

It does not study the cause and

effect

of a variable

between two variables

It studies the cause and


relationship

between variables and

establishes a functional
The coefficient of correlation
relative measure and
+1

and

is a

it lies between

-1

effect

relationship

The coefficient of regression


absolute figure.
of an

If you

is an

know the value

independent variable,

you can

calculate the value of a dependent


variable
It is

not of much use for further

algebraic treatments
Table 6 . 4 a :

It is very useful for further algebraic


treatments

Distinguish between Correlation and

www.itmuniversityonline.org

Regression

Page

143

I n t r o d u c t i o n to Q u a n t i t a t i v e T e c h n i q u e s

06.

R a n k Correlation

and

eBook

Regression A n a l y s i s

6 . 5 Types of Regression

Simple and

In

simple

Multiple

regression,

dependent

and

relationship

Regression

only

other

is

two

an

between demand

Linear and

When

one

called

are

independent
and

variables are taken for study,

variables

supply.

taken

for

variable.

For

study,

which

example,

On the other hand,

it is a m u l t i p l e

in

the

when

one

is

functional

more t h a n

two

regression.

Non-linear Regression

variable

linear

changes

regression.

with

In

another

non-linear

variable

in

regression,

some

as

one

constant

ratio,

variable

varies,

it

is

the

other variable does not change in a constant ratio.

Partial and Total Regression

The

regression

consideration
other

is

and

variables

called

the

partial,

relation

constant.

simultaneously,

In

in order to find

total

deviations

drawn

of the

by this method

Scatter Diagram

In

this

drawn

method,
passing

non-linear.

It

shows the

lines

o
f

Regression

variables
all

the

is

are

studied,

variables

taken

into

keeping
are

the

studied

them.

regression

lines.

values

in

from

such a way that the sum of squares of


the

fitted

line

shall

be

least.

The

line

is known as the line of best fit.

Method

values

through

of

variables

the

plotted

are

ability and

plotted

points.

This

not used,

on

graph.

regression
as

free

line

can

it varies from

hand
be

person

line

linear
to

is
or

person

interpretation of the result.

Line of Best Fit

relationship

regression

variables

Lines

This technique is generally

Line or

two

regression,

line is fitted

observed

in their decision-making

Regression

only

more

Method

In this method, a regression


the

or

the relationship among

are the methods of obtaining

Least Square

two

between

6 . 6 O b t a i n i n g Regression
Following

if

between

two variables.

If two variables are

X and

Y,

then

two

are:

line of X on

Y:

this

line gives

the

best

estimate

for the

value

of X for

any given value of variable Y.

www.itmuniversityonline.org

Page

144

Introduction to Q u a n t i t a t i v e T e c h n i q u e s

06.

Rank C o r r e l a t i o n a n d

Regression

eBook

R e g r e s s i o n Analysis

line of Y on

X:

this

line gives the

best

estimate

for the value of Y for

any given value of variable X.

Regression

Equations

It is an algebraic representation of a regression

lines. There are two forms of regression

lines.

Regression

It

is

Line of Y on X

obtained

by

using

the,

"Least

Square

Method."

The algebraic

form

of the

line

of

regression of Y o n X i s :
.............

Y=a+bvxX

Where, "a" is Y-axis intercept and

Let

(X1,

Then,

Y1),

(X 2,

Y2)

(X,,

... ,

the constants a and

Y,)

bvx

(1)

''bvx" is slope value of the

regress ion

be "n" pairs of observations on

in equation

( 1 ) , can

be obtained

line.

the variables X and

by solving

following

Y.

two

normal equations:

LY=na+bvxLX

L XY

The

aL x

values

equations

+ bvx

of

(2)

...........

L x'

'2.Y,

'2.X,

and

(3)

(2)

(3)

'2.XY,

and

'2.X'

simultaneously,

can

be

obtained

constants

"a"

from

and

the

"bvx"

given

are

data.

Solving

calculated

by

the

following formulae:

If assumed

mean

method

is used,

the constant

bvx

is calculated

by

using

the following

formula:

bvx

nLdxdv - L d x L d v
= ----=="c-nI; d x 2 - {z: d x J 2

Where,

dx = (X - A)

= Deviation of the observations from assumed

mean A for variable X

dv = (Y - B)

= Deviation of the observations from assumed

mean B for variable Y

www.itmuniversityonline.org

Page 1 4 5

I n t r o d u c t i o n to Q u a n t i t a t i v e T e c h n i q u e s

06.

R a n k Correlation

Regression

It

gives

and

eBook

Regression A n a l y s i s

Line of X on Y

the

best

estimate

for

the

value

of

for

any

given

value

of

variable

X.

The

algebraic form of the line of regression of X on Y is:


X=c+bxvY

Where,

"c"

determined

(4)

is

X-axis

intercept

by solving

and

"bxv"

equations

LX,

of

(5)

value

of the

regression

line.

Both

are

(5)

. . . . . . . . . .

values

slope

following two normal equations:

LX=nc+bxvLY

The

is

and

LY,
(6)

LXY,

and

(6)

LY'

simultaneously,

can

be

obtained

constants

c and

from
bxv

are

the

given

data.

calculated

by

Solving

using

the

following formulae:

If assumed

mean

method

is

used,

the

constant

bxv

is

calculated

by

using

the

following

formula:

bxv

nLdxdv - L d x L d v
= --=-=nL d v 2

- CT; d v ) 2

Example 04:

Find

the equation of regression

Y for given value X =

line Y o n

X from the following

data.

Estimate the value of

53.

x
y

Table 6 . 6 a :

Observations of Variables X and Y

Solution 0 4 :

Finding

the

equation

of

regression

line

on

means

to

find

values

of

slope

bvx

intercept a of the l i n e of regression:


Y = a +

bvxX

www.itmuniversityonline.org

Page

146

and

Introduction to Q u a n t i t a t i v e T e c h n i q u e s

06.

Rank

First,

Correlation

we

count

the

columns XY and

and

number

eBook

R e g r e s s i o n Analysis

of

values

"n",

Here,

5.

Then,

For the calculation, we construct Table

calculate

the

values

of

6.6b.

xv

so

3.1

155.0

2500

51

3.5

178.5

2601

53

3.3

174.9

2809

55

4.0

220.0

3025

56

3.9

218.4

3136

17.8

IXV = 946.8

IX=

IV=

265

Table 6 . 6 b :

From

IX

14071

Observations of Variables X and

the table:

}:X

= 265

}:Y

17.8

}:XY = 946.8
2

}:X

14071

Substitute the above values in the formula of a slope.

Slope, bYX

" 'X:'f - " X"

=
,:_,

,:_,

,:_,

"L

x2 -

o: x)2

Sx 946.8-265x 17.8

=-------2

5 x 1 4 0 7 1 - (265)
4734-4717
=------

70355 - 7 0 2 2 5
17
= - = 0.13

130

Now,

substitute

the

value

of

bvx

0.13,

in

the

formula

Ly=

na + byx

LX

to

compute

the value of intercept a.

Intercept,

a=

LY -

byx

LX

n
1 7 . 8 - 0.13(265)
=----

5
- 16.25
=---

-3.33

www.itmuniversityonline.org

Page

147

I n t r o d u c t i o n to Q u a n t i t a t i v e T e c h n i q u e s

06.

R a n k Correlation

Now,

and

line Y o n X.

substitute these values in the equation of regression

Regression equation:

That

eBook

Regression A n a l y s i s

Y = a + bvxX

-3.33 + 0.13X

is, Y =

Therefore,

the estimated

Y =

-3.33

+ 0.13(53)

-3.33

+ 6.89

value of Y for the given value of X =

53

is:

= 3.56

Hence, the estimated

value of Y for the given value of X

53

is Y

3.56

Example 0 5 :

Using

the following data find out the equations of regression

X on Y

Yon X

line:

v
Table 6.6c:

Observations of Variables X and V

Solution O 5 :

Following

table is used

to find out the equations of regression

l i n e X on Y and

Y o n X.

xv

27

20

729

400

540

33

16

1089

256

528

34

28

1156

784

952

49

31

2401

961

1519

so

36

2500

1296

1800

so

46

2500

2116

2300

51

54

2601

2916

2754

IX=294

IV=

231

Table 6 . 6 d :

www.itmuniversityonline.org

IX

12976

IY

= 8729

IXY =

10393

Observations of Variables X and V

Page

148

Introduction to Q u a n t i t a t i v e T e c h n i q u e s

06.

Rank

Correlation

and

eBook

R e g r e s s i o n Analysis

The equation of regression line X on Y is:


X=c+bxvY

n I: X Y - I: X I: Y
Where,

bxv

"L
bxv

I: X - b x v I: Y
c = =-

and
y2 -

(I: Y)

2
n

= (7 x 1 0 3 9 3 ) - (294 x 2 3 1 ) =

0_62

(7 x 8729) - (231)2

C =

I: X - b x v I: Y

= 294-0.62x231

=
21.54

Hence,

the equation of regression

line X on Y is:

X=c+bxyY
X = 21.54

+ 0.62Y

The equation of regression line Y o n X is:


Y=a+byxX

n I: X Y - I: X I: Y
W h ere,

b vx

d
an

I: Y - b v x I: X
a

= ----

n I: X 2 - (I: x )

_ nI:

xv -

I: XI: Y

(7 x 1 0 3 9 3 ) - (294 x 231)

(7 x 12976) - (294)2

=
vx

nI: x2 _ (I: x)2

I: Y - b v x I: X
a =

2 3 1 - ( 1 . 1 x 294)
=

= - 1 3 . 2

Hence,

1.1

the equation of regression

line Y o n X i s :

Y=a+byxX
Y =

-13.2

Arithmetic Mean

1.lOX

Method to Find

If the values of variables are


simple

linear

equation

is

large,

Regression

Lines

then calculation of the regression equation through a

cumbersome.

Therefore,

the

arithmetic

mean

method

is

for large v a l u e s .

Regression equation of Y o n X i s :

(v - y) =

r cry

(x -

x)

crx

www.itmuniversityonline.org

Page

149

used

I n t r o d u c t i o n to Q u a n t i t a t i v e T e c h n i q u e s

06.

R a n k Correlation a n d

I (X

cry
byx

- X)(Y - Y)

r - = =--=-

eBook

Regression A n a l y s i s

(X - X)

Regression equation of X on Y is:

(x-x)= r(v-v)
cry

cr x
- r

b
XY

(X - X)(Y - Y)

-=----

I (Y

- Y)2

Where,

X and

r crx

= Arithmetic means of variables X and Y,

= The regression coefficient of

respectively

X on Y

cry

r cry

= The

regression coefficient of Y on X

crx
r = Correlation coefficient between X and Y
2

cry = Standard deviation of Y =

JI (Y n-

Y)

ex = Standard deviation of X =

JI (X n-

X)

Example 0 7 :

Using

the following data find out the equations of regression l i n e :

Yon X

X on Y

x
y

Table 6 . 6 e :

www.itmuniversityonline.org

Observations of Variables X and Y

Page

150

Introduction to Q u a n t i t a t i v e T e c h n i q u e s

06.

Rank

Correlation

Solution

and

07:

(X-X)

( Y - Y)

(X - X)2

(Y - Y)2

(X - X)(Y - Y)

11

12

-29

-33

841

1089

957

19

24

-21

-21

441

441

441

29

34

-11

-11

121

121

121

41

44

-1

-1

49

59

14

81

196

126

61

68

21

23

441

529

483

70

74

30

29

900

841

870

280

315

2826

3218

2997

Table 6.6f:

From

eBook

R e g r e s s i o n Analysis

the above table:

X =

Observations of Variables X and Y

280
x =

= 40

and

y =

Regression equation of Y on X is:

Y =

(y - Y) = r "v (x -

315

= 45

x)

"x
ay

L (X

- X)(Y - Y)

r- =

2997
=

"x

(X - X)2

= 1.0605
2826

Therefore, ( Y - 4 5 ) = 1 . 0 6 0 5 ( X - 4 0 )
Y - 45 = 1 . 0 6 0 5 X - 4 2 . 4 2
Y = 2 . 5 8 + 1.0605X
Hence,

the regression equation of Y o n

Regression equation X on Y is:

X is:

(x- X ) = r

"X

Y = 2. 58 + 1 . 0 6 0 5 X

(y - Y)

ay
-

r _<J_)(_ =
ay

( X - XH't' - Y)

L (Y

Therefore,

= 2997

- Y)2

= 0.9313

3218

(X - 40) = 0 . 9 3 1 3 ( Y - 45)

X - 40 = 0 . 9 3 1 3 Y - 4 1 . 9 1
X = 0.9313Y-1.91
Hence,

the regression equation of X on Y is:

www.itmuniversityonline.org

X = 0.9313Y - 1 . 9 1

Page

151

I n t r o d u c t i o n to Q u a n t i t a t i v e T e c h n i q u e s

06.

R a n k Correlation

and

eBook

Regression A n a l y s i s

Properties of Regression Coefficients

The value of regression coefficients lies between

The

correlation

coefficient

between

two

of the regression coefficients, therefore,

X and

If the variable

to co.

-ro

variables

is

equal

to

the

geometric

mean

r = Jbxybyx

Y are independent,

then the

regression

coefficients are equal

to zero.

If one of the regression coefficients is greater than


That

is,

if bxv >

1 then

bvx

< 1 and

if bxv <

1,

then the other is less than

1 then bvx >

1.

1.

Example 08:

If ex = 3,

bxv = 0. 94, and

bvx = 0. 98, then f i n d :

Correlation coefficient

Standard

deviation of Y

Solution 0 8 :

r = Jbxybyx

r = .J 0 . 9 4 x 0 . 9 8

Standard

bxv

= 0.96

deviation of Y:

"X
= r

cry

3
0.95=0.96xcry
0 . 9 5 cr y

= 0.96 x 3

Therefore,

cry

= 3.03.

6 . 7 Assumptions of L i n e a r Regression

The

relationship

coefficient can

between

two variables

be m u l t i p l i e d

nor divided

must

be

linear,

The independent variable X is not random.

The expected

The variance of the error term

The covariance between the error terms is assumed

The distribution of the error term

www.itmuniversityonline.org

is,

neither

regression

by another regression coefficient.

value of the error term

that

is assumed

to

be zero.

is constant for all observations.

is

to be zero.

normal.

Page

152

Introduction to Q u a n t i t a t i v e T e c h n i q u e s

06.

Rank

Correlation

and

eBook

R e g r e s s i o n Analysis

6.8 Coefficient of Determination


The square of correlation coefficient
denoted

regression

Where,

by

or R

line.

and

and

is

It is given

the

r is termed

percent

of

by the following

Y are the means of X and

as the coefficient of determination.

variation

that

can

be

explained

It

is

by

the

in

the

formula:

Y,

respectively.

O'x =

L, ( Y - Y )
and

O'y

6.9 Properties of Coefficient of Determination

The

coefficient

of

dependent variable, which

The

coefficient

of

implies

it

that

is

the

proportion

of

the

variation

is predictable from the independent variable.

determination

between X and Y and


2

determination,

is

ranges from
the

the
O to

dependent

square

of

the

correlation

coefficient

(r)

1.
variable

cannot

be

predicted

from

the

independent variable.

i m p l i e s that the dependent variable can

be

predicted

without error from

the

independent variable.

The value of r' indicates the extent to which the dependent variable
r'

0.30

remaining

The

means

scatter

of

data.

plot,

variables.

30%

of

the

70% of the total variation

coefficient

represents

that

it

determination

If the
means

If the

line

line

on

is

regression
that

the

is away

in

is

predictable

from

and

the

in Y remains unexplained.
a

line

line

from

variation

is predictable.

is

the

measure
passes
able
points,

of

how

exactly

to
it

well

through

explain

all

means

that

the

regression

every

the

point

variations

the

on

line
the

among

l i n e explains

less

variation.

The

regression

can

be

graphically

represented

as

shown

in

the

following

figure.

www.itmuniversityonline.org

Page

153

I n t r o d u c t i o n to Q u a n t i t a t i v e T e c h n i q u e s

06.

R a n k Correlation

and

eBook

Regression A n a l y s i s

Regression line: Y= a+bx

'

.,

..

'
'

'

'

Fig.

The following

6.9a:

Regression

example shows the computation

Line Y o n X

of regression

line Y o n X and

coefficient of

determination.

Example 0 6 :

Five

randomly

selected

their statistics course.

students

took

The following

mathematics

aptitude

test

before

table shows the scores of students,

they

started

X represents the

scores of the aptitude test and Y represents the scores of statistics:

Student

Table 6 . 9 a :

Find

the

based

92

75

84

77

66

88

85

75

66

70

Students' Scores in Aptitude Test and in Statistics

regression

equation

that

predicts

the

performance

of

statistics,

on the aptitude scores.

If a student scores 85
to get

linear

in the aptitude test, what score would

you expect h i m or her

in statistics?

How well does the regression equation fit the data?

www.itmuniversityonline.org

Page

154

Introduction to Q u a n t i t a t i v e T e c h n i q u e s

06.

Rank

Correlation

and

eBook

R e g r e s s i o n Analysis

Solution 0 6 :

Following

table is used

to obtain

regression

line.
-

Student

(Y - Y)

(X -

(X - X)(Y - Y)

(Y - Y)

XJ'

92

88

13.2

11.2

174.24

125.44

147.84

75

85

-3.8

8.2

14.44

67.24

-31.16

84

75

5.2

-1.8

27.04

3.24

-9.36

77

66

-1.8

-10.8

3.24

116.64

19.44

66

70

-12.8

-6.8

163.84

46.24

87.04

394

384

382.8

358.8

213.8

78.8

76.8

Total

Mean

Table 6 . 9 b :

Students' Scores in Aptitude Test and in Statistics

The regression equation


Now,

(X - X)

we have to find

is of the form:

Y = a +

bvxX

out the coefficients of regression a and

bvx, which

is calculated

as:

a = Y -bX
a =

76.8 - 78.8b

I (x

L (x

vx -

= 0.56

(2)

the value of bvx in equation

( 1 ) , we get:

32.67

(3)

Therefore, the regression equation

In

...........

xf

382.8

Substituting
a =

(1)

xXv - v)

= 213.8
YX

........

the

example,

The dependent

the

independent

variable Y is the

aptitude test, the estimated

+ 0.56

y =

32.67

y =

80.272

Once

you

variable

have

Y for

any

is, Y = 3 2 . 6 7

variable

student's

is

+ 0.56X

the

statistics

statistics score would

student's
score.

score

in

the

If a student

aptitude

scored

85

test.

in

the

be:

85

regression
value

of

determination we proceed

equation,

independent

you

can

variable

calculate
X.

the

For the

value

of

calculation

the

dependent

of coefficient

as follows:

To check how well the regression equation fits the data,

www.itmuniversityonline.org

2.

we have to compute R

Page 1 5 5

of

I n t r o d u c t i o n to Q u a n t i t a t i v e T e c h n i q u e s

06.

R a n k Correlation

and

2
R

= ji

L (X

X)

x (Y -

eBook

Regression A n a l y s i s

Y))

= ji

( 2 1 3 . 8))
74.12

cr x cr y

R2 = { 4 2 . 7 6 } 2 = 0 . 3 3
74.12

2
Where

JL (xN-x)

ox

2
= 8.7499

The coefficient of determination


the statistics grade is based

Standard

The

will

cry

is equal to

JL (v v)

= 8.4711

0.33,

implies that

which

33%

of variation

in

on the mathematics aptitude scores.

Error of Estimates

regression

values

and

(xs.

equation,

v.).

(x,,

be Y = y, and

a - bx,

y,)

Y = a

...

(x,,

the

bX

y,)

the estimated

determines

is an

estimated

of variables,

value for x, w i l l

error

committed

form

X and
be

while

Y.

of the sample of "n"

For X =

= a +

xr,

pairs of

the observed

value

b x . : Therefore, the difference

estimating

the

value

of

by

the

regression equation.

In

simple

regression

for all observed

analysis error variance

values to the total

I; ( y , - y , ) 2

is

the

ratio of the

number of samples, that is,

sum

of squares of errors

sample size n.

I; ( Y ; - a - b x ) 2
1

cre

===

This

is also called

a residual variance. The square root of variance is called the standard

error of estimates.

mean ( Y )

This

The variance of deviations of estimated values

y,

from the sample

is given a s :

equation

is

known

as

variance

of

the

values

of Y

explained

by

the

regression

equation of Y o n X.

www.itmuniversityonline.org

Page

156

Introduction to Q u a n t i t a t i v e T e c h n i q u e s

06.

Rank C o r r e l a t i o n a n d

The

coefficient

analysis.

The

of

determination

coefficient

eBook

R e g r e s s i o n Analysis

of

measures

determination

(r

effectiveness
2

lies

or

between

reliability
and

of

The

1.

regression
greater

the

value of r', the better the effectiveness or reliability of the regression equation.
2

Explained

variation

r' = =
2

Total

oe

variation

6 . 1 0 M u l t i p l e Regression
In

simple

regression,

independent
function

variable

of more than

between them

The linear

dependent
X.

Whereas,

one

is measured

relationship

in

is

multiple

independent

expressed

regression

variable,

say

X1,

as

the
X2,

function

dependent

X3

and

of

single

variable

the

is

relationship

by the m u l t i p l e correlation coefficient.

between

and X, can be expressed

variable

dependent variable Y and

two

independent variables

X1

in the form of a regression equation of Y o n X,, X, as:

Y = b0 + b 1 X 1 + b 2 X 2

Where,

b0 = Intercept of the line


b,,

b2 = Regression coefficients or constants

The
least

values

of constants

squares.

Here,

bo,

b1,

and

b2 are

estimated

with

the

the formulae to calculate the values of bo,

help
b,,

of the

and

principle

of

b, are obtained

by solving the normal equations simultaneously.

The normal equations are given as follows:

IY

nb0 + b 1 I X 1 + b 2 I X 2

LX1Y

LX2Y

The

b0LX1 +b

b0LX2

regression

LX! +b2LX1X

+b1LX1X2 +b

constants

b0,

LX

b 1,

and

b2

can

be

thus

obtained

by

using

the

mentioned formulae.

www.itmuniversityonline.org

Page 1 5 7

below

I n t r o d u c t i o n to Q u a n t i t a t i v e T e c h n i q u e s

06.

R a n k Correlation

and

_ ( 1: Y X 1
1

2
-2
2
-2
- 2
( I: X 1 - n X 1 ) ( I: X 2 - n X 2 ) - ( I: X 1 X 2 - n X 1 X 2 )

_ (1: YX2

-2

- n Y X 2 ) ( 1: X1

2
-

b0

-2

- n Y X 1 ) ( 1: X 2 - n X 2 ) - ( I: Y X 2 - n Y X 2 ) ( I: X 1 X 2 - n X 1 X 2 )

eBook

Regression A n a l y s i s

(1:

Xt -

- n X 1 ) - (1: YX1

x -

nXtHI

- -

- n Y X 1 ) ( 1: X 1 X 2

n X ) - (1: X 1 X 2

- nX1X2)

- nX1X2>2

=Y-b1X1-b2X2

Multiple Coefficient of Determination


It

is

defined

as

the

ratio

of

variation

explained

by

the

variation of the dependent variable Y. This is denoted

by

regression

equation

and

total

R2 =

Therefore,

Explained
Total

Where,

the

total

variation

variation

variation

in

in

is

eq u a l

equation plus the variation unexplained


Total variation
-

= explained

L ( Y ; - Y)

L(Y,

Y =

in

to

Yl

L (Y;

L (Y;

- Y)

the

variation

explained

by

the

regression

by the regression equation, that i s :

+ unexplained variation

variation

+ 1: ( Y ; - Y ; )

- Y)

Thus, above equation of R

can

be represented as follows:
2

_ Unexplained

variation

in

Y =1-L(Y;-Y;)

1
Total

The

square

root

variation

of

2,

in

where

L(Y, - Y ) 2

is

known

coefficient that explains the correlation


derived

as

the

coefficient

of

multiple

between the dependent variable and

from the m u l t i p l e regression equation.

It lies between O and

correlation
its estimate

1.

Example 09:
The owner of ten
of next year's
the current
given

projected

year's

below.

stores

sales

in a supermarket wishes to forecast

the

sales of vegetarian and

non-vegetarian

(in

profit

crores)

and

the

net

(in

net

food

crores)

profit with
items.

of a l l

the

the

The data for


ten

stores

Also obtain the coefficient of multipl e correlation coefficient.

www.itmuniversityonline.org

help

Page

158

is

Introduction to Q u a n t i t a t i v e T e c h n i q u e s

06.

Rank

Correlation

and

eBook

R e g r e s s i o n Analysis

Supermarket

Net Profit

Store

Sale of Veg.

Sale of Non-veg.

Food Items

Food Items

x,

X2

5.2

22

5.1

13

5.2

20

5.7

22

5.3

17

6.6

23

5.6

25

8.4

27

5.5

26

10

6.5

20

5
.

Table 6 . l O a :

Current Year's Sales and the

Net Profit of All Ten Stores in a

Supermarket

Solution

09:

The multiple regression equation can

be written as:

b0

+ b1X1

+ b2X2

Where,
Y =

Net profit

X1 = Sales of vegetarian food

items

X2 = Sales of non-vegetarian food


bo,

b..

items

ha = Regression constants and can be obtained

www.itmuniversityonline.org

by the

principle of least squares

Page

159

I n t r o d u c t i o n to Q u a n t i t a t i v e T e c h n i q u e s

06.

R a n k Correlation a n d

eBook

Regression A n a l y s i s

Following table is used to obtain the multiple regression equation.


Supermarket
y

X1

x'1

X2

X1Y

xl

X2Y

X1X2

Store

5.2

22

484

114.4

20.8

16

88

5.1

13

169

66.3

30.6

36

78

5.2

20

400

104.0

36.4

49

140

5.7

22

484

125.4

22.8

16

88

5.3

17

289

90.1

26.5

25

85

6.6

23

529

151.8

46.2

49

161

5.6

25

625

140.0

16.8

75

8.4

27

729

226.8

67.2

64

216

5.5

26

676

143.0

11.0

52

10

6.5

20

400

130.0

32.5

25

100

Total

59.1

215

51

4785

1292

310.8

293

1083

Mean

5.91

21.5

5.1
.

Table 6 . l O b :

bo,

b . , and

Current Year's Sales and the

b, can be determined
-

Net Profit of All Ten Stores

by using the formulae:

-2

( I; Y X , - n Y X 1 ) ( I: X 2 - n X 2 ) - ( I; Y X 2 - n Y X 2 ) ( I; X 1 X 2 - n X 1 X 2 )

b, =-------------
-2

( I; X ,

-2

_ (1292 - 1 0 x 5 . 9 1 x 2 1 . 5 ) ( 2 9 3 - 1 0 x 5 . 1
1
-

- n X 1 ) ( I: X 2 - n X 2 ) - ( I; X 1 X 2 - n X 1 X 2 )

(4785 - 1 0 x 2 1 . 5

2
)-

( 3 1 0 . s - 1 0 x 5 . 9 1 x 5 . 1 ) ( 1 0 8 3 - 1 0 x 2 1 . 5 x 5.1)

)(293-10 x 5.1

) - ( 1 0 8 3 - 1 0 x 2 1 . 5 x 5.1)

b, = 0 . 1 6

-2

- -

_ (I; YX2 - nYX2HI: x, - nx, ) - (I; YX, - n Y X i l ( I: x,X2 - n X , X 2 l


2

2
-

( I; X f - n X t ) ( I; X - n X l - ( I X , X 2 - n X , X 2 )

_ ( 3 1 0 . 8 - 1 0 x 5. 91 x 5.1)(4785 - 1 0 x 21.5
2
-

(4785 - l O x 21.5

= 0.35

bo

=Y-b1X1-b2X2

2
)-

(1292 - 1 0 x 5. 91 x 21.5)(1083 - 1 0 x 21.5 x 5 . 1 ) )

)(293 - 1 0 x 5.1

)-

(1083 - 1 0 x 2 1 . 5 x 5.1)

bo = 5 . 9 1 - 0 . 1 6 x 2 1 . 5 - 0 . 3 5 x 5 . 1

b0 = 0 . 6 8 5

www.itmuniversityonline.org

Page

160

Introduction to Q u a n t i t a t i v e T e c h n i q u e s

06.

Rank C o r r e l a t i o n a n d

eBook

R e g r e s s i o n Analysis

Therefore,

by

substituting

the

values

of

written as:

Y = 0.685+0.16X1 +0.35X2

b0,

b 1,

and

b2;

regression

equation

This equation is known as the multiple regression equation of Y o n X, and


how

change

coefficient
constant

in

X,,

for

X1

and

that

is,

every

changes

X2

b,

0.16

additional

is

unit

increases

by t 0 . 1 6 crore

or t16

constant,

then

additional

for

every

the

value

that,
of

lakh.

if

sale

of Y.

of

interpretation

X,

(sales

of

vegetarian

Similarly,

unit

The

sale

if X1
of

of

of

(sales

items,

food

the

value

food)
the

of vegetarian

non-vegetarian

be

X,. It indicates

non-vegetarian
food

can

is

net

food)

items,

of

held
profit

is

held

the

net

of

this

profit increases to t 0 . 3 5 crore or t35 l a k h .

The

multiple

coefficient

relationship and

of

determination

helps

us

to

determine

the

reliability

is given b y :
2

R2

_ Unexplained

variation

= 1 - I(Y; - Y ; )

1
2

Total variation

ICY; - Y )

To obtain the coefficient of multiple correlation coefficient following table is computed.

X1

y, =

X2

0.685 + 0 . 1 6 X 1 + 0 . 3 5 X 2

(y;-y,)'

<v; -

v>'

5.2

22

5.605

0.164

0.5041

5.1

13

4.865

0.055

0.6561

5.2

20

6.335

1.288

0.5041

5.7

22

5.605

0.009

0.0441

5.3

17

5.155

0.021

0.3721

6.6

23

6.815

0.046

0.4761

5.6

25

5.735

0.018

0.0961

8.4

27

7.805

0.354

6.2001

5.5

26

5.545

0.002

0.1681

6.5

20

5.635

0.748

0.3481

2.706

9.369

Total

Table

6.10c:

Current Year's Sales and the

www.itmuniversityonline.org

Net

Profit of All Ten Stores

Page 1 6 1

I n t r o d u c t i o n to Q u a n t i t a t i v e T e c h n i q u e s

06.

R a n k Correlation a n d

Here,

eBook

Regression A n a l y s i s

Y;S.91

After substituting values from the above table, we can write:


2

R2

i _

L (Y;

- Y;)
2

L ( Y ; - Yl

; 1 - 2.706

9369

; 1 - 0.2888
; 0.7112

www.itmuniversityonline.org

Page

162

Introduction to Q u a n t i t a t i v e T e c h n i q u e s

06.

Rank

Correlation

and

eBook

R e g r e s s i o n Analysis

6 . 1 1 Chapter Summary

Spearman's
relation

rank

Regression

In

is

the

most

analysis studies the cause and

simple

establishes a functional

regression,

dependent

and

only

other

is

an

two

When

one

called

variable

variables

independent

linear

regression.

with

In

are

taken

variable.

it

variables

linear

is

and

simultaneously,

neither

identify

the

called

another

non-linear

the

partial,

relation

constant.

variable

In

regression

total

coefficient

can

only

two

between

as

one

two or more

which

hand,

one

when

is

more

all

the

ratio,

variable

varies,

is

are

studied,

variables

taken

it

the

into

keeping
are

is

the

studied

them.

two variables

multiplied

constant

variables

variables

regression,

be

in

other

some

more

the relationship among

relationship

study,

the

in

regression,

two

between

in order to find

regression the

if

between

is a multiple regression.

or

regression

for

On

The

In

to

between them.

ratio.

other

measure

relationship

other variable does not change in a constant

consideration

changes

effect

relationship

t h a n two variables are taken for study,

appropriate

between two qualitative variables.

variables and

correlation

nor divided

must
by

be

linear,

another

that

is,

regression

coefficient.

The

coefficient

of

determination

between X and Y and

In

multiple

independent
measured

is

it ranges from

regression
variable,

the
say

the
O to

dependent

X,,

X,,

square

the

correlation

coefficient

and

Y
the

is

function

relationship

of

more

between

than

Page

one

them

by the m u l t i p l e correlation coefficient.

www.itmuniversityonline.org

(r)

1.

variable

X, . . .

of

163

is

Business Forecasting
a n d T i m e Series Analysis

I T M

Introduction to Q u a n t i t a t i v e T e c h n i q u e s

07.

Business

Forecasting

and Time

eBook

Series Analysis

7 . 1 Introduction
This

chapter

business,
the

past,

competition
the

and figures,

Prediction
a

introduces

process

plays

present,

and

attempting

and

the

concept
an

business

important

what

the

future

role
will

forecasting

and

this

and

time

generates

possibly

be.

the

Without

series.

need

to

In

any

analyze

proper data,

facts,

to answer questions of the future is difficult.

estimation of such

that

of

helps

the

future conditions

management

to

is

known as 'forecast'.

make

decisions

and

Forecasting

understand

is

the

implications of these decisions on the future of the organization.

Time
of

series

data

collection,

movement
making
is why
and

is one of the most


with

of economic

process of an
time

series

business

known

respect
factors

to

that

i n d i v i d u a l firm

has

occupied

statistics.

This

methods of forecasting.
time

are

variable.

continuously

within

changing

important

introduces

the

of

helps

and

to

affect

analyze
the

in

the

time

field

series

you

will

in

both

be able to:

Define time series

Explain the different components of time series

Discuss methods of measuring

State mathematical model of time series

Describe the decomposition of time series

Discuss analysis of time series

www.itmuniversityonline.org

This

of economics

fields.

After reading t h i s chapter,

the

decision

economy as a whole.

place

role

is a systematic method

series

industries or in an

a u n i q u e and

chapter

Time

It

trend

Page

165

these

I n t r o d u c t i o n to Q u a n t i t a t i v e T e c h n i q u e s

07.

B u s i n e s s Forecasting

and

Time

eBook

Series Analysis

7 . 2 Time Series
The

arrangement

the

incidence

of

of

statistical

time,

is

known

quantitative methods that can

According

to

data

Patterson,

in

as

time

be used

time

chronological
series.

order,
Time

to

Spiegel,

time

series

series

is,

series

in

concurrence

analysis

is

one

with

of

the

to predict the future.

consists

collected, recorded or observed over successive

According

that

is

set

of

of

statistical

data

which

are

increments.

observations

taken

at

specified

times, usually at equal intervals.

Mathematically,
(temperature,

time

closing

series

is

defined

price of a share,

etc.)

by

the

values

at times ti,

Y1,

t, . . .

Y,

Thus,

...

or

variable

Y is a function of

Y
't,'

given asY = f(t).

Table

7.2a

is

an

example of time

of students registered for an

series data.

MBA program

Year

It

shows

the

time

series

for the

number

in an Indian college.

2000

2001

2002

2003

2004

2005

2006

2007

3000

3050

4000

4500

5000

5300

5500

6000

Number of Students
Registered
Table 7 . 2 a :

N u m b e r of Students

Registered for an

MBA Program

Components of Time Series


Time

series

T h i s could
or

is

be daily,

temperature,

national

set

of observations

weekly,

weekly

at

equal

monthly, quarterly or yearly.

crimes

in

city,

monthly

intervals

over

period

of time.

For example, d a i l y share prices

sales

or

annual

profit

of

firm,

income, etc.

The analysis of time series


the

recorded

variable

considering

is

the

used
types

to forecast the
of variations

monthly, weekly or quarterly values of

present

in

it.

There

responsible for changes in the values of the variable at different

www.itmuniversityonline.org

are

various

factors

periods of time.

Page 166

Introduction to Q u a n t i t a t i v e T e c h n i q u e s

07.

Business

These

Forecasting

influencing

and Time

factors are

eBook

Series Analysis

broadly classified

into the following

four categories,

known

as components of time series:

Secular trend

(T)

Seasonal variation

Cyclical variation

Random or Irregular variation

(S)

(C)
(!)

Some or a l l of these may be present in a time series in varying

degrees.

7.3 S e c u l a r Trend
According

to

and

Simpson

basic tendency of production,


over a period of time.

"Trend also caf/ed secular or long-term

Kafka,

safes,

income,

employment,

or the like

trend,

is

the

to grow or decline

The concept of trend does not include short-range oscillations but,

rather, steady movements over a long time".

Secular

trend

cannot move

can
in

upward

either

tendency

is

sales and

profits recorded.

not

circulation,

seen

in

is

necessary

throughout

the

the

movement and

currency

It

in

both the directions.

not exhibit erratic


an

go

that

given

values of certain

etc.

in

The

direction

upward

and

or

data

related

or

Occasionally,

one

may

trends
may

a constant

the

downward
direction

be

come

be

across

reading,

in
a

in

production,

data

the

related

same

situation

which does

where

Increasing/::Jecreasing trends of a

vanable

Line

Time in Years

www.itmuniversityonline.org

the

not change with

Fig. 7 . 3 a :

to

direction

time.

Secular Trend

It

For example,

agricultural

noticed

should

direction.

movements do

duration of time.

population,

tendency

decreased

series fluctuate around

to

in

downward

are constant for a long

downward

increased

period.

upward

Secular Trend

Page

167

I n t r o d u c t i o n to Q u a n t i t a t i v e T e c h n i q u e s

07.

eBook

B u s i n e s s Forecasting a n d Time S e r i e s A n a l y s i s

If the consumer price index is considered over a period of time, the secular trend can
represented
trend

with

secular

graphically,

an

upward

trend

line.

It

can

the straight

line.

linear,

otherwise

is

In

linear

trend,

a m o u n t . This

the

The

be

in

actual

seen

Here,

Fig.

that

the trend

7.3a.

data
the

is

The

straight

shown

values

of

is executed

by
the

line

a curve
series

by a straight

indicates
moving

gather
line and

secular

around

more

or

the
less

is termed

as

non-linear.

time

linear trend

linear secular trend

shown

slope.

around

it

as

be

series'

is also

values

increase

known as straight

is given by the equation:

or decrease
line trend

by

a constant

absolute

or first degree curve.

The

Yt = a + bX

Where,

Y, = Trend

value of the variable

X = Time
a and

b = Constants

The trend
shown

in

for production of a particular factory can be either increasing or decreasing, as


Fig. 7 . 3 b .

Decreasing Trend

Increasing Trend

cc

cc

.,

Y = a + bX

u
:,

s
:,

Q.

Q.

Fig. 7 . 3 b :

N o n - l i n e a r trend

Y = a + bX

is a type of trend

Linear Trend

where the rate of c h a n g e in growth and

decline is not

constant.

www.itmuniversityonline.org

Page

168

Introduction to Q u a n t i t a t i v e T e c h n i q u e s

07.

Business

Forecasting

and Time

eBook

Series Analysis

Uses of Secular Trend:

To determine basis of fluctuations

To study forecasting

To study the effect of short term

To compare data that

To analyze the

variations

has been collected

long-term

impact of such a policy

for a long

results of a policy

based

duration of time

implemented

and

determine the future

on the analysis

7.4 S e a s o n a l Variation
Seasonal
pattern
such

variations

during

as

the

certain

seasons,

seasonal

are

for

that

months

weather

movements.

demand

those

In

of

within

the

to

business,

products,

Prof.

period

or

can

fluctuation

variation

changes

etc.

due

plays

refers

to

within

of data
to

an

natural

important

systematic

in

recurring

similar
forces,
role

variations

time

in
in

period.

be foreseen.

variations

the

variation

precise, and

from

the

years.

consumption,

"Seasonal

results

show

climate

seasonal

services,

Hirsch,
that

successive

conditions

Seasonal variations are definite,

According

periodically

are

operation

the

recurrent

of forces

pattern

connected

of

with

change

climate

or

custom at different times of the period."


Some examples of seasonal variation are:

The

demand

for sunscreen

be h i g h d u r i n g

summer and

is

likely

Demand

for cold drinks and

Demand

for woolen wear tends to

variations

fashions, and

can

also

customs of local

have an

annual

cycle,

in which

demand

will

low in other seasons.

Seasonal

to

ice-cream go up

occur

increase

due

to

in summer.

in winter.

manmade

conventions,

such

as

habits,

like

Diwali,

populations.

Some examples of seasonal variations due to manmade conventions are:

Sale of jewelry

Sales and
Eid,

is higher d u r i n g

marriage seasons.

profits of departmental

Dusshera,

stores go

up

during

festive

seasons,

Christmas, etc.

Such variations occur in a regular manner and

www.itmuniversityonline.org

repeat year after year.

Page

169

I n t r o d u c t i o n to Q u a n t i t a t i v e T e c h n i q u e s

07.

eBook

B u s i n e s s Forecasting a n d Time S e r i e s A n a l y s i s

Therefore,

seasonal

variations

are

due

to

change

in

the

environment

or

other

cultural

factors, which causes people to have different types of requirements in different seasons
but within a specific time-period of the year.

Seasonal behavior within one year

Seasonal Variation

x
Time in Years

Fig. 7 .4a:

Seasonal Variation

Uses of Seasonal Trend:

It ai d s in the study of irregular and

cyclic variations.

It helps in understanding the pattern of change in a particular commodity.

7 . S Cyclical Variation
The oscillatory movements of variables in a period of more t han one year are termed as
cyclical variation.
occur d u r i n g

According

to

One complete oscillation

longer periods of time, say,

Spiegel,

"Cyclical

may

or

may

not

be

periodic,

patterns after equal interval of time.

a cycle.

Cyclical variations generally

7 or 10 years.

movements

oscillations or swings about a trend fine


caf/ed,

is called

or cyclical

or curve.

i.e.

they

variations

These

may

or

cycles,
may

refer
as

not

to

the

long-term

they are sometimes

fof/ow

In business and economic activity,

exactly

similar

movements are

considered cyclical only when they occur after time intervals of more than a year."

www.itmuniversityonline.org

Page

170

Introduction to Q u a n t i t a t i v e T e c h n i q u e s

07.

Business

Forecasting

and Time

eBook

Series Analysis

Long-term cyclical

behavior of a variable

Cyclical Variation

Trend Line

Time in Years

Fig. 7 . S a :

A cyclical variation
1.

Prosperity

2.

Recession

3.

Depression

4.

Recovery

Cyclical Variation

has the following four phases:

Use of Cyclical Variation

A business cycle
with

long-term

is one of the most appropriate examples of a cyclical variation.


economic

aspects,

like

prosperity,

recession/decline,

It deals

depression,

and

recovery.

7.6 Random or I r r e g u l a r Variation


Random

or

unpredictable.
beyond

irregular
They

are

human control

episodic

and

identified.
removing

variations/fluctuations

residual

but are

fluctuation.

Such

to

strikes,

fluctuations,

fluctuation

is

non-recurring

and

part of the system.

variations.

For example,
episodic

due

are

Episodic

wars,
the

The

earthquakes,
is

nor

erratic,

circumstances

irregular variation
are

famines

residual

identified.

are

is d i v i d e d

into

or

variation
It affects

but

can

floods.
or

be

After

chance

business and

economic activities unexpectedly but the effect exists for a very short period.

www.itmuniversityonline.org

and

that

unpredictable

epidemics,

called

predictable

random,

irregular

fluctuations

variation

neither

purely

Page

171

I n t r o d u c t i o n to Q u a n t i t a t i v e T e c h n i q u e s

07.

B u s i n e s s Forecasting

and

Time

eBook

Series Analysis

Random movement of variable

'--x
Time in Years

Fig. 7 . 6 a :

Random or Irregular Variation

7 . 7 Trend Measurement Methods


The four methods used

to measure trend

Free Hand

Semi Average Method

Moving Average

Least Squares Method

Free

As

trend.

Curve Method

Hand Curve

the

name

are:

Method

Method

suggests,

this

is a completely graphical and

The values of the variables are plotted

the trend

can

be determined

Semi Average

According

to

by

subjective

with time along

method

the X-axis.

and

Kafka,

"Semi-average

means average

of semi

that is the average of each half of the series." In t h i s method,

divided

into

parts and
is skipped

Moving Average

In this method,
will

have

this method,

inspection only.

halves,

the middle data

locate a

Method

Simpson

two

In

to

the

middle

year of the

and the remaining

data

data

is

marked.

is divided

the o r i g i n a l data

In

into two

or average

case of odd

of

is

data,

parts.

Method

if the origina l

similar

pattern.

www.itmuniversityonline.org

data
If

the

gives a straight
original

series

l i n e on a g r a p h ,
gives

concave

the moving
curve,

the

Page

average
moving

172

Introduction to Q u a n t i t a t i v e T e c h n i q u e s

07.

Business

average

Forecasting

will

be

average w i l l

below

it

and Time

and

if

eBook

Series Analysis

the

original

series

gives

convex

curve,

the

moving

be above it.

Least Squares Method

This method

minimizes the sum of the squared errors of the data series. This method

a widely used

method to obtain the trend.

7.8 M a t h e m a t i c a l
The

is

following

models

Model of Time Series

are

commonly

used

to

decompose

time

series

into

its

components:

Additive

Model

According

to the additive model, a time series can

be expressed

as:

Ut = Tt + S t + C t + Rt

Where,
U1

= Time series value at time t

Tt = Trend
s.,

c,,

values

R, = Seasonal, cyclic, and

This model

is based

random components of time series,

on the assumption

that all

the components are

respectively.

independent of each

other.

Multiplicative

Model

Decomposition

by

multiplicative

multiplicative relation.

Ut

The

Tt

St

Ct

According

method

is

one

in

which

all

the

four

components

have

to this m o d e l :

Rt

multiplicative

model

of

time

series

is

equal

to

the

additive

decomposition

logarithmic values of the o r i g i n a l time series.


log U, = log Te + log St + log Ct + log Rt

www.itmuniversityonline.org

Page

173

of

I n t r o d u c t i o n to Q u a n t i t a t i v e T e c h n i q u e s

07.

eBook

B u s i n e s s Forecasting a n d Time S e r i e s A n a l y s i s

7 . 9 Decomposition of a Time Series


The analysis or decomposition
various components.
series are affected

Not all

of a t i m e

series

is used

to

segregate and

types of variations affect the whole time

by a few of these variations.

determine the

series.

Most time

The objective of the decomposition of a

time series is to obtain data free from the effect of one component on the other,

so that

it can be used for more accurate forecasting.

The moving

average method

is another method

that

is used

especially when a seasonality factor is present in the data.


by moving average method
Step

1:

Step 2:
Step

3:

Decomposition of time series

Moving Average (CMA) for each month or quarter

obtain seasonal ratios by dividing each observed


raw

values,

involves the following steps:

calculate a Centered

obtain

to determine trend

seasonal

indices

by

averaging

value "yt"

the

quarterly

or

monthly

seasonal

ratios

Step 4:

obtain the adjusted seasonal indices such that the sum of the indices is e q u a l to

12 (monthly) or 4 (quarterly)
5:

deseasonalize

adjusted

seasonal index

Step

Step

6:

determine

the

Obtain the forecasted


Step

7:

the

trend

in the

of

series

by

dividing

deseasonalized

values by trend

determine adjusted

component

time

trend

observed

time

series

y,

by

value

curve

by

respective

fitting

method.

line equation

values

projected or forecasted

of forecasted
trend

values

quarter.

Adjust

by multiplying

the

seasonality

the corresponding

quarter seasonal index with the forecasted trend value

The

aforementioned

steps

are

illustrated

through

the

following

example.

Table

shows the quarterly sales of a particular medicine in lakh.

At the End of Each Quarter


Year

March

lune

September

December

(Q1)

(Q,)

(Q,)

(Q4)

2006

351

469

472

492

2007

549

532

522

610

2008

650

640

712

770

2009

718

853

780

840

Table 7.9a:

www.itmuniversityonline.org

Quarterly Sales of a Pharmacy

Page

174

7.9a

Introduction to Q u a n t i t a t i v e T e c h n i q u e s

07.

Business

Step

1:

Forecasting

calculate Centered

The CMA is calculated


and

i r r eg u l a r

considered
values,

and Time

for calculation.

consider

.
.
First moving

of

average method

the

In this

values

average is obtained

Moving Average (CMA } for each quarter

by moving

components

eBook

Series Analysis

for

time

series.

problem,

obtaining

by averaging

so as to isolate the combined


In

this

method,

the time series

moving

one

is given

average.

The

in

first

year

seasonal

of

data

is

terms of quarterly
4-quarter

moving

the first four quarters of year 2006 as follows:

351+469+472+492
average=

= 446
4

Since

first

placed

moving

between

quarter. The

average

and

second

3'

represents
quarter.

moving average=

Second

moving

51

quarter

of

average

2007,

series,

moving

hence

446

place

first
the

year

so

average

value
value

by averaging

has

to

be

against

3'

the 2,

3'd,

4th

place

549
= 495.5

mid-value

average

of 2,

value

as shown

is

calculated.

The

first

3',

against

4th
4th

quarters of year
quarter

of

2006.

2006

and

Similarly,

in Table 7.9b.

average does not correspond

CMA

of

is obtained

represents

average as shown

First CMA=

average

4 7 2 ; 492

remaining average values are obtained

time

simplicity

thus,

The 4-quarter moving

mid-value

1" quarter of 2007 as follows;


469

Second

For

4-quarter moving

quarters of year 2 0 0 6 and

the

directly to the original quarter of the

CMA

is

an

average

of

first

and

second

below:

4955
+

= 470.75

2
Similarly,

remaining

plotted as shown

CMAs are calculated

as shown

in Table 7.9b. These

in Fig 7.9a to indicate that the trend

www.itmuniversityonline.org

CMA values are

is l i n e a r .

Page 1 7 5

I n t r o d u c t i o n to Q u a n t i t a t i v e T e c h n i q u e s

07.

B u s i n e s s Forecasting

Step 2:

and

Time

calculation of Seasonal

In the multiplicative model,

eBook

Series Analysis

Ratios

the original data y can

be expressed

as:

y = Ut = Tt x St x Rt

If

is

divided

components
the moving

will

the

trend

remain.

averages,

component will

.J....

by

component

However,

even the

be left.

if y

random

Mathematically,

is

T,,

only

divided

the
by

component
it can

seasonal

the

trend

(S,)

and

values

is separated

random

obtained

(R,)

through

and o n l y the seasonality

be written a s :

st

Tt

On

the

basis

obtained
centered

by

of

the

above

dividing

moving

the

formula,
original

averages.

Thus,

seasonal

values

of

respective

ratio
y

of

is

calculated.

the

quarter

time

The

series

seasonal

seasonal

by

ratios

the

are

ratio

4-quarters

calculated

Table 7.9b.

Seasonal

ratio=

Original values of sales

(y)

Corresponding centered moving average

values

Centred
4-quarter
Sales
Year

Quarter

Seasonal
Moving

Moving
(y,)

Ratio
Averages

Average

(yt/CMA)
(CMA)

Q,

351

Q,

469

Q,

472

446.00

470.75

1.00

Q.

492

495.50

503.38

0.98

Q,

549

511.25

517.

so

1.06

Q,

532

523. 75

538.50

0.99

Q,

522

553.25

565.88

0.92

Q4

610

578.50

592.00

1.03

Q,

650

605.50

629.25

1.03

Q,

640

653.00

673.00

0.95

Q,

712

693.00

701.

so

1.01

Q4

770

710.00

736.63

1.05

2006

2007

2008

www.itmuniversityonline.org

Page

is

176

in

Introduction to Q u a n t i t a t i v e T e c h n i q u e s

07.

Business Forecasting

eBook

and Time Series Analysis

Q,

718

763.25

7 7 1 . 75

0.93

Q,

853

780.25

789.00

1.08

Q,

780

797.75

2009

840

Q.
.

Table 7 . 9 b :

Calculation of CMA and Seasonal

Ratios of Quarterly Sales of a

Pharmacy

1000-

BOO

2
"'
c :l;i

600-

3
0

iii
ro

...

400-

"'

200
0

10

12

14

16

Quarters

Fig. 7.9a:

Step 3 :

Centered

Moving Averages

calculation of Raw Seasonal Indices

Summarize the seasonal ratios as per quarters as shown in Table 7.9c.

Raw seasonal

index is obtained

Raw seasonal

by averaging the seasonal ratios for the same quarter.

index for the first q u a rt e r of complete time series is calculated as follows:

Raw seasonal index for first q u a r t e r = 1 . 0 6 + 1.i3 +

Seasonal

Year

Qi

o.

93

= 1.007

Ratios per Quarters

Q,

Q2

Q.

2006

".

" .

1.00

0.98

2007

1.06

0.99

0.92

1.03

2008

1.03

0.95

1.01

1.05

2009

0.93

1.08

".

"

Average (Raw

1.007

1.007

0.980

1.018

Seasonal Index)
Table 7.9c:

Raw Seasonal

www.itmuniversityonline.org

Indices for Quarterly Sales of a Pharmacy

Page 1 7 7

I n t r o d u c t i o n to Q u a n t i t a t i v e T e c h n i q u e s

07.

eBook

B u s i n e s s Forecasting a n d Time S e r i e s A n a l y s i s

These seasonal

indices for various quarters

indicate only the seasonal component

original time series. They are shown in the following g r a p h :

1.2

IG
1.0

.
!..!

.s
"'

.,.

0.8

ls

0.6

.,:
w

0.4

e
"'

;;

0.2

Quarters

Fig.

7.9b:

Average Seasonal Indices for Quarterly Sales of a


Pharmacy

www.itmuniversityonline.org

Page

178

in the

Introduction to Q u a n t i t a t i v e T e c h n i q u e s

07.

Business

Step 4:

The

and Time

eBook

Series Analysis

obtain adjusted seasonal indices

table

model

Forecasting

summarizes

of seasonality

be e q u a l

to

1.00,

is

raw

requires

so the

then adjustment
the following

the

sum

required

seasonal

that

the

of the

in

the

indices

average

four

of

various

seasonal

indices

raw seasonal

should
indices.

quarters.

The

multiplicative

indices of four quarters


be 4 . 0 0 .

If the total

The adjusting

is

should

not 4 . 0 0

factor is given

by

formula:

4
Adjusting

factor=

----------
Sum

As the

sum

of the

factor is obtained

Adjusting

of

raw

seasonal

indices

indices

is greater

four seasonal

than

4.00

in

this

case,

the adjusting

as follows:

factor=

0. 997

4.;12

The adjusting

factor is m u l t i p l i ed

indices as shown

to the

raw

seasonal

indices to obtain adjusted

seasonal

in Table 7.9d

Adjusting

Raw Seasonal

Factor

Adjusted

Quarter
Seasonal

Indices

Q,

1.007

0.997

1.004

Q2

1.007

0.997

1.004

Q,

0.980

0.997

0.977

Q,

1.018

0.997

1.015

Total

4.012
.

Table 7 . 9 d :

Step 5 :

The

Deseasonalize the Time series

process

expressed

If

4.00
.

AdJusted Seasonal Indices for Quarterly Sales of a Pharmacy

of

removing

deseasonalization

Indices

of

the

data.

In

seasonal
the

effect

from

multiplicative

time

model,

series

the

data

original

is

data

known

as

be

can

as:

Ut = Tt x St x Rt

is

divided

components

will

by
be

the

seasonal

left.

season from time series is

If

is

component
divided

removed.

www.itmuniversityonline.org

by

St,
the

only

the

seasonal

Mathematically,

it can

trend
index

(Tt)

and

then

the

random
effect

of

be written as:

Page 1 7 9

(Rt)
the

I n t r o d u c t i o n to Q u a n t i t a t i v e T e c h n i q u e s

07.

B u s i n e s s Forecasting

and

Time

eBook

Series Analysis

y
- = T,

s,

Thus,

the

new

deseasonalized
seasonal

time

index,

for the sale at

trend

values

series.

value

obtained

Thus,

obtained

is

if the
called

pharmacy is calculated

will

be

quarter

free

of

sale

value

deseasonalized

seasonal
is

sale.

effect

divided
The

by

and

will

give

corresponding

deseasonalized

value

in Table 7.9e.

Adjusted
Deseasonalized
Year

Sales

Seasonal

(y,)

Index

Quarter

Sales
(yt!S.)
(S.)

Q,

351

1.004

349.6

Q,

469

1.004

467.13

Q,

472

0.977

483.11

Q,

492

1.015

484. 73

Q,

549

1.004

546.81

Q,

532

1.004

529.88

Q,

522

0.977

534.29

Q,

610

1.015

600.99

Q,

650

1.004

647.41

Q,

640

1.004

637.45

Q,

712

0.977

7 2 8 . 76

Q,

770

1.015

758.62

Q,

718

1.004

715.14

Q,

853

1.004

849.6

Q,

780

0.977

798.36

Q,

840

1.015

8 2 7 . 59

2006

2007

2008

2009

Table 7 . 9 e :

Step 6 :

Deseasonalized Sales of a Pharmacy

determine trend of deseasonalized time series

The trend

is determined

by fitting

a curve. The equation of a straight line can

be written

as:

y,

a + bt

www.itmuniversityonline.org

.............

(1)

Page

180

Introduction to Q u a n t i t a t i v e T e c h n i q u e s

07.

Business Forecasting

eBook

and Time Series Analysis

Where,

Y = Deseasonalized medicine sales


t = Quarter of the year
a and b = Constants which can

be determined

P r e l i m i n a r y Adjustment before Analyzing

Since

one

of the

variables

is

time,

given

by the principle of least squares

the Time Series

here

in

terms

of quarters,

you

can

represent

time variable by sequential number as follows:


1 for the quarter e n d i n g

Q,, 2 0 0 6

2 for the quarter e n d i n g

Q 2, 2 0 0 6

3 for the quarter e n d i n g

Q3, 2 0 0 6

4 for the quarter e n d i n g

Q., 2006

16 for the quarter e n d i n g

Sales at the

Q., 2009

End of Each Quarter

The values of a and

b can be obtained

using

the following formula:

L Y d t ; - nydt
b

= -- and
2
I tt - n(tJ

a=

y - bf

Where,
n = Highest code assigned

The values o f t , Yd,

Yd!;,

L IF

www.itmuniversityonline.org

can be calculated as shown in Table 7.9f.

Page 1 8 1

I n t r o d u c t i o n to Q u a n t i t a t i v e T e c h n i q u e s

07.

eBook

B u s i n e s s Forecasting a n d Time S e r i e s A n a l y s i s

Deseasonalized
Year

Quarter

t;

Sales

yot;

t?

Q,

349.6

349.6

Q,

467.13

934.26

Q,

483.11

1449.33

Q,

484. 73

1938.92

16

Q,

546.81

2734.05

25

Q,

529. 88

3179.28

36

Q,

534.29

3740.03

49

Q,

600.99

4807.92

64

Q,

647.41

5826.69

81

Q,

10

637.45

6374.5

100

Q,

11

728. 76

8016.36

121

Q,

12

758.62

9103.44

144

Q,

13

715.14

9296.82

169

Q,

14

849.6

11894.4

196

Q,

15

798.36

11975.44

225

Q,

16

827.59

13241.44

256

Total

136

9960

94862.44

1496

Average

8.5

622.50

2006

2007

2008

2009

Table

By substituting

7 .9f:

the values o f t , Yd,

Fitting trend

Z:

Ydt,,

Z: t t ;

b = 9 4 8 6 2 . 4 4 - 1 6 x 6 2 2 . 5 x 8 . 5 = 10202.44 =

Line

Method

we get:

.0l
30

1496-16(8.5)

340

a = 622.5 - 3 0 . 0 1 x 8.5 = 367.41

www.itmuniversityonline.org

Page

182

Introduction to Q u a n t i t a t i v e T e c h n i q u e s

07.

Business

Forecasting

Therefore, the equation

and Time

eBook

Series Analysis

representing

the trend component

is:

y = 367.41 + 30.0lt

Here,

the

increased
obtained

interpretation
by ? 3 0 . 0 1

by

of

lakh

substituting

the

value

of

every quarter.

30.01

Quarterly trend

the values of the

respective

is

that

the

medicine

sales

values for the year 2 0 1 0 can

number of the quarter

in

the

has
be

trend

equation y d = 3 6 6 . 9 5 + 3 0 . 0 2 t .

For example,
is:

for the first quarter of 2 0 1 0 ,

the code of the quarter

is

17,

so the forecast

Y=367.41+30.01x17=877.58

Similarly,

you can determine the forecast of trend

values for the other quarters of 2 0 1 0 .

They are tabulated as follows:

Forecast of Trend Values


Code of
Year

Quarter

for the Quarters


the Quarter

(f in lakhs)
March

17

877.58

June

18

907.59

September

19

937.60

December

20

967.61

2010

Table 7 . 9 g :

Deseasonahzed

Forecasted Quarterly Sales for year 2 0 1 0 of a


Pharmacy

Step 7 :

The

derivation of Adjusted

forecast

for

with the seasonal

Year

quarters

of

the

Quarterly Forecasts of 2 0 1 0

year

2010

will

be

obtained

by

multiplying

the

values

indices.

Quarter

Deseasonalized

Seasonal

Adjusted

Quarterly Sales

Index

Quarterly

(in Lakhs)

March

(Q,)

(f)

sales

877.58

1.004

881.09

June (Q2)

907.59

1.004

911.22

September (Q,)

937.60

0.977

916.04

December (Q 4 )

967.61

1.015

982.12

2010

Table 7 . 9 h :

www.itmuniversityonline.org

Quarterly Sales of a Pharmacy

Page

183

I n t r o d u c t i o n to Q u a n t i t a t i v e T e c h n i q u e s

07.

B u s i n e s s Forecasting

and

Time

eBook

Series Analysis

7 . 1 0 Analysis of Time Series


Analysis

of

time

series

can

be

explained

appropriately

with

the

help

of

the

following

examples.

Example 0 1 :

The following

table shows the seasonal indices for the sale of refrigerators by a dealer:

Seasonal
Quarter
Indices

Table 7 . 1 0 a :

If

the

should

sales
be

in

the

stocked

first
by

0.95

II

0.87

III

0.85

IV

1.33

Seasonal Indices for Sales of Refrigerator

quarter

of

the dealer to

year

are

180,

determine

meet the demand

of the

how

many

refrigerators

other three quarters of the

year.

Solution O 1 :

According
seasonal

to

the

multiplicative

model,

the

total

index in the first quarter is 0 . 9 5 , which

Thus, for a seasonal

index of 0 . 9 5 ,

Therefore,

second

for

the

of

four

is equal to

the amount to s t o c k =

quarter

with

seasonal

seasonal

180

indices

is

4.00.

The

in terms of sales.

180

index

of

0.87,

the

number

of

0
refrigerators to stock=

8 7 x 180 = 165
0.95

For the third

quarter with a seasonal

0 85

161

180

index of 85,

the

number of refrigerators to stock

0.95

For

the

fourth

quarter

33
stock= 1.

x 180

with

seasonal

index

of

1.33,

the

number

of

refrigerators

252

0.95

www.itmuniversityonline.org

Page

184

to

Introduction to Q u a n t i t a t i v e T e c h n i q u e s

07.

Business

Forecasting

and Time

eBook

Series Analysis

Example 0 2 :

Assuming

there

is

no

trend

in

the

series;

calculate

the

seasonable

indices

for

the

following data.

Quarter

Year

II

III

IV

2005

74

68

80

80

2006

82

72

86

74

2007

74

64

83

72

2008

72

74

81

76

2009

76

74

86

80

Table 7 . 1 0 b :

Solution

In

the

Quarterly Observations over Five Years

02:

given

data

components are

the

trend

present.

is

not

Seasonal

present,

that

factor method

is,

only

is applied,

the

seasonal

and

random

the steps of seasonal

factor

is as follows:
Step

1:

calculate average demand

Step

2:

calculate seasonal

per year or period

ratio for each

corresponding average demand

period

by d i v i d i n g

actual demand

calculate seasonal index by taking average of seasonal

Step

calculating average demand

ratios for each quarter

per year

Average

Quarter

Total

Demand

Year

II

III

IV

2005

74

68

80

80

302

75.5

2006

82

72

86

74

314

78.5

2007

74

64

83

72

293

73.25

2008

72

74

81

76

303

75.75

2009

76

74

86

80

316

79

Table 7 . 1 0 c :

www.itmuniversityonline.org

by the

per year

Step 3 :

1:

value

A v e r a g e Demand

per Year

over Five Years

Page

185

I n t r o d u c t i o n to Q u a n t i t a t i v e T e c h n i q u e s

07.

B u s i n e s s Forecasting

S t ep 2:

and

Time

calculating Seasonal

Seasonal

ratio for 74

eBook

Series Analysis

Ratio

= 0.9801
75.5

Seasonal

ratio for 82

= 1.0446
78.5

Similarly seasonal

ratios are given

below:

Quarter

Year

II

III

2005

0.9801

0.9007

1.0596

1.0596

2006

1.0446

0.9172

1.0955

0.9427

2007

1.0102

0.8737

1.1331

0. 9829

2008

0.9505

0.9769

1.0693

1.0033

2009

0.9620

0.9367

1.0886

1.0127

Total

4.9475

4.6052

5.4462

5.0012

Table 7 . l O d :

Step 3:

IV

Calculation of Seasonal

Ratios over Five Years

Calculating Seasonal Indices

Quarter
Year
II

III

IV

Five-year
Seasonal

4.9475

4.6052

5.4462

5.0012

0. 9895

0. 9 2 1 0

1.0892

1.0002

Ratio

Average
Seasonal Indices
Table 7 . l O e :

Since,
their
and

seasonal

moving

ratios

average

have

been

values,

Seasonal Indices over Five Years

calculated

the

trend

by

and

obtaining

random

the

ratios of original

components

have

been

values

removed

the values represent only seasonal components.

www.itmuniversityonline.org

Page

to

186

Introduction to Q u a n t i t a t i v e T e c h n i q u e s

07.

Business Forecasting

eBook

and Time Series Analysis

Example 0 3 :

The

profit data of a company

trend

for five

years

is available.

Determine the equation

of the

line. The data is as given below:

Code of Year in

Profit

Cumulative Order

(in Crores) (')

Year

2005 - 06

64

2006 - 07

73

2007 - 08

100

2008 - 09

113

2009 - 10

110
.

Table 7 . l O f :

Solution

03:

Trend equation can be represented


y = a

Profit Data of a Company for Five Years

as:

bt

Constants "a" and

"b" can

be obtained

using

the principle of Least Squares.

Therefore, by using formulae for the calculation of constants a" and

b =

L Y;t;
L t;2

a =

y-

Here n =

b, we have:

- nyt

- n(tl

bt
-

Calculations to obtain the values of

L y,t,,

y, t,

Lt,

Code of Year

Profits

(t;)

(y;)

t?

Yiti

64

64

73

146

100

300

113

16

452

110

25

550

Total

15

460

55

1512

Average

92

Table 7 . 1 0 9 :

www.itmuniversityonline.org

Profit Data of a Company for

Five Years

Page 1 8 7

5.

I n t r o d u c t i o n to Q u a n t i t a t i v e T e c h n i q u e s

07.

B u s i n e s s Forecasting

Substituting

and

Time

eBook

Series Analysis

the values in the above equation, we get:

b = 1 5 1 2 - 5 x 3 x 92 = 132
55-5x9

= 13.2

10

a = 92 - 1 3 . 2 x 3 = 5 2 . 4

Therefore,

the equation of trend

y = 52.4 + 1 3 . 2 t

line is:

Example 04:

Calculate

the

five-yearly

distribution from

Year

moving

averages

from

the

following

data

related

to

income

1990 - 2 0 0 0 :

1990

91

92

93

94

95

96

97

98

99

2000

159

129

150

145

147

164

182

179

155

160

159

Income
(in

thousand
units) (t)

Table 7 . l O h :

Income Distribution from

1990 - 2000

Solution 0 4 :

For the calculation of five-yearly moving

Year

averages, Table

7.lOi

is constructed.

Income

Five-yearty

Five-yearly Moving

(in thousand

Moving Total

Averages

units) (')

1990

159

1991

129

1992

150

730

146

1993

145

735

147

1994

147

788

157.6

1995

164

817

163.4

1996

182

827

165.4

1997

179

840

168

1998

155

835

167

1999

160

2000

159

Table 7. l O i :

www.itmuniversityonline.org

Income

Distribution

From

1990 - 2 00 0

Page

188

Introduction to Q u a n t i t a t i v e T e c h n i q u e s

07.

Business

Forecasting

and Time

eBook

Series Analysis

7 . 1 1 Coding of Time V a r i a b l e
To

s i m p l i fy

the

variable can
from

the

Suppose

be translated

time
the

then

the

time

series

computation

series and

time

mean
is

series

( X )

value is represented

For odd

For even

to a coded
then

is

to

the

new

To code,

that

of five

2003.

l i n e a r trend

form.

subtract

consists

value

translated

of fitting

years

Now

coded

value

series

each

2001,
the

-2,

least

first find

from

(X)

subtract

by

O,

method

out the mean


of the given

2002,

mean

-1,

square

2003,

value

1,

and

2.

time

time

2004,

from
If

the

the

period

periods.

and

the

time

2005,

years,
coded

the
time

by "d" t h e n :

number of time periods in time series, d


n u m b e r of time periods

in time series,

= X - X .

d = 2 x (X - X ) .

Example O S :

The following

is data

related

to the output of a factory for 7 years.

Year

1980

1981

1982

1983

1984

1985

1986

47

64

77

88

97

109

113

Output
(In Tonnes)
Table 7 . 1 1 a :

Calculate

the

production
Solution

in

trend

values

1 9 9 0 and

by

Output of a Factory for 7 Years

using

the

Least

Squares

method.

Also,

forecast

1993.

05:

To calculate trend

values following

table is used.

Year

Output

Coded Time

(X)

(Y)

( d = X - X )

d2

dY

1980

47

-3

-141

1981

64

-2

-128

1982

77

-1

-77

1983

88

1984

97

97

1985

109

218

1986

113

339

13881

595

308

28

Table 7 . 1 1 b :

www.itmuniversityonline.org

Output of a Factory for 7 Years

Page

189

the

I n t r o d u c t i o n to Q u a n t i t a t i v e T e c h n i q u e s

07.

B u s i n e s s Forecasting

When there is odd

Note:

Lx

= 13881 =

and

Time

eBook

Series Analysis

number of elements in t i m e series, coded

timed= X- X

1983

Lv

595
Y=--=-=85
n
7

The trend

b =

dY - ndY

L d'
As

b =

line is

Y e = a + bd

and

a = Y - bd

nci'

d = 0

dY = 308 = 1 1

Ld'

28

a = Y = 85

The trend

line equation

is

Ye= 85 + l l d

Therefore,
Y19so = 85 + 1 1 ( - 3 ) = 52

Y19s1 = 85 + 1 1 ( - 2 ) = 63

Y19s2=85+11(-1)=74

Y19s3 = 85 + 1 1 ( 0 ) = 85

Y1984 = 85 + 1 1 ( 1 ) = 96

Y1985 = 85 + 1 1 ( 2 ) = 1 0 7

Y1986 = 85 + 1 1 ( 3 ) = 1 1 8

Now,
is 1 0 .

since the code for the year


So,

production

1986

is 3,

in tonnes for the years

for the year 1990


1990 and

it will

1 9 9 3 can

be 7, and

for

be calculated

as:

1993

Y1990=85+11(7)=162

Y1993 = 85 + 1 1 ( 1 0 ) = 1 9 5

www.itmuniversityonline.org

Page

190

it

Introduction to Q u a n t i t a t i v e T e c h n i q u e s

07.

Business

Forecasting

and Time

eBook

Series Analysis

Example 0 6 :

From

the following

data,

forecast production

fit a straight

(in lakhs)

Table

Solution

by using

in lakhs for the years 2006 and

Year

Production

line trend

z.i re:

the

Least Square method.

Also,

2009.

1995

1996

1997

1998

1999

2000

2001

2002

45

44

66

59

71

78

75

84

Production in

Lakhs from

1995 - 2002

06:

For the calculation of the trend

values following

table is used.

Production
y

Coded Time

(in lakhs)

d = (X - X) x 2

YearX

dY

d'

(f)
1995

45

-3.5x2=-7

-315

49

1996

44

-2.5x2=-5

-220

25

1997

66

-1.5x2=-3

-198

1998

59

-0.5x2=-1

-59

1999

71

0. 5 x 2 = 1

71

2000

78

1. 5 x 2 = 3

234

2001

75

2.5x2=5

375

25

2002

84

3.5x2=7

588

49

15988

522

476

168

Table 7 . l l d :

When there is even

Note:

Lx
X=

Lakhs from

number of elements

1995

- 2002

in time series, coded t i m e d = (X - X) x 2

= 15988

Lx

= 15988 = 1998.5

Production in

Lv

= 522 = 6 5 . 2 5

The trend

line is

Y = a + bX

www.itmuniversityonline.org

Page

191

I n t r o d u c t i o n to Q u a n t i t a t i v e T e c h n i q u e s

07.

B u s i n e s s Forecasting

b =

dY - ndY

z: d2
As

b =

d
an

a=

and

Time

eBook

Series Analysis

b
d
-

- n,i2

d = 0

Z: d Y

= 476

Z: d 2

168

2.83

a= Y = 65.25

The trend

line equation

is

Ye= 6 5 . 2 5 + 2 . 8 3 d

Y1995 = 6 5 . 25

2. 8 3 ( - 7 ) = 4 5 . 4 4

Y1996 = 6 5 . 2 5

2.83(-5) = 5 1 . 1

Y1997 = 6 5 . 2 5 + 2. 8 3 ( - 3 ) = 56. 76

Y1998 = 6 5 . 2 5

2 . 8 3 ( - 1 ) = 62.42

Y1999 = 6 5 . 2 5 + 2 . 8 3 ( 1 ) = 68.08

Y2000 = 6 5 . 2 5

2.83(3) = 73.74

Y2001 = 6 5 . 2 5

2.83(5) = 79.4

Y2002 = 6 5 . 2 5 + 2 . 8 3 ( 7 ) = 8 5 . 0 6

Therefore,

production

Y2006 = 6 5 . 2 5

Y2009 = 6 5 . 2 5

in

lakhs for the years 2006 and

2.83(7.5

2.83(10.5

2009 can

be calculated

as follows:

2) = 1 0 7 . 7

www.itmuniversityonline.org

2) = 124.68

Page

192

Introduction to Q u a n t i t a t i v e T e c h n i q u e s

07.

Business

Forecasting

and Time

eBook

Series Analysis

7 . 1 1 Chapter Summary

The

arrangement

with

the

of statistical

incidence of time,

is

data
known

the quantitative methods used

Time

series

time.

This could

share

prices,

of a firm,

is

Secular

trend

can

go

as

recorded

at

eq u a l

monthly, quarterly,

weekly

either

are

in

those

certain

movements

cyclical

crimes

in

a city,

intervals

monthly

an

upward

direction

over

concurrence
is one of

period

For example,

of

daily

sales or annual

or

downward

that

profit

the

fluctuation

show

variables

One

in

complete

period

of

oscillation

Random

or

or

irregular

variations

They are due to

human control

The

linear trend

the

line

and

direction.

It

is

data

in

fluctuations

non-recurring

are

and

more
is

than

called

say,

purely

one

year

cycle.

are

Cyclical

7 or 10 years.
random,

erratic,

and

irregular circumstances that are

but are part of the system.

equation

"t"

of

months of successive years.

of

variation.

periodically

longer periods of time,

beyond

in

series analysis

or yearly.

variations generally occur d u r i n g

unpredictable.

Time

is,

both the directions.

variations

oscillatory

termed

series.

that

national income, etc.

Seasonal

The

order,

to predict future demands.

be daily, weekly,

similar pattern d u r i n g

chronological

as time

of observations

temperature,

cannot move in

set

in

the

isY = a + b t ;

time.

The

w h e r e ,a is the Y intercept,

trend

equation

is

determined

b i s the slope of
using

the

Least

Squares principle.

A moving

The

moving

variables.
and

average is used

It

to smoothen the trend

average trend
will

will

be a curve,

be

a straight

if there

is

in a t i m e series.

line,

if there

is

regular

irregular movement due

to

movement

of

business cycles

seasonality.

Seasonal

factors are

factor is used

usually

computed

to adjust forecasts, taking

www.itmuniversityonline.org

on

monthly

or quarterly

basis.

Seasonal

into account the effects of the season.

Page

193

C o n c e p t of P r o b a b i l i t y

I T M

Introduction to Q u a n t i t a t i v e T e c h n i q u e s

08.

eBook

Concept of P r o b a b i l i t y

8 . 1 Introduction
In general, the term

probability means "chance." The Probability Theory

the seventeenth century.

It was first a p p l i e d

In

of

1812,

many

Laplace,

others,

contributed

The

like

rain on

one

J.

his

works,

Bernoulli,

gave

the

De-Moivre,

T.

classical
Bayes,

etc.

of

The

probability
probability

a particular day;

the

is

immense

statements

rise

in

the

in

different

that

sales

you

of

these

statements are
event

and

to

statements

has

not certain.

measure

Ya-Lin Chou,

chapter

two

Thus,

such

of

Mises,

in g a m b l i n g .

Probability.
and

R.A.

Also,
Fisher

probable

the above

statements,

fields

often

science,

encounter are:

engineering,

the forecast

store d u r i n g

introduces

the

Thus,

outcomes,

Probability

probability is used

concepts

of joint,

weekends,

that

is,

not

Theory

the

outcomes

indicate certainty
is

applied.

of

you

will

According

Explain various terms related

Define probability

State the basics of probability

Explain the operations of an event

Explain the addition and

marginal,

and

conditional

Define Bayes' Theorem and

List the applications of probability

www.itmuniversityonline.org

risks

probability.

to probability

multiplication

to

to measure uncertain events.

be able to:

the

of any

addition to this, you will study the Bayes' Theorem.

After reading t h i s chapter,

of

petrol cars.

statements do

the

like

Probability is the science of decision-making with calculated

in the face of uncertainty.

This

Levy,

of a departmental

the prices of diesel cars are likely to go h i g h e r than

Each

definition

later,

to the establishment of the "Theory of P r o b a b i l i t y . "

applicability

medicine,

in

in a game of chance and

has its o r i g i n s in

law of probability

its applications

Page

195

In

I n t r o d u c t i o n to Q u a n t i t a t i v e T e c h n i q u e s

08.

eBook

Concept of P r o b a b i l i t y

8.2

Basic Terminologies

Probability

It can

be defined

Random

An

as, a chance that something

will

happen.

Experiment

experiment whose outcomes cannot

is referred

to as a random experiment.

be

predicted

in

advance with complete certainty

For example, one cannot

predict the n u m b e r that

w i l l show up after rolling a die.

Trial and

Event

Performing

random

outcomes is called

Flipping

Throwing

experiment

is

called

an event or case.

a fair coin

and

and

the

outcome

or a

combination

of

Some examples of trial and event are:

is a trial and

a d i e is a trial

a trial

getting

getting

heads or tails is an event.

a n y number among

the faces,

1,

2,

3,

4,

5,

or

6 is an event.

Favorable Cases

The

desired

example,

results

occurring

when d r a w i n g

d i a m o n d card

is

a card

out

of

from

an

experiment

a pack of cards,

is

called

favorable

case.

For

the cases favorable to drawing

of a

13.

E q u a l l y Likely Cases

When

there

likely case.

is

no

preference to the

For example,

on

rolling

result
a die,

in

an

there

experiment,
is

no

it

is

known

preference for a

to

be an equally

particular outcome

repeatedly.

Notations Used in

Probability Theory

Different notations with their meanings used

Au B:

A union

A n B :

A intersection

n:

the n u m b e r of points in a sample space

S:

sample space

n(A):

the n u m b e r of points corresponding to happening

of event A

n(B):

the number of points corresponding to happening

of event

n(AB):

the

B or ( A +

BJ

to denote events are given below.

implies either A or B, or both

B or (AB)

i m p l i e s A and

B both

number of points corresponding to happening

a null set implies the set

www.itmuniversityonline.org

of both events, A and

is empty

Page

196

I n t r o d u c t i o n to Q u a n t i t a t i v e T e c h n i q u e s

08.

Venn

eBook

Concept of P r o b a b i l i t y

Diagram

diagrammatic

various

representation

sets of variables

that

is a Venn

shows

diagram.

all

possible

For example,

logical
Fig.

relationships

8.2a

between

is a Venn

diagram

for the g i v e n data:


N u m b e r of students in a class:

n =

10

6 out of 10 students who

have opted

for Biology form

4 out of 10 students who

have opted

for Mathematics form

There

are

two

students

who

have

opted

for

section A:

both

nA

section

= n(A) = 6

B:

subjects

= n(B) = 4

form

section A n B :

nAB = n(A n B) = 2

Therefore, the Venn diagram for above information can

be represented as:

AnB

Fig. 8 . 2 a :

Sample Space and Sample

The
A

set of all

sample

referred

is

generally

to as sample

space S =

points.

Diagram

Point

possible outcomes

space

Venn

resulting

denoted

by

For example,

from

an

"S"

or

experiment

(omega)

"Q"

in f l i p p i n g

is called
and

a sample space.

the

outcomes

a fair coin experiment,

the

are

sample

{head, tail}, where heads or tails are the outcomes.

Independent Events

Events
say

A,

are
is

example,
of getting

said

not

to

be

independent

affected

on flipping
a head

by

the

if the

happening

happening

or

or

non-happening

non-happening

a fair coin the event of getting

a head

of

the

of any

other

in the first flip

one event

event

B.

For

is independent

in the second flip.

Dependent Events

The

events

possibility

are

said

of the

to

be

happening

dependent
of another.

www.itmuniversityonline.org

if

the

happening

For example,

of any

if you

one

of

them

draw a card

affects

from

Page

197

the

pack of

I n t r o d u c t i o n to Q u a n t i t a t i v e T e c h n i q u e s

08.

eBook

Concept of P r o b a b i l i t y

well-shuffled

cards and

replace

it

before drawing

draw

is independent of the first draw.

draw

before

drawing

draw as the total

Mutually

Two
them

or

events

restricts

events,

the

A and

B,

second

card,

the

said

to

be

mutually

happening

of

all

the

are

are

such

that

they

there is

Fig. 8 . 2 b :

Exhaustive

not

the

replace the card

second

draw

is

result of the second


selected

dependent

at the first
on

the

first

Events

mutually exclusive. That is,

For example,

then

do

card,

number of occurrences change.

Exclusive

more

the

If you

a second

exclusive

other

cannot

events

happen

no common

if
in

the

happening

the

same

together,

then

of

any

one

experiment.
they

are

point between two sets A and

If

said

of

two

to

be

B.

Mutually Exclusive Events

in flipping of a fair coin experiment,

heads and tails cannot occur together.

Event

An

event which

an

exhaustive

contains of all

event.

possible outcomes

For example,

when

a die

is

resulting
rolled,

from

the

an

experiment

outcomes,

1,

2,

3,

is called
4,

5,

are collectively called an exhaustive event.

8.3

Definitions of Probability

Probability
prevails
between

in

is

the

our

science

day-to-day

of

providing

life.

two teams planning

For

numerical

example,

what

will

a cricket tournament or the

day? The chance that an event will occur is called


which something

measure
be

the

to

the

result

uncertainty
of

fair

coin

flip

possibility of rain on any given

probability.

Probability

is the extent to

is possible.

www.itmuniversityonline.org

that

Page

198

Introduction to Q u a n t i t a t i v e T e c h n i q u e s

08.

eBook

Concept of P r o b a b i l i t y

Classical

Definition of Probability

If an experiment

is performed

in

"n" possible ways, out of which,

"m" ways, then the probability of the h a p p e n i n g


is given
P(E)

by the following

of that event

an

event "E" occurs

E is denoted

in

as " P ( E ) " and

formula:

Number of favorable cases of an event E

Total number of cases

=
p

Where, "p" is the probability of occurrence of an event E

Similarly,

given
P(E')

the

probability

by the following

of non-occurrence of that event

_ N u m b e r of favorable cases of an event E

_ m
1

Total number of cases


e

denoted

by " P ( E ' ) " and

is

formula:

=> i = p

E i s

Where, "q" is the probability of non-occurrence of an event E.

Therefore,

p + q

Properties of Probability

The

probability of an event lies between zero and one,

If P(E)

o,

then event E will not occur at all and

will

If P(E)

1,

then the event E w i l l occur and

be called a certain event.

P(A)

+ P(A')

happening

of an

1,

is

the

result

when

it will

the

including

be called an

probability

of

the

both.
impossible event.

happening

and

non

event when added

Limitations of Probability

A classical definition cannot

It

has a limited

reach and

be used when the total number of outcomes is infinite.

in

some cases,

the total

number of outcomes cannot

be

identified.

Classical

definition

involves

ascertain

probability

in

www.itmuniversityonline.org

the

"equally

likely"

probability.

Thus,

it

fails

random cases.

Page

199

to

I n t r o d u c t i o n to Q u a n t i t a t i v e T e c h n i q u e s

08.

eBook

Concept of P r o b a b i l i t y

Some examples of probability are:


If

an

experiment

outcomes,

of

head

or

possible, that is,

A box

flipping
tail.

fair

coin

Therefore,

is

the

performed,
occurrence

then
of

there

one

will

among

be

two

them

possible

is

equally

contains four

probability

of

balls.

picking

A ball

of red,

particular

orange,

color

ball

green,

and

(for example,

white color,

a white

ball)

each.

will

The

be

.!. .
4

Where,

one

is

the

number of favorable outcomes

and

four

is

the

total

number

of

outcomes.

There

say,

are

seven

in

a week.

can

be

written

Wednesday,

outcomes and

Statistical or
The

days

seven

Empirical

empirical

is the total

The

as,

probability

Where,

of

raining

one

is

on

the

one

particular day,

number of a favorable

number of outcomes.

Definition of Probability

probability

of

an

event

is

defined

as

the

relative

frequency

occurrence of the event when the number of observations is very large.

there are

in

"n" repetitions of an event and

total

particular event

i n n

repetitions.

T h e n , " ( ) " is

of

Suppose

"rn" is the number of occurrence of a

known

as the

relative frequency

of that

event.

Some examples of empirical definition of probability are:

In

basket,

picking

there

are

30

oranges

and

70

up an orange from the basket w i l l

mangoes.

The

probability

of

randomly

l.Q_ = 0 . 3 or 3 0 % .

be

100

boy

the

has

100

probability

crackers.

of

Out

randomly

of those

picking

100

up

crackers,

defective

60

are

cracker

defective.

will

be

Therefore,

0.6

or

happening

to

1
6
00
60/o.

Probability Defined
The

odds

the

probability

and

"q" is the

as "Odds"

of occurrence

are " p : q"and

of

its

of an

event

non-happening.

probability

are

the

If "p"

of non-happening

o d d s a g a i n s t its h a p p e n i n g

www.itmuniversityonline.org

ratio
is

of the

the

of an

probability

probability

event,

then

is, " q : p ," where

of

of

its

happening

odds

in

of an

event

favor of the event

q = 1 - p.

Page 200

Introduction to Q u a n t i t a t i v e T e c h n i q u e s

08.

eBook

Concept of P r o b a b i l i t y

For example,

face is

and

when

a die

is thrown

the

probability

the p r o b a b i l i t y of not getting

of getting

number 6 is

Moreover, the odds against number 6 are

If

you

have

odds

in

favor

probability of the happening

event are

3 : 5 , then the

of

an

event

of that event.

.!:.

number "6" on

- : - = 1 : 5
6
6

= 5 : 1

or

odds

against

For example,

probability of its happening

an

event,

if given

is given

you

odds

against

probability of its

an

event

happening

is:

are

3 : 7,

7
__ =

the

odds

in

by:

favor

find

the

3
3
__ =

are

7 : 3.

Therefore,

the

7
10

7+ 3

8.4

then

can

that, odds in favor of an

3+5
If the

the top

1
Thus, o d d s in favor of number 6 in the throw of a d i e :

the

Basics of Probability

Permutations without Repetition

The

total

number

"r" repetition

p , - (

n!

of

permutations

is denoted

out

by " " P r " and

of "n" different

is defined

objects

taken

at

a time,

without

as:

O < r < n

n - r .

'

Where,

n!=n(n-l)(n-2) ... 3.2.1

For

n = 1, 2, 3, . . .

you get:

1!= 1
2!= 2 x 1
3! = 3 x 2 x 1 and

so on

www.itmuniversityonline.org

Page 2 0 1

I n t r o d u c t i o n to Q u a n t i t a t i v e T e c h n i q u e s

08.

eBook

Concept of P r o b a b i l i t y

Example 0 1 :

How

many

u s i n g each

three-letter

words

can

be

formed

from

the

letters

of the

word

N U M E RA L S ,

letter only once?

Solution O 1 :

You

have to form

word

a three-letter word

without repetition from eight different

letters of the

" N U M E RA L S . "

Therefore,

number of words =

'P

8!

= 8!

(8 - 3)

5!

8 x 7 x 6 x 5 !
=

5!
= 8 x 7 x 6
=

Permutations with

The

total

number

336

Repetition of Objects

of different

permutations

where any of the objects are repeated

any

of "n"

different

objects

number of times:

taken

"r" at

x .. x

time

n = n'

Example 0 2 :

How

many

four-digit

d i g i t is used

numbers can

be

formed

from

the

digits

3,

4,

7,

8,

and

9,

if each

any number of times?

Solution O 2:

Since
one

the

of

repetition

the

thousand's

given

place,

multiplication
4

each,

is allowed,

in

five

ways.

can

be

the

total

the d i g i t

in

Similarly,

occupied

in

number

ten's

five
of

unit

place can
place,

different

ways

of

be

occupied

hundred's
ways.

Repetition of Not All

type,

by

any
and
the

number

is

Different Objects

number of permutations of "n" objects taken all at a time where, " n

one

place,

Hence,

forming

by

625

Permutations for

of

digits

principle,

5 x 5 x Sx 5 = 5

The

of a d i g i t

"n2"

is

similar

and

of

second

type

...

"nk"

is

similar

1"

and

is similar and

of

k"

type,

given as:

www.itmuniversityonline.org

Page 202

is

Introduction to Q u a n t i t a t i v e T e c h n i q u e s

08.

eBook

Concept of P r o b a b i l i t y

Example 0 3 :

How

many

different

words

can

be

formed

by

permuting

all

letters

of

the

word

APPLICATION?

Solution

03:

The word

"APPLICATION" contains

C, T,

0,

L, and

N each occur once.

11

letters,

in which A,

Hence, the total

P,

and

I occur twice and

letters

number of permutations is:

1 1 x lOx 9 x 8 x 7 x 6 x 5 x 4 x 3 x 2'

11!

--- = ---------------

2!

2! 2! 2!

2!

x 2!

19958400
=---,----

4
= 4989600

Example 04:

Ten

books are placed

randomly

in a shelf.

that a particular p a i r of books shall

Always together

Nevertogether

Find

the

number of different arrangements so

be:

Solution 04:

Since

two

Therefore,

books

are

now you

packed

total

you

have n i n e books that can

However, the two packed


Hence,

together,

number

always together is 9!

of

books can

of

regard

be arranged

be arranged

arrangements

x 2!

should

in 9!

them

as

such

that

single

book.

ways.

among themselves in 2!

books

particular

different ways.
pair

of

book

= 725760

Total

number of ways for a r r a n g i n g

Total

number of arrangements such that a particular pair of books is never together:


=

(total

number

number
of

of

ways

arrangements

10 books among themselves is 1 0 1

for
of

arranging

books

such

10
that

books
a

among

particular

themselves)
pair

of

book

is

1 0 ! - ( 2 ! x 9 ! ) = 10 x 9 ! - 2 ! x 9 !

= ( 1 0 - 2 ) x 91

8 x 91

= 2903040

www.itmuniversityonline.org

(total

always

together)

is

Page 203

I n t r o d u c t i o n to Q u a n t i t a t i v e T e c h n i q u e s

08.

eBook

Concept of P r o b a b i l i t y

Combinations

A combination

of "n" different objects taken "r" at a time

a selection of "r" objects out of n objects and

is given as:

is denoted

"C,

by

""C,

"or"

( ) " is

= (n) = ( _n')I
r
n
r .r.I

Remarks:

"C 0 = 1

nc , -

" C0 1
-

Example 0 5 :

bag

contains

random.

Find

10

white,

red,

black,

and

blue

balls.

When

five

balls are drawn

the number of combinations so that two of them are red

and

at

one is black.

Solution O 5 :

You

have to select five balls out of 22

There should

be two

red

balls and

balls.

one

black

ball,

from

10

that

is,

the

remaining

two

balls can

be

either white or blue.


Thus,

these

selected

Two

red

4)-

15!

2!

be

selected

13!

3)1

15

balls.

These

as shown

below.

balls and

one

two

balls

can

be

four can

_ 4

2!

be selected as shown

below.

2! _

6
(4-2)!2!

2!

2!

In addition, one black ball out of three can

= 1 5 x 1 4 x 1 3 ! = l 0 5

13!2!

balls from

4!

can

below.

15!
(15-2)!

(2

balls

as shown

15J=
( 2

two

3!

_ 3

(3-1)'1!

Therefore,

2'

be selected

2'1!

the total

number of ways to

principle of multiplication

select two

red

black

ball

using

is given as:

www.itmuniversityonline.org

Page 204

the

Introduction to Q u a n t i t a t i v e T e c h n i q u e s

08.

eBook

Concept of Probability

8.5

Operations of Events

Operations of events

lead

to formation of new events. These new events can

of the o r i g i n a l sample space.


Union of Two or More

If "A" and

Operations of events are given by the following

be subsets
types:

Events

"B" are two events defined

on a sample space "Q," then " A u B " is the event

that contains all points belonging to either A or B, or both.

Let

0={1,2,3,4,5,6},

Let B

A={l,2,3,4}

{2, 4, 6}

Then, A u B

{ 1 , 2, 3, 4, 6}

Remarks:

A u B = B u A

Au A'= Q
Au,p=A

A u r:J = Q

Intersection of Two or More

If "A" and

Events

"B" are two events defined

on a sample space "Q," then

the event which contains all the points, which are in both, A and

Let

intersection

A r. B

B.

{1, 2, 3, 4, 5 ,6, 7, 8}

Let A = { 1 , 2, 4, 6, 7}
Let B

{2, 4, 6, 8}

Then, A r. B = {2, 4, 6}

Remarks:

A n B = B n A
A r. t = t

A r. A ' = t

A n r:J = A

www.itmuniversityonline.org

Page 2 0 5

is

I n t r o d u c t i o n to Q u a n t i t a t i v e T e c h n i q u e s

08.

eBook

Concept of P r o b a b i l i t y

Complement of an

Event

If "A" is an event defined


points

of Q

that

are

not

simultaneous flipping

on "Q,'' then the complement of A is the event that contains all


in

and

is

denoted

by

A.

For

example,

in

the

event

of

the

of three fair coins, the sample space is

Q = { H H H , H H T , THH,HTH,HTT, THT, TIH, TTI}

The event A getting

at least one head

is:

A = {HHH, HHT, THH, HTH, HTI, THT, TTH}

The event A' =

{TTI}

which

is not in A.

Remarks:
'

(A')

Q'

<1 > ' = 0

q,

Relative Complementation

If A and

B are events,

A' n B

or (B - A ) "

relative

complement

then the

is

the

of

relative complement of A with

event

B with

that

respect

contains

to

A,

all

the

denoted

respect to

points

of

but

B,

denoted

not

by " A n B' or ( A - B ) "

of

A.

is the

that contains a l l the points of A but not of B.

Set Identities

Commutative laws

A u B = B u A

A n B = B n A

Associative laws
o

( A u B) u C = A u ( B u C)

( A n B) n C = A n (B n C)

Distributive laws
o

A u (B n C) = ( A u B) n ( A u C)

A n ( B u C) = ( A n B) u ( A n C)

De-Morgan's laws
o

( A u B)' = A' n B'

( A n BJ' = A' u B'

www.itmuniversityonline.org

Page 206

by"
The

event

Introduction to Q u a n t i t a t i v e T e c h n i q u e s

08.

8.6
To

eBook

Concept of P r o b a b i l i t y

Probability Laws

calculate

sample

the

probability

space and

calculate

identify

probability

in

of various

the

such

events,

corresponding

cases,

it

may

sample

different

laws

not

always

points

of

be

possible

for a said

addition,

to

event.

draw

Hence,

multiplication,

etc.

a
to

are

applied.

Addition

It states

Law of Probability

that

exclusive

the

probability of occurrence of either an event "A" or "B" of two

events

represented

is

equal

as, "P(A u B)

Here, "A" and

to

the

P(A)

sum

of

their

individual

probabilities.

mutually

It

is

also

+ P(B)"

"B" are mutuall y exclusive events.

" ( A u B ) " denotes the event that A or B

occur when the experiment is performed.

If

the

stated

two

events,

as the

A and

probability

are

not

mutually

exclusive,

of occurrence of either an

to the probability that event A occurs,

plus the

then

the

addition

event A or event

probability that event

B,

or

law

both

can

be

is equal

B occurs m i n u s the

probability that both events occur.

It can be represented

as,

P(A u B) = P(A) + P ( B ) - P(A n B)

Here, " ( A n B ) " denotes the event,

Similarly

addition

law

of

that both A and

probability

same sample space "S" can

be stated

for

three

B occur.

events

"A,"

\\B,"

and

\IC"

defined

on

the

as follows:

P(A u B u C) = P(A) + P(B) + P(C) - P(A n B) - P(B n C ) - P(A n C) + P(A n B n C)

Where " ( A u B u C ) " denotes the occurrence of either an event A or event

or

any

two

of them

occur,

or

all

three

occur,

" ( A n B rv C)"

denotes

the

B,

or event

event,

that

the three events occur.

Example 0 6 :

Find the probability that at least one head occurs when two fair coins are flipped.

Solution

06:

Let A be the event of getting

heads on the first coin.

Let B be the event of getting

heads on the second

www.itmuniversityonline.org

coin.

Page 207

all

I n t r o d u c t i o n to Q u a n t i t a t i v e T e c h n i q u e s

08.

eBook

Concept of P r o b a b i l i t y

The p r o b a b i l i t y of getting

a head on flipping

The probability of getting

a tail on flipping

a fair coin

a fair coin

is

is

P(A)

P(A')

P(B)

P(B')

= ;

.!:.
2

The event that there is a head

According

to the addition

the event of getting

second

least one coin can

law of probability,

P(A v B) = P(A) + P ( B ) - P(A

Here

on at

r,

it can

be written as P(Av B)

be obtained

as:

B)

heads

on

the first coin

and

the event of getting

heads on

the

coin are independent events.

If two events are independent then

( A n B) = P(A) x P(B)

P(A v B) = P(A) + P ( B ) - P(A r. B)

=;+;-(;x;)=!

Example 0 7 :

single

drawn

card

is

drawn

from

deck

of

52

cards.

What

is

the

probability

that

the

is a queen or a diamond?

Solution O 7 :

Let "Q" be an event that the card

drawn

is a queen.

Let "D" be an event that the card

drawn

is a d i a m o n d .

As there are four queens in a deck of 52 cards

Therefore,

P( Q) =
5

As there are

13

diamonds in a deck of 52 cards


13

Therefore,

P(D)=
52

Also as there is o n l y one queen of diamonds in a deck of 52 cards,

Therefore,

P(Q n D) =
512

The probability that the card


P(Q u D)

drawn

is a queen or a diamond

is given a s :

P(Q) + P(D) - P(Q n D)

4
13
1
- + - - 52
52
52

www.itmuniversityonline.org

Page 208

card

Introduction to Q u a n t i t a t i v e T e c h n i q u e s

08.

eBook

Concept of P r o b a b i l i t y

16

= - = -

52

13

The probability that a card

drawn from a deck of 52 cards is a queen or diamond

is

Example 0 8 :

Two
the

dice,
sum

showing
A, B,

B,

of

green

the

the

and

points

number

one

on

red

them

one.

are
is

thrown

odd

Describe

A u B, A n B, A, ( A u B),

Solution

The

one

and

and

the

simultaneously.
"B"

be

the

complete

Let "A"

event

sample

that

be

at

space,

the event

least

also

one

the

find their probabilities.

sample

space

"S"

is as given

corresponding

to

the

experiment

of

throwing

two

below.

(2, 1), (2, 2), (2, 3), (2, 4), (2, 5), (2, 6)
(3, 1), (3, 2), (3, 3), (3, 4), (3, 5), (3, 6)
(4, 1 ) , (4, 2), (4, 3), (4, 4), (4, 5), (4, 6)
( 5 , 1 ) , ( 5 , 2), (5, 3), (5, 4), ( 5 , 5), ( 5 , 6)
(6, 1 ) , (6, 2), (6, 3), (6, 4), (6, 5), (6, 6)}
n(S)

36

A = The event that the sum of the points on the dice is odd.
Therefore,

sample space for A is:

A = { ( 1 , 2), ( 1 , 4 ) , ( 1 , 6), (2, 1), (2, 3), (2, 5)


(3, 2), (3, 4), (3, 6), (4, 1 ) , (4, 3), (4, 5)
(5, 2), ( 5 , 4), (5, 6), (6, 1 ) , (6, 3), (6, 5)}

n(A)

Therefore

18

P(A)
'

n(A)
n(S)

18
=

36

_! and

P(A)

1 - P(A)

Therefore,

1 - _!

B = The event of atleast one d i e is showing

_!
2

number 1.

sample space for B is:

B = { ( l , 1), ( 1 , 2), ( 1 , 3), ( 1 , 4), ( 1 , 5), ( 1 , 6)


(2, 1), (3, 1 ) , (4, 1 ) , (5, 1 ) , (6, 1 ) }
Here,

is

events,

S = { ( 1 , 1), ( 1 , 2), ( 1 , 3), ( 1 , 4), ( 1 , 5), ( 1 , 6)

Here,

die

08:

simultaneously

Here,

that

n(B)

11
n(B)

Therefore,

P(B)

11

= - = -,

n(S)

36

www.itmuniversityonline.org

P(B)

n(B)
=

1-

= - =

n(S)

11
1- -

36

25
= -

36

Page 209

dice

I n t r o d u c t i o n to Q u a n t i t a t i v e T e c h n i q u e s

08.

eBook

Concept of P r o b a b i l i t y

A n B = The event that sum


Therefore,

is odd and

sample space for

atleast one d i e is showing

number 1.

A n B is:

A n B = { ( 1 , 2) (1, 4), ( 1 , 6), (2, 1), (4, 1), (6, 1 ) }


n(A n B)

Here,

= 6
n(A n B)

Therefore,

P(A

rv

B)

= - = -

n(S)

36

A u B = The event that the sum is odd or at least one is showing


Therefore,

Au B

sample space for

Au B

number 1.

is:

= { ( 1 , 1 ) , ( 1 , 2 ) , ( 1 , 3 ) , (1, 4), ( 1 , 5 ) , ( 1 , 6),


(2, 1 ) , (3, 1 ) , (4, 1 ) , ( 5 , 1), (6, 1 ) , (2, 3),
(2, 5), (3, 2), (3, 4), (3, 6), (4, 3 ) , (4, 5 ) ,
(5, 2), (5, 4), (5, 6), (6, 3), (6, 5)}

Here,

n(A u B)

23
23

Therefore,

P( A u B)

n(A u B)

n(S)

36

P(A u B) = 1 - P(A u B)

1 - 23

36

_!2
36

Example 09:

If two dice are

thrown

the numbers appearing

Greater than 9

Neither 7 nor 1 1

simultaneously,

then

what will

on the face of two dice

be the

probability

that the sum

is:

Solution 0 9 :

Sample space corresponding

to the experiment of throwing

two dice simultaneously is:

S = { ( 1 , 1 ) , ( 1 , 2), ( 1 , 3), ( 1 , 4), ( 1 , 5), ( 1 , 6)


(2, 1 ) , (2, 2), (2, 3), (2, 4 ) , (2, 5 ) , (2, 6)

(3, 1 ) , (3, 2), (3, 3), (3, 4), (3, 5), (3, 6)
(4, 1 ) , (4, 2), (4, 3), (4, 4), (4, 5), (4, 6)
(5, 1), ( 5 , 2), ( 5 , 3), (5, 4 ) , (5, 5), (5, 6)
(6, 1 ) , (6, 2), (6, 3), (6, 4), (6, 5), (6, 6)}
Here,

n(S)

36

Let "T" be the event of the sum of the numbers appearing

www.itmuniversityonline.org

on the face of two dice.

Page 2 1 0

of

Introduction to Q u a n t i t a t i v e T e c h n i q u e s

08.

eBook

Concept of P r o b a b i l i t y

Probability that the sum

is greater than 9 means

Therefore, the required event "(T

P(T > 9)

> 9)" can happen in the following ways:

T=10, T=11, T = 1 2

Hence,

by the addition

law of probability:

P(T > 9) = [P(T = 10) + P(T = 11) + P(T = 12)]

In the sample space of two dice, there are 36 sample points.


The sample points favorable for sum

10 are (4, 6 ) ,

1 1 are (6,

(6, 4 ) ,

and

(5,

5).

P(T = 10) = -2_


Therefore,

36

The sample points favorable for sum

Therefore,

P(T

11)

5) and

(5,

6).

.3...
36

And

the sample

Therefore,

P(T

points favorable for s u m =

12

is (6,

6).

12) = ....!:...

36

Therefore,

P(T > 9) = [P(T = 10) + P(T = 1 1 ) + P(T = 12)]

= - + - + -

36

36

36

= - = -

36

Let A be the event that sum of the numbers appearing

on the face of two dice is 7.

Let B be the event that sum of the numbers appearing

on the face of two dice is

The sample space corresponding to the event A will

11.

be:

A = { ( l , 6), (6, 1 ) , (2, 5), ( 5 , 2), (3, 4), (4, 3)}


Here,

n(A)

Therefore,

= 6
n(A)
6
1
P(A) = -- = - = n(S)
36
6

The sample space corresponding to the event B will

be:

B={(5,6),(6,5,)}

www.itmuniversityonline.org

Page 2 1 1

I n t r o d u c t i o n to Q u a n t i t a t i v e T e c h n i q u e s

08.

eBook

Concept of P r o b a b i l i t y

Here,

n(B)

Therefore,

P(B) = n(B)

n(5)

The events

A, B

18

36

denote the events of not getting

Therefore,

An B

is the event getting

Therefore,

P(A n B) = P(A u B)

sum

sum

7 and

11

respectively.

neither 7 nor 1 1

=1-P(AuB)

= 1 - [ P ( A ) + P(B) - P(A n B)]


= 1 - P(A) - P(B)

Here,

P(A n B) = $ ,

because A and

P(A n B) = 1 -

Therefore,

Conditional

B are disjoint events.

18

Probability

Probability "P(A)" of an

event A represents the

result

in

in

an

experiment.
must

be

in

outcome
If you

a "set

have

"set
prior

B" of 5,

that the outcome will also


is

read

as the conditional

A"

information

then
be

relative

in

this
B.

likelihood

to
that

information

the

sample

the

outcome

must

T h i s evaluated

probability

that a random

be

of the

used

probability

of the event A,

space

given

"5"

experiment will
of

the

random

experiment

to evaluate the

is denoted

that

random

likelihood

as "P(A/B)" and

the event

B has already

occurred.

Marginal

Probability

The probabilities of the happening


n (A)

P(A) = n (5)

n (B)

and

P(B) = n (S)

of A and

B,

are defined as

.
, respectively.

They are also known as marginal probabilities of A and

B.

Joint Probability

It

is

the

probability of a joint event An B or AB,

denoted

as "P(AB) "and

is given

by

following formula:

P(A n B)

P(AB)

n(AB)

n(S)

www.itmuniversityonline.org

Page 2 1 2

the

Introduction to Q u a n t i t a t i v e T e c h n i q u e s

08.

eBook

Concept of P r o b a b i l i t y

Thus,

P(AB)

is the joint probability of the happening

Let "A" and "B" be two events defined


then the conditional

probability

P(A/B) = P ( A " B) P(B)

The

P(B)

'

probability

of

given

of both events, A and

B together.

on the sample space "S" of a random experiment,

P(A/B)

is given

by the following

already

happened,

formula:

0
>

B,

given

by the following

that A

has

is denoted

by

"P(B/A)" and

is

formula:

P(B/A) = P ( A " B) P(A)


0
P(A)
'
>

Suppose a candidate appears for two examinations successively.


Let the sample space S for his possible result

be:

S = {PP, PF, FP, FF}, n(S) = 4

Where,
Let P be the candidates who have passed
Let F be the candidates who have failed
Let A be the event that candidate

probability of A is

P(A)

Let

"B"

be

the

event

space corresponding

Then the marginal

that

the examination
passed

both the examinations,

then

the marginal

= .!

n ((A))

has

the examination

the

candidate

to the event B w i l l

probability of B is

be

passed

at

least

one

examination.

The

sample

B = {PP, PF, FP}

P(B) = n ((B))
n

The sample space corresponding to the event

I
4

A n B = {PP, PF, FP}

n(A rv B) = 3

n(AnB)
Therefore,

P(A n B) =

Probability that a candidate


passed

at

3
= -

has

least one examination

." It is given

by the following

www.itmuniversityonline.org

passed

both

the examinations

is the conditional

given

p r o b a b i l i t y and

that

the candidate

is denoted

by "P(A/B)

formula:

Page 2 1 3

I n t r o d u c t i o n to Q u a n t i t a t i v e T e c h n i q u e s

08.

eBook

Concept of P r o b a b i l i t y

P(A/B) = P(A n B) = 1 / 4 =

_!

P(B)

3 / 4

Properties of Conditional

If "A" and
"S

"B"

are

Probability:

mutually

events

defined

on

the

same

sample

space

" then
'

'

P(A/B) = P(A n B) = P($)


P(B)

exclusive

= O

P(B)

If A is a subset of B that is

P(A/A') = P(A n A ' ) =

an

P(A')

P($)

= O

P(A')

A c B then,

P(B/A) = P(A n B) = P(A) = l


P(A)

Example

10:

is

dig it

P(A)

selected

at

random

from

dig its

A = { 1 , 3, 5, 7, 9 } , B = {2, 3, 5, 7}, C = {6, 7, 8, 9 } .

Solution

Find

to

9.

Consider

P(A/B) and

the

events

P(B/C).

10:

Since a d i g i t

equal and

is selected

at

random,

the

probability

of selecting

any d i g i t

from

1 to 9

is
9

P(A)

= P(l)

+ P(3)

+ P(5)

+ P(7)

+ P(9)

1
1
1
1
1
= - + - + - + - + 9

5
=

9
P(B)

= P(2)

+ P(3)

+ P(5)

+ P(7)

+ P(7) + P(B)

+ P(9)

= - + - + - + -

4
=

P(C)

= P(6)

1
1
1
1
= - + - + - + 9
9
9
9
4
=

9
( A n B)

= {3, 5, 7}

Therefore,

www.itmuniversityonline.org

Page 214

is

Introduction to Q u a n t i t a t i v e T e c h n i q u e s

08.

eBook

Concept of P r o b a b i l i t y

P(A n B) = P(3) + P(S) + P(7)

P(A n B) =

(B n C) = {7}

P(B n C) = P(7) =

P(A n B)
Therefore,

P(A/B)

3 /9

P(B)

P(B/C)

P(B n C) =
P(C)

Example

bag

1/9

4/9

!
4

blue

and

other without replacement.

Total

4/9

11:

contains

Solution

= - - = -

red

Find

balls.

Two

balls

are

drawn

the p r o b a b i l i t y that both

at

random,

balls drawn are

one

after

the

red.

11:

number of b a l l s =

Let A be the event of getting

a red

ball

in the first attempt.

Let B be the event of getting

a red

ball

in the second

Therefore,

Probability

P(A)

attempt.

of d r a w i n g

red

ball

in

second

draw,

given

that

the

first

ball

selected

red:
3
P(B/A)= -- =
3+ 5

Therefore, the probability that both the balls are red:


P(A n B) = P(A) P(B / A)

= - X - = -

www.itmuniversityonline.org

Page 2 1 5

is

I n t r o d u c t i o n to Q u a n t i t a t i v e T e c h n i q u e s

08.

eBook

Concept of P r o b a b i l i t y

Law of Multiplication

For any two events A and

Also,

B,

P(A n B) = P(A) P(B

P(B n A ) = P(B) P(A / B ) ;

If A and

P(B)

B a r e independent, then

I A);

P(A)

P(A n B) = P(A)P(B),

P(B n A ) = P(B)P(A)

Remarks:

If A and

P(at least one occurrence)

For

example,

B are independent events,

if

two

fair

coins

are

then

A and

are also independent events.

1 - P(no occurrence)

flipped,

then

the

probability

of

getting

heads

on

at

least one of the two coins is:


P(at

least one head

occurs)

1 -

P(no head

occurs)

= 1 - P(tails on both sides)


= 1 - P(tails on one coin and tail on the other coin)
= 1 - P(T) P(T)

=1-(X)
=1-()
=

Example

[!)

12:

The odds against student A solving


of student

B solving

independently, find

Solution

these

Mathematical

problems are

12:14.

problems are 7 : 5 and

If both

of them

the probability that these problems w i l l

try to

the odds in favor


solve the

problem

be solved.

12:

Let A denote the event that student A solves the problem.


Let B denote the event that student B solves the problem
Given that, odds a g a i n s t A are 7 : 5 .
Therefore,

odds in favor of A will

Therefore,

P(A)

5
-

= 2.

5+ 7

12

12
Similarly,

P(B)

12

=
12 + 14

P(problem w i l l

be 5 : 7

be solved)

26
=

1 - P(problem

www.itmuniversityonline.org

is not solved)

Page 2 1 6

Introduction to Q u a n t i t a t i v e T e c h n i q u e s

08.

eBook

Concept of P r o b a b i l i t y

Since, A and

Bare

1 - P(A')P(B')=

1 - P(neither A nor B can solve the problem)

1 - P( A' n B ' )

1 -

P( A')P(B')

independent, A' and

1 - [1

- P(A)]

independent.

[ 1 - P(B)]

1 - [1-/2][1-

1 -

!]

7
14
-x-

12
=

B' w i l l also be

26

1 - 0.314

= 0.6858

Example

13:

The probability that A hits the target is

.!

and

the

probability that B hits the target

Both fire once at the target, find

the

Both hit the target

The target is hit exactly once

Solution

probability that:

Let A denote the event that A hits the target.


Let B denote the event that B hits the target.

P(A)

.!:_ , P(B) = .!:_


5

3
4

Therefore,

P(A')

1-

P(A)

=
5

P(B')

1-

P(B)

P(both

hit the target)

P(A n B) =

P(A)P(B),

1
=

Since

A n B'

has hit exactly once)

and

A' n B

since both A and

B a r e independent.

- x - = -

P(the target

1
3

15

P(either A or B hit the target)

P((A n B') u P(A' n B))

are disjoint events:

www.itmuniversityonline.org

.!
3

13:

Given that,

is

Page 2 1 7

I n t r o d u c t i o n to Q u a n t i t a t i v e T e c h n i q u e s

08.

eBook

Concept of P r o b a b i l i t y

P((A n B') u P(A' n B))

P(A)P(B') + P(A')P(B)

1
=

- x - + - x -

Therefore,

by independence

P(the target has hit exactly once)

15

Example

14:

Candidate X can
of the

same.

solve 85% of the Accounting

When an Accounting

that at least one of them

Solution

problem

problems while candidate Y can solve 95%

is selected

at

random,

what

is the

probability

will solve it?

14:

Let A be the event that candidate X can

solve the problems

Let B be the event that candidate Y can solve the problems

Given that,
Then,

P(A)

= 0 . 8 5 and

by theory of addition

Here A and

P(B)

= 0.95,

you

have to find

law of probability:

P(AuB).

P(A u B) = P(A) + P(A)- P(A ,.-, B)

B a r e independent events.

Hence P(An B) can

be found

by

law of multiplication a s :

P(A n B) = P(A) P(B) = 0 . 8 5 x 0 . 9 5 = 0 . 8 1

Substituting

the values,

you get:

P(A u B) = 0 . 8 5 + 0 . 9 5 - 0 . 8 1
P(A u B) = 0 . 9 9

Thus,

the probability that at least one of the candidates w i l l solve the problem

Example

From

15:

a division

A can

solve

solve is

is 0 . 9 9

1/3.

three

students A,

problem

in

B,

and

Mathematics

If all of them try

is

C a r e selected
1/4,

that

independently, find

at

B can

random.
solve

is

The
1/3,

probability
and

that

that

C can

the probability that the problem will

be

solved.

Solution

The

15:

problem

is solved

by at

least one of them,

that

is,

either A,

B, or C,

or two of them,

or all of them solve the problem.


Therefore, by the law of total p r o b a b i l i t y :

www.itmuniversityonline.org

Page 2 1 8

Introduction to Q u a n t i t a t i v e T e c h n i q u e s

08.

eBook

Concept of P r o b a b i l i t y

P(A u B u C) = P(A) + P(B) + P(C) - P(A n B) - P(B n C) - P(A n C) + P(A n B n C)

This problem can

be solved

in two w a y s :

Method I :

Since A,

B, and

C a r e independent events

1
P(A

r-,

B J = P(AJP(BJ =

1
P(B n C J = P(BJP(CJ =

l2

1
x

1
P(A n CJ = P(AJP(CJ =

1
= g

1
x

l2

=
3

1
P(A n B n CJ = P(AJP(BJP(CJ =

1
=

36

P(A u B u C) = P(A) + P(B) + P(C) - P(A n B ) - P(B n C ) - P(A n C) + P(A n B n C)

1
Thus,

P(A u B u CJ = 4 + 3 +

3 - 12 -

9 - 12 + 36

2
=

Method II:

The

probability that

none of A,

B,

or C are able to solve the problem when each of them

tries independently.

P(A does not solve) x P(B does not solve) x P(C does not s o l v e ) =

3
2
2
-x-x-

4
Therefore,

1
=

P(at least one s o l v e s ) = 1 - P ( n o n e solves)

= 1 - 3
2
=

Therefore

it is the same as the value of probability obtained

in method

I.

Example 1 6 :
A car rental agency
randomly selected

has

16 compact cars and

for a safety check,

12

intermediate size cars.

how many ways can they

If 5 of them

be selected

so that there

are:

2 cars of compact

At least 2 cars are of compact size

At the most,

All

size

2 cars are of compact size

5 cars are of compact size

www.itmuniversityonline.org

are

Page 2 1 9

I n t r o d u c t i o n to Q u a n t i t a t i v e T e c h n i q u e s

08.

Solution

1.

eBook

Concept of P r o b a b i l i t y

Out

16:

of

selected

cars,

are

of

compact

size,

therefore,

there

will

be

cars

of

are

of

intermediate size.
1
2 cars of compact size is

The number of ways of selecting

1}
3 cars of intermediate size is (
)

Therefore,

the

compact size
.

.
rs

m u l t i p l i ca t i o n

(16)

( 1 2 )

2.

Selecting

at

least 2

principle

1 6 !

of

1 2 !

cars

out

of

which

26400

91 31

141 2 1

(z 2)

selection

;J

The number of ways of selecting

compact cars out of 5 selected

cars

is equal to the following

four possible combinations.

Combination

1:

2 compact cars and

3 intermediate size cars

Combination 2 :

3 compact cars and

2 intermediate size cars

Combination 3 :

4 compact cars and

1 intermediate size car

Combination 4 :

5 compact cars and

no intermediate size car

Using

the multiplication and

16!

=
14!2!

=
=

3.

12!

16!

x--+

9!3!

addition principle,

12!

16!
+

13!3!

10!2!

12
x

12!4!

the total

16!

number of ways

is:

11!5!

26400 + 36960 + 21840 + 4 3 6 8


89568

At the most,

2 cars are of compact size:

Combination

Combination 2 :

1 car of compact size and 4 of intermediate size

Combination 3 :

0 car of compact size and

Using

1:

either 2 cars are of compact size and

the multiplication and

www.itmuniversityonline.org

3 of intermediate size

5 of intermediate size

addition principle, the total

number of ways

is:

Page 220

Introduction to Q u a n t i t a t i v e T e c h n i q u e s

08.

eBook

Concept of P r o b a b i l i t y

16!

12!

16

12!

x--+

14! 2!

12!

x--+ 1 x

9! 3!

8! 4!

7! 5!

= 35112

4.

All cars are of same size;


Using

either all

the multiplication and

16!

addition

principle,

the total

n u m b e r of ways is:

12!
x 1

11!

5 are compact cars or all are of intermediate size.

+ 1 x--

5!

7! 5!

= 5160

8.7

Bayes' Theorem

This chapter discusses


taken

place and

the

are known to be "a

of the

happening

the computation

happening

of a favorable event

priori." However,

of the event

of probability,

has to

when the event

is computed,

it

where

be

physical

known.

experiment

Such

has already occurred

is called

"a

has

probabilities

and

the cause

posterior." The following

topic

will cover such cases of probability.

Bayes'

Theorem

is

named

estimates

the

probability

example,

regarding

after
of

Thomas

the

cause

particular

Bayes.
by

ailment

This

which
this

an

there

events

each

be

an

B,, B

of

the

event
B"

A,

has

events

which

happened.

and

of

P(B,) and P(A/B,)

are known.

Now,

is

due

suppose
to

it

known

B1,orB2,or

happening

of

A.

can

Bayes'

Also,

conditional

that

. . . ,or

happen,

Bn.

event
That

Theorem

A
is,

gives

only

suppose

events

has

if

this

event

one

the

(A

of

theorem

of

may

could

probability

have

be

occurred.

used

to

I
I

A)

P(B,) P(A

L P(B,)

P(A

B,)

(i

B,) '

www.itmuniversityonline.org

2 .. .

For

find

the

"n"

probabilities

I B,)(i

B,

of

of

= 1, 2, 3 . . . n)

but

it

is

criteria

information,

in

mutually

the

not
was

terms

the

are

of

happening

known.

known
the

exclusive

cause

it

for

probabilities,

n)

'

Page 2 2 1

of

Thus,

whether

follows:

P(B

that

patients.

happened

which

theorem

probability of cause due to which the a i l m e n t occurs in

Let

is

is

the

as

I n t r o d u c t i o n to Q u a n t i t a t i v e T e c h n i q u e s

08.

eBook

Concept of P r o b a b i l i t y

Example

There

and

B1

are

box

17:

two

B2

boxes

contains

B1

and

2 white

B2

where

balls and

given that the ball drawn from

Solution

box

B1

3 black

contains

balls.

Find

3 white

the

either of the two boxes

balls

and

black

probability of selecting

B1or

B2

balls

box

is white.

17:

Let us define the events as follows:


A:

The ball drawn

is white

B1:

Box selected

is

B1

B2,

Box selected

is

B2

A / B1 : White ball

is drawn, given that the box selected

A/8

is drawn, given that the box selected

2:

B1 / A :

B2 / A :

You

White

Box

Box

ball

A)=

B2

is selected, given that the ball drawn

I A)

P(B,

P(B;)P(A

is white

I B;)

i = 1, 2

L P(BJP(A I B ; ) '

P(B

is

B1

is selected, given that the ball drawn is white

have to find

P(B

is

)
1

1
= P(B,) = 2

P(A/81) = 3 / 5
P(A/82) = 2 / 5
_
Th

f
ere ore,

P(B

I A)

P(B1 )P(A

I 81)

I B i ) + P(B2 )P(A I 8 2 )

- P(Bi)P(A

3
-

10

=
3
2
- + -

10

3
-

10
3

Thus,

the

probability

of

selecting

box

B1

given

that

ball

drawn

is

white

is

and

probability of selecting

box

B2

www.itmuniversityonline.org

is

1- =

Page 222

the

Introduction to Q u a n t i t a t i v e T e c h n i q u e s

08.

eBook

Concept of P r o b a b i l i t y

Example

Three

18:

machines

produce

defective

particular d a y ,
the

producing

proportion

separately

and third

machines.

Solution

18:

M 1 , M 2 , and

Let

P(M

M3

30%,

of

a u n i t of output

probabilities

Let

45%,

of

items,

is

and
as

selected

the

25%

0.04,

at

defective

of

0.06,

random
item

the

is

being

total

and
found

output

0.03,
to

by

be the probability of an

item

being

produced

by machine

Let

P(M )

be the probability of an

item

being

produced

by machine

Let

P(M

be the probability of an

item

being

produced

by machine

Given that:

You

P(Mi) = 0 . 4 5 ,

know that, total

P(M

the

first,

On

Find

an

"D"

item that

represent

probabilities

out

second,

P(M 2) = 0 . 3 0 ,

probability

P(M 3 ) = 0 . 2 5

is always eq u a l to one.

of

has to come from any of the three machines.

defective

defective

items,

items

then

being

P(D

M i ) , P(D

produced

by

M 2 ) and P(D

the

machines

M3)
M

1,

denote

2,

and

the

respectively.
P(D

I M i ) = 0.04

P ( D / M 2 ) = 0.06
P(D/M3)=0.03

Let "P(M
Using

P(M

/ D ) " be the probability that an

item

produced

by m a c h i n e

being

defective.

Bayes' Theorem:

/D) =
1

)+P(M,)+P(M,)= 0.45+0.30+0.25 = 1

That is,

Let

defective.

to

be three machines a n d :

known

respectively.

be

produced

are

P(Mi) P(D / M i )

L P(M;)

P(D I M ; ) '

(i = 1

2
'

3)
'

P(M1) P(D I M1)


=------------
P(M1) P(D I M1) + P(M,) P(D I M , ) + P(M,) P(D I M , )
0 . 4 5 x 0.04

=-------------0 . 4 5 x 0.04 + 0 . 3 0 x 0 . 0 6 + 0 . 2 5 x 0 . 0 3

www.itmuniversityonline.org

Page 223

I n t r o d u c t i o n to Q u a n t i t a t i v e T e c h n i q u e s

08.

eBook

Concept of P r o b a b i l i t y

0.018

=-------0 . 0 1 8 + 0.018 + 0.0075


0.018
= - --

0.0435
= 0.4138

Thus,

the probability of defective items produced

Similarly,

"P(M

/D)"

be

the

probability

that

an

by machine

item

M1

is 0 . 4 1 3 .

produced

by

machine

being

defective.

Using

Bayes' Theorem:

P(M2/D)=

P(M2)P(D/M2);

L P(M,)

(i=l,2,3)

P(D / M ; )

P(M ) P(D I M )
2
2
=--------------P(M,) P(D I M 1 ) + P(M ) P(D I M ) + P(M,) P(D / M )
2
2
3
0.30 x 0.06

=-------------0 . 4 5 x 0.04 + 0 . 3 0 x 0 . 0 6 + 0 . 2 5 x 0 . 0 3
0.018

0.018

0.018 + 0.018 + 0.0075


=

0.0435

0.4138

Thus, the probability of defective items produced

Similarly,

"P(M

/ D)"

be

the

probability

that

an

by machine

item

M
2

produced

is 0 . 4 1 3 8

by

machine

defective.

Using

Bayes' Theorem:

P(M3/D)=

P(M3)P(D/M3);

L P(M,)

(i=l,2,3)

P(D I M ; )

P(M

)
3

P(D / M

)
3

=--------------P(M,) P(D / M 1 ) + P(M,) P(D / M,) + P(M,) P(D / M 3 )


0 . 2 5 x 0.03
=

-------------

0 . 4 5 x 0.04 + 0.30x 0 . 0 6 + 0 . 2 5 + 0 . 0 3
0.0075
0.0435
= 0.1724

www.itmuniversityonline.org

Page 224

being

Introduction to Q u a n t i t a t i v e T e c h n i q u e s

08.

eBook

Concept of P r o b a b i l i t y

Thus, the probability of defective items produced

8.8

by machine

Applications of Bayes' Theorem

Bayes' Theorem
engineering

has various applications.

processes

to

telecommunications to
received.

Also

test

the

It

quality

is used

of

predict the variations

diagnostic

test

conducted

in exploratory and

product.

It

in transmitted

for

medical

also

of Bayes' Theorem can

Example

responsible
the

is

experiencing

are

price,

dissatisfaction

quality,

and

its

in

applications

treatment

is

an

the message

ideal

scenario

By

conducting

respectively.

dissatisfied

With

overall,

this

what

with

survey

probabilities of a customer going

below.

away

the

the

three

amongst

the

its

customers.

By conducting

dissatisfied

information,
are

amongst

due

that

to

these

wants
it

is

analysis,

are

customers,

company

probabilities

factors

an

The

0.45,

it

to
due

know
to

it

is

0.40,

also

factors

three

factors

assessed
and

is
if

0.4,
a

price,

0.6,

B, be a customer is dissatisfied d u e to price.

Let

be a customer is dissatisfied

due to quality.

Let

B,

be a customer is dissatisfied

due to packaging.

Let A be a customer is dissatisfied

P(B

P(B

= 0.45,

P ( A / 8 1 ) = 0.4

= 0.40,

P(A

= 0.15

Find out

Using

I B2)

P(A / 8 3 )

P(B1

quality

or

overall.

= 0. 6

= 0.5

A), P(B2

A), P(B3

A)

Bayes' Theorem:

www.itmuniversityonline.org

and
is

Given:
P(B

the

customer

19:

Let

0.15,

evaluated

packaging?

Solution

for

A few more applications

through the examples g i v e n

packaging.

probabilities of dissatisfaction

respectively.

0.5,

in

19:

company

that

be better understood

finds

troubleshooting

messages from

application of Bayes' Theorem to check the authenticity of tests.

is 0 . 1 7 2 .

Page 2 2 5

I n t r o d u c t i o n to Q u a n t i t a t i v e T e c h n i q u e s

08.

eBook

Concept of P r o b a b i l i t y

P(B

/A)-

P(B1)P(A/Bi)

1
- P(B1 )P(A

I B i ) + P(B2 )P(A I B 2 ) + P(B3 )P(A I 8 3 )


0.45 x 0.40

=-----------0.45 x 0.4 + 0.6 x 0.40 + 0 . 5 x 0 . 1 5


0.18
=

0.495
= 0.3636

Therefore,

a customer is dissatisfied

due to price with

probability

is 0 . 3 6 3 6

Similarly,

I A) _

P(B

P(B2 )P(A

2
- P(B1 )P(A

I B2)

I B 1 ) + P(B2 )P(A I B 2 ) + P(B3 )P(A I 8 3 )


0.40x0.60

= -------------

0.45 x 0.4 + 0.6 x 0.40 + 0 . 5 x 0 . 1 5


0.24
=

0.495
= 0.4848

Therefore,

a customer is dissatisfied

I A) _

P(B

P(B3 )P(A

due to quality with

probability

is 0 . 4 8 4 8

I 83)

3
- P(B1 )P(A

I B 1 ) + P(B2 )P(A I B 2 ) + P(B3 )P(A I 8 3 )


0 . 1 5 x 0.5

= -------------

0 . 4 5 x 0 . 4 + 0 . 6 x 0 . 4 0 + 0. 5 x 0 . 1 5
0.075
=--

0.495
=0.1515

Therefore,

Example

a customer is dissatisfied

due to

packaging

with probability

is 0 . 1 5 1 5

20:

A salesperson

sells

four

products in 6 attempts.

products

in

6 attempts,

while the other

salesperson

sells three

Find the probability that:

No sale will take place when they

Either of them w i l l succeed

www.itmuniversityonline.org

both try to sell the product

in selling

the product

Page 226

Introduction to Q u a n t i t a t i v e T e c h n i q u e s

08.

eBook

Concept of P r o b a b i l i t y

Solution

20:

Let A be the event that the first salesperson


Let

A'

be the event that the first salesperson

Let B be the event that the second


Let

B'

P(A)

4
and P(B)

is a b l e to sell the product.

salesperson

is not able to sell the product.

6
4

P(A') = 1 -

2
and

=
6

The required

product.

=i

Therefore,

product.

is not a b l e to sell the

salesperson

be the event that the second

Given that,

1.

is a b l e to sell the

P(B') = 1 -

=
6

probabilities are calculated

i
6

as:

P(no sale w i l l take when they both try to sell the product)

= P(A' r; B ' )

P(A')P(B')

2
3
1
= - X - = 6
6
6

Therefore,

the

by multiplication

law

=0.167

probability

that

no

sale

will

take

place

when

they

both

try

to

sell

the

product is 0 . 1 6 7

2.

P(either of them w i l l succeed

in selling the product)

= P(AuB)

= P(A) + P(B) - P ( A " B),

by addition

law

= P(A) + P(B) - P(A)P(B)

:::: - + - - - x -

::;

- - - x -

---

5
6

Therefore,

the

probability

that

either

of

them

will

succeed

in

selling

the

product

0.8333

www.itmuniversityonline.org

Page 227

is

I n t r o d u c t i o n to Q u a n t i t a t i v e T e c h n i q u e s

08.

eBook

Concept of P r o b a b i l i t y

8.9

Applications of Probability

Applications of probability in various fields are as given below.

It

is

used

in

investment

companies

to

evaluate

prospects

of

stocks

of

various

companies.

In

trading

companies,

it

is

used

to

assess

the

prices of commodities

and

trade,

accordingly.

In b a n k i n g ,

it is used to develop methodology for assessing the creditworthiness of

an i n d i v i d u a l or a company.

A manufacturer or producer conducts market research to evaluate the current and


future demand for their product and services in the market.

An

operations

manager can

use

it

to

produce

better q u a l i t y

products

with

lesser

defects.

Doctors use it to study abnormality of a part or a system of a h u m a n being.

An

H R manager can

use

it

to

develop

a system

so

that

recruited

employees

will

happen to be more useful to the company.

Palmists and

astrologers

use

it to develop

t h e i r own

methodology

to evaluate the

chance of happenings of various events.

www.itmuniversityonline.org

Page 228

Introduction to Q u a n t i t a t i v e T e c h n i q u e s

08.

eBook

Concept of P r o b a b i l i t y

8 . 1 0 Chapter Summary

Probability

is

probability

of

happening

An

chance

the

The

whose

events

are

Events

are

said

said

event,

say

A,

to
is

an

1/2.

outcomes

will

event

Thus,

is

happen.

1/2

the total

cannot

be

and

It

the

is

assumed

probability

probability is

predicted

in

of

that
the

the
non

1.

advance

with

complete

to as a random experiment.
to

the possibility of the

of

of an event is also

experiment

something

happening

certainty is referred

that

be dependent

if the

happening

of any one

of them

affects

happening of another.

be

independent,

not

affected

by

if the

the

happening

happening

or

or

non-happening

non-happening

of any

of

the

one

other

event B.

The

probability of an event lies between zero and one,

including

The

topic

number

on

of

permutation

ways

in

and

which

combination

something

can

provides
be

done

both.

technique

or

counting

of

obtaining

the

number

a
of

possible outcomes of an experiment.

To

conduct

may

an

experiment

be dependent,

Union

If A and

B are

A and

B,

If A and
then

trial

and

its

result

is the

set of points

two events defines on

that

an

event.

belong

These

events

or impossible.

to any one of them

points.

the same sample

P(Au B) = P(A) + P(B)- P(A

B a r e two mutually exclusive events defined

P(An B) = 0 ,

is

mutually exclusive, exhaustive,

intersection is the set of common

law of probability states that

called

independent,

of two events,

or both, whereas,

is

space
r-,

5,

then

the addition

B).

on the same sample space 5,

P(A u B) = P(A) + P(B).

and

Joint probability of an event means the h a p p e n i n g

of both the events,

say, A and

together.

The

marginal

event,

of

an

event

regardless of the happening

Bayes' used
He

probability

assumed

calculate
industries,

prior and
that

posterior

posterior

on

the

is

probability

of

the

basis

happening

of

that

of the other event.

probabilities for the evaluation

probabilities.

of

some

It

has

social, engineering, medicine,

www.itmuniversityonline.org

the

additional

wide

prior

applications

of better solutions.

information,
in

various

you

can

fields,

like

etc.

Page 229

Probability Distribution:
Binomial

I T M

Introduction to Q u a n t i t a t i v e T e c h n i q u e s

09.

Probability

Distribution:

eBook

Binomial

9 . 1 Introduction
The

concept

Many
For

business

example,

amount
the

of probability

by

processes

the

call
loan

can
and

arriving

be

activities

at

the

department.

n u m b e r calls that

applied

per

Statisticians study
pattern

in

variation.

probability
outcome

in

the

P.M.

technicians are

This

In

chapter

the

process

required

will

such

probability

space

to 2 P . M .

time

interest at

which

might

loan

random

required
the

varies from

be

are

to

process.

in

nature.

sanction

loan

service center m i g h t
hour

to

processing

in the values of the variables and

statistics

sample

of

or

which

activity and

hour.

time,

be

Whereas,

which

varies

loan a m o u n t .

variation

distribution.

characteristics
between

such

center

hour,

business

outputs

variable of

for loan department the variable of interest


as per documentation and

analyze any

generate

service

The

have arrived

to

This

distribution

defined
like

pattern

how

by

variation

model

random

many

helps to make

of

calls

is

assigns

process.

will

arrive

find

out a systematic

represented
a

probability

This
at

helps

the

by
to

to

the
each

analyze

service

center

predictions or decide whether extra

service

in t h i s timeslot.

discuss

discrete

distribution

known

as

binomial

distribution.

also h i g h l i g h t the importance and applications of b i n o m i a l distribution.

After reading this chapter,

you

will

be able to:

Define a random variable and

probability function

Explain the meaning

Define the expectation and

Define binomial distribution

List the properties of b i n o m i a l distribution

State the applications of binomial distribution

Define mean and

Explain the fitting

of probability distribution
variance of a random

variable

variance of binomial distribution


of a binomial distribution

www. itmu niversityonl i ne.org

Page 2 3 1

It

will

I n t r o d u c t i o n to Q u a n t i t a t i v e T e c h n i q u e s

09.

Probability

Distribution:

eBook

Binomial

9 . 2 Basic Terminologies
A

probability

distribution

Frequency distribution
then

indicating

an

experiment

four.

To

by

listing

with

all

reference

is conducted

Outcome

Outcome

possible outcome.

with

a fair coin.

The coin

A probability

Tails in Tosses

Heads in Tosses

(T)

(H)

above

Outcomes of Tossing

table

the

is tossed

Number of

emphasize

distribution.

Number of

the

frequency

twice.

the

number

total
of

number
tails

and

of Two Coins Experiment

of outcomes
its

for

tossing

probability

the

of two

following

coins
table

computed.

Sample Points

Probability of
Number of

Number

Depending upon the

Number of
Sample

of Tails

Number of Tails

Tails
Points

(T)

(P(T))

(H,

= 0.25

HJ

4
1

2
(T,

HJ,

( H , T)

2
(T, T)

= 0 50

1
-=

0.25

Table 9 . 2 b :

Probability

www.itmuniversityonline.org

The

be written as:

Second Toss

in

to

possible outcomes of an experiment and

value or frequency of each

First Toss

Table 9.2a:

shown

understood

how outcomes may vary in a particular experiment.

results of the experiment can

As

be

prepared

the observed

distribution describes

For example,

is

can

Distribution of Number of Tails

Page 232

is
is

Introduction to Q u a n t i t a t i v e T e c h n i q u e s

09.

Probability

Distribution:

eBook

Binomial

The probability distribution of number of tails can

be represented

in a graphical form.

0.6

0.5

0 4

"

0.3

J'I

a:
0.2

0
0

2
Number of Tails

Fig. 9.2a:

Random

Probability

Distribution of Number of Tails

Variable

A random

variable

experiment.

It

can

"X"

means

take

any

that
of

the

real

number

various

is

associated

possible

values.

with
Each

the

outcome

one

has

of an

definite

probability.

For example,
dice,

then

equal

X is a random

a fair or unbiased
variable which

can

if X denotes a number obtained

take the values

variable X assumes only a finite

it is known as a discrete random

example,

1,

2,

3,

4,

5 or 6,

on the

each with

the

marks

obtained

by

Continuous

it

is

infinite

integer,

distinct

set of

students

in

an

examination

or

the

number

of

variables.

Random Variable

random

interval,

or countable

variable.

students in a college are discrete random

If the

dice,

Random Variable

random

values,

For

throwing

probability.

Discrete

If a

in

variable
said

height of people,

to

X can

be

assume an

continuous

infinite and

random

uncountable set

variable.

For

of values

example,

speed

in
of

electricity consumption, etc.

www. itmu niversityonl i ne.org

Page 233

some
a

car,

I n t r o d u c t i o n to Q u a n t i t a t i v e T e c h n i q u e s

09.

Probability

Distribution:

eBook

Binomial

9 . 3 P r o b a b i l i t y Functions
If a function of a random
probability

function.

Function and

variable computes the probability for an event

There

are

two

types

of

probability

it

is termed

as a

functions:

Discrete

Probability

is

discrete

probability

Continuous Probability Function.

Discrete Probability Function

The

probability

function

for

discrete

random

function since the domain of the function

called

is discrete.

Probability Mass Function (p.m.f.)

probability

function

which

gives the

random variable occurs is called


by:

f(x)

Also,

variable

f(x)

P(X

with

single

value of discrete

a probability mass function or p . m . f .

;;, O for all values of X and

It is denoted

p.m.f.

greater than x,

f(x),

or e . m . f .

which

written as:

1f(x)

(e.m.f.)

The cumulative mass function


F(x),

that any

; x)

Cumulative Mass Function

by

probability

F(x)

gives

P(X

of a discrete
the

random

probability

that

variable X is denoted

value

would

not

be

,; x)

Continuous Probability Function

The

probability

function

probability function,

for

continuous

random

since the domain of the function

Probability Density

For

continuous

variable

is

called

continuous

is continuous.

Function ( p . d . f . )

random

variable

probability density function.

It

the

is denoted

corresponding
by f(x) and

function

abbreviated

f(x)

is

called

as p . d . f .

Where, f(x)

2' 0 for all values of x and

f f(x);

1, a

Cumulative Density Function

(c.d.f.)

Cumulative

observed

value

variable X .
F(t)

P(X

density
of

function
X

is

not

It is denoted

of

continuous

greater

than

t,

by F(t), abbreviated

variable

where

is

"t"

as c . d . f.

is

and

the
any

probability
value

of

that

random

is written a s :

s t)
t

That

is:

F(t)

P(X s t)

f f(x)dx
-ro

www.itmuniversityonline.org

the

Page 234

Introduction to Q u a n t i t a t i v e T e c h n i q u e s

09.

Probability

Distribution:

eBook

Binomial

Note:

The concept of probability distribution


explains

how

Similarly,

the

total

probability

frequency

is an analogue to that of frequency distribution.

is distributed

distribution

explains

the various values that a random

among

how

different

the tota l

values of the

probability

is

It

variables.

distributed

among

variable can take.

9.4 Expectation a n d Variance


Managers
as:
a

often

have to

the expected

firm,

the

cement,

life of an

expected

item,

profit

refrigerator,

mathematically

take deci sions

etc.

credit cards held

1,

E(X)

...

by an

then

o r and

expected

or

the

values

expected
of

the expected

demand

random

of

variables

variable is

such

sales of

product

can

be

like

obtained

known.

Variable

per day, the

n u m b e r of finite values.

number of children

in a family,

For

instance,

the number of

i n d i v i d u a l , etc.

variable

the

values of variables,

life of a car battery,

variable can take only a limited

random

Pn,

company

Random

the number of u n i t s produced

the expected

if the probability distribution of the random

random

If a discrete

on

the expected

These

Expectation of a Discrete

A discrete

of

based

X takes the values

expected

is defined

as:

value

E(X) =

of this

L p,x,

,x

. . . Xn

discrete

with

random

respective

variable

probabilities

is

denoted

by

The expectation of a random

Variance of a Discrete

If a discrete

,p

...

p"'

random

and

variable represents the mean of the d i s t r i b u t i o n .

Random Variable

variable

X takes the values

the expectation

variance of variable X is given

Where,

E(X')

Lp x
1

of the discrete

by:

V(X)

E(X

,x

. . . Xn

random

2
)

[E(X)]

with

respective

variable X i s

E(X)

probabilities

or,

then the

12

For example,

when

occurring

the

on

a fair dice

top

probability equal to

face

of

is thrown;
the

dice.

suppose the variable


X

can

take

any

value

represents the

from

1,

... 6,

number
with

for all the possible outcomes.

www. itmu niversityonl i ne.org

Page 2 3 5

the

I n t r o d u c t i o n to Q u a n t i t a t i v e T e c h n i q u e s

09.

Probability

Therefore,

Distribution:

the expected

eBook

Binomial

value of X denoted

by

and

is given

by:

E(X) =

E(X)

LP;X;

= P1X1 + P 2 X 2 + p 3 X 3 . . . + p 5 X 5

= 5 ( 1 + 2 + 3 + 4 + 5 + 6 ) = 6 (21)

1=1

E(X) = 3 . 5

Also, variance is given

by:

V(X)

S o , to calculate V(X), first E(X2)

L P;X;2

E(X2) =

= P1 x2 + P 2 X 2

- [E(X)]2

is to be calculated.

+ p 3 X 2 . . . + p5x2

E(X2)

= !(12 + 22 + 32 + 42 + 52 + 5 2 )
6

15

1
91
= 6(1 + 4 + 9 + 16 + 25 + 36) =

6
= 15.167
Therefore,

V(X)

2
= E(X )

[E(X)]2 =

15.167

- (3.5)2 =

15.167

12.25

= 2.917

will

never

Note:
Even

though

However,

it

the
is

expected

located

at

value

the

situation where the average


1.25.

Obviously,

however,
children

there

centre

of

is

3.5,

the

the

be

family

variable

values

number of children

cannot

quite relevant.

of

from

in a certain

with

When this is m u l t i p l i e d

1.25
by

to

This

is

locality with

children

1000,

6.

take

but

it gives

the

this

similar

value.
to

the

1 0 0 0 families
number

1.25

is
is,

1 2 5 0 as the number of

in the entire locality.

Expectation of Continuous Random Variable


If X is a continuous
height

of an

random

individual,

variable is defined as:

Where, f(x)

is the

life

variable,
of

E(X) =

an

it

w i l l take all

electric

bulb,

etc.

possible values
The

in

expectation

its

range,

of the

like the

continuous

f xf(x)dx

probability function of the variable X.

Theorems on Expectations

E(X + Y) = E(X) + E ( Y ) , where, X and Y are two variables

E(X - Y) = E(X) - E(Y)

E(XY) = E(X) E(Y), when X and Y are two independent variables

E(cX)

= cE(X), where "c" is a constant

www.itmuniversityonline.org

Page 236

Introduction to Q u a n t i t a t i v e T e c h n i q u e s

09.

Probability

Distribution:

eBook

Binomial

Variance of a Continuous Random

Variable

The variance of a continuous random variable X i s given as:


V(X)

= E(X

Where,

f(x)

2
)

[E(X)]

E(X);

xf ( x ) d x and

E(X

2
);

f x'

f(x)dx

is the p . d . f of the variable X.

9.5

Binomial

Binomial
Bernoulli

Distribution

distribution
process

is

process is a random

The

Every trial

Each trial

The

James
from

is

also

known

mandatory

in

as

the

order

to

outcome
describe

of
a

Bernoulli
binomial

process.

Knowledge

distribution.

of

Bernoulli

process in which:

process is performed

for a finite number of times,

under the same conditions.

is independent of each other.


has two mutually exclusive possible outcomes.

probability of success remains constant for each trial.

Bernoulli

(1654-1705)

discovered

the

Bernoulli

process

and

hence

it

is

derived

his name.

Some examples of b i n o m i a l distribution are:

The tossing of a coin

Inspection of an

The

Shooting

results in either head

item determines whether it

result of an examination

be

discrete

independent trials.

Here,

if the

random

is either pass or fail.

Distribution

variable,

which

denotes

the

The possible values that X can take are:

probability

distribution with

of success

or

P(Success)

parameters "n" and "p" with

; x :::

0, 1, 2 . . .

, p + q ; l ,

;Q

is defective or non-defective.

a target with g u n results in either hit or miss.

Representation of B i n o m i a l

Let

or tail.

its

p,

p.m.f.

then

number

of

successes

is

said

to

follow

binomial

given as:

, otherwise

www. itmu niversityonl i ne.org

"n"

0, 1, 2 . . . n

o s e s

in

Page 237

I n t r o d u c t i o n to Q u a n t i t a t i v e T e c h n i q u e s

09.

Probability

If a

random

denoted

by

equal to

1.

Distribution:

variable
xB(n,

X follows

p).

eBook

Binomial

The

binomial

sum

of a l l

distribution
the

with

parameters

n and

p then

it

P(X = 0), P(X = 1) . . . P(X = n)

probabilities

is
is

It can be mathematically expressed as:

P(X = 0) + P(X = 1) + . . . + P(X = n) =

(n) p ' q n - x

X:::0

= 1

9 . 6 Properties a n d Applications of B i n o m i a l Distribution


Following

are some important properties of b i n o m i a l distribution:

Range of the variable X is from zero to n, where n is the number of trials

Mean=

Variance =

Shape of the distribution


p <

Following

np

npq, and

1/2 and

are

distribution

skewed

some

is used

is always

to

less t h a n mean

is symmetrical for p =

left when

examples

of

p >

binomial

1/2

(skewed

to the right when

1/2)

distribution

in

real

life

scenarios.

Binomial

to identify the number of:

Students remaining

Correct answers in a multiple choice test of n questions

Seeds germinated

Heads observed

N u m b e r of times the target is hit out of n attempts

Defective articles found

in

absent in a class of n students

in a row of n planted

seeds

n tosses of a coin

www.itmuniversityonline.org

in a lot containing

n articles

Page 238

Introduction to Q u a n t i t a t i v e T e c h n i q u e s

09.

Probability

Distribution:

eBook

Binomial

9 . 7 Mean a n d Variance of B i n o m i a l Distribution


Every

probability

mean

is

the

distribution

expected

variable are defined

E(X) =

L xP(x)

is

value

identified

of

the

by

its

variable.

parameters,

The

expected

or mean

and

values

E(X)

variance.
of

The

random

as:

L x (:J

p'qn-x

= np

The variance of variable X is defined


=

L (x

- mean)' P(x)

L (x

- n p ) ' ( : ) p'q"-'

E(X - mean)'

as:

= npq

Note:

For a binomial distribution with

Mean=

Variance = npq

Standard

parameters n and

p, where p + q =

1:

np

deviation

..jnpq

Some examples of b i n o m i a l distribution are:


Example 0 1 :

In

chemistry

laboratory

is

experiment,

successful.

probability that at

Solution

there

is

chance

If 7 experiments were

least one w i l l

that
done

6%
in

of the
the

experiment

laboratory,

then

done
what

be successful?

01:

Let X denotes the number of successful experiment out of the seven experiments.
Given that:

n = 7, p

= 0.06

then q

1 - p = 1 - 0 . 0 6 = 0.94

'

The distribution of X i s b i n o m i a l with

parameters n =

7 and

p = 0.06.

Therefore,

P[X = x] = ( : ) ( 0 . 0 6 ) ' ( 0 . 9 4 ) 7 - '

; x = 0 , 1 , 2

... 7

, p + q = l , O s p s l
= 0

www. itmu niversityonl i ne.org

; otherwise

Page 239

in

the

is the

I n t r o d u c t i o n to Q u a n t i t a t i v e T e c h n i q u e s

09.

Probability

Distribution:

eBook

Binomial

P(at least one experiment is successful)= P(X 2'. 1)

= 1-

P(X < 1)

1 - P[X

OJ
0

= 1-()co.06) co.94)7-

= 1-(0.94)7
= 0.3515
Therefore,

the

probability

that at

least one experiment out of seven

will

be successful

is

0.3515.

Example 0 2 :

The overall

percentage of personal

that out of a group of 8 borrowers,

loan

payers in

at least 6 w i l l

a bank is 60%.

What

is the

probability

repay the loan?

Solution O 2:

For

the

given

situation,

the

because there are only two


The binomial distribution

problem

can

possibilities:

is defined

be

solved

using

binomial

a borrower may or may

distribution.

This

is

not repay the loan.

as:

0-

P(X = x) = (:) p'q

Given that,

n = 8
x =

N u m b e r of payers among

the 8 borrowers

p = Probability of success that a borrower is a p a y e r =

0.6

q = Probability of failure that a borrower is a n o n - p a y e r =

The

probability

that at

least

6 out

of a group

of 8

1 - p = 1 - 0 . 6 = 0.4

borrowers

will

pay

the

loan

is

by:
p ( n u m b e r of pavers z 6) = P(X = 6) + P(X = 7) + P(X = 8)

P(X = 6) = (!Jro.6) (0.4)

28

0.2090

6
-

x 0.0467

x 0.16

P(X = 7) = (Jro.6)7(0.4)

8 x 0.0280 x 0.4 = 0.0897

www.itmuniversityonline.org

Page 240

given

Introduction to Q u a n t i t a t i v e T e c h n i q u e s

09.

Probability

Distribution:

P(X = 8) = (!)o.6)

(0.4)

1 x 0.0168

eBook

Binomial

8
-

= 0.0168

Thus,

P(X 6) = P(X = 6) + P(X = 7) + P(X = 8)

0.2090 + 0.0897 + 0.0168

= 0.3155

Therefore,

the

probability that at

on a particular day,

would

least

6 out of 8

repay their loan

borrowers,

who were

sanctioned

loans

is 0 . 3 1 5 5 .

Example 0 3 :

The

probability

that an

thrown 4 times,

number occurs when

Only twice

At least twice

of

parameters

Therefore,

are

0,

1,

n = 4 and

P[X

x]

2,

3,

4,

and

even

is

0.5.

If the

number occurs out of 4 throws.

p = 0.5.

dice

is

The

distribution

of

is

The possible

binomial

p = 0.5.

= (:)o.5)'(0.5)4'

P(even

is thrown

03:

Let X denotes the number of times an


values

a dice

what is the probability that even n u m b e r occurs:

Solution

even

; x

number occurs only twice)

www. itmu niversityonl i ne.org

= o, 1, 2, 3,

4.

; otherwise

P(X

2)

()o.5)2(o.5)4-'

6x0.25x0.25

0.375

Page 2 4 1

with

I n t r o d u c t i o n to Q u a n t i t a t i v e T e c h n i q u e s

09.

Probability

P(even

Distribution:

eBook

Binomial

number occurs at least twice)

P(X

2 2)

P(X 2 2) = 1 - P(X < 2)


= 1 - [P(X = 0) + P(X = 1)]

= 1 - [ ( ) o . 5 f (o

5)

1-[0.0625+0.25]

0.6875

+ ()o.5J(o.5)

1
-

9 . 8 Fitting a B i n o m i a l Distribution
If

the

observed

data

for

the

values

of

and

condition of applicability of b i n o m i a l distribution,


observed

data

an example.

by determining

For example,

the values of n,

its

frequency

distribution

a binomial distribution can

p, and

q.

The

procedure

satisfy

be fit to the

is explained

the frequencies for various values of the variable X, from

4 are:

X = x

Frequency

12

28

34

18

Table 9.Sa:

Frequency Distribution

If,x,
The mean of the data

is defined

as:

M e a n ::::
=

,
1

If,
1 ::: 0

www.itmuniversityonline.org

the

Page 242

with
O to

Introduction to Q u a n t i t a t i v e T e c h n i q u e s

09.

Probability

Distribution:

eBook

Binomial

The table for the calculation of values of "fx" is:

X = x

Frequency

12

28

28

34

68

18

54

32

Total

100

182

Table 9 . S b :

Therefore,

182)

mean=

fx

Frequency Distribution

= 1.82

( 1 )

..................

(2)

( 100
The mean of b i n o m i a l distribution

Now,

is:

Mean=

comparing the two equations ( 1 ) and

The maximum value of X is 4 therefore,


Therefore, 4p
This implies,

frequency

is

p = 0 . 4 5 and

the

q =

distribution,

particular value and

1.82

1 - 0.45 = 0.55

by

one

can

multiplying

the total frequency

the fitted

binomial distribution

is:

calculate
the

the

expected

probabilities

that

in the observed data.

frequencies.
variable

will

This is illustrated

Expected
assume
below:

is:

P(X = x) = [:) ( 0 . 4 5 ) ' ( 0 . 5 5 ) " -

The probabilities for

; x =

x = 0, 1, 2, 3, 4 can

P(X = 0) = f(O) = (}0.45)

; x = 0, 1, 2, 3, 4

obtained

The distribution

np

1. 82

[:) ( 0 . 4 5 ) ' ( 0 . 5 5 ) " -

fitting

( 2 ) , gives:

n = 4

Once the value of "n" and "p" are known,

After

np

1(0.55)

(0.55)

0, 1, 2, 3, 4

be calculated

as:

0
-

0.0915

www. itmu niversityonl i ne.org

Page 243

I n t r o d u c t i o n to Q u a n t i t a t i v e T e c h n i q u e s

09.

Probability

P(X

1)

f(l)

D is tri b u ti o n :

= (;)

= 4

eBook

Binomial

(0.45) (0.55)

x 0.45 x ( 0 . 5 5 )

1
-

= 0.2995

P(X

2)

f(2)

= (;)

(0.45) (0.55)

0.2025

_,

0.3025

= 0.3675

P(X

3)

f(3)

= ()

= 4

( 0 . 4 5 ) ( 0 . 5 5 ) ._

x 0.091125

x 0.55

= 0.2005

P(X

4)

f(4)

= (:)

Expected

(0.45) (0.55)

0.0410

frequencies are computed

in the following table.


Expected

Observed Frequency
X = x

P(X = x)

Frequency

f(x)

= N x P(X = x)
0

12

0.0915

9.15

28

0.2995

29.95

34

0.3675

36.75

18

0.2005

20.05

0.0410

4.10

100

Total

N =

100
.

Table 9.Bc:

Frequency D1stnbut1on

Note:

The measure of closeness between expected frequencies and the observed frequencies is
ca ll ed

a "goodness of fit."

www.itmuniversityonline.org

Page 244

Introduction to Q u a n t i t a t i v e T e c h n i q u e s

09.

Probability

Distribution:

eBook

Binomial

Example 04:

Five

coins

following

are

tossed

500

times

and

the

number

of

tails

is

recorded

as

given

in

the

table.

10

51

111

185

95

48

No. of Tails

Frequency
Table 9.Sd:

Frequency Distribution

Fit a binomial distribution to the given data.

Solution

Let

is

04:

the

number of tails.

Given,

500

and

5.

The following

table

is

used

compute mean of the given d i s t r i b u t i o n .


No. of Heads

Frequency

f(x)

10

51

51

111

222

185

555

95

380

48

240

500

1448

Total
Table 9 . S e :

The mean of a binomial d i s t r i b u t i o n =

xf(x)

Frequency

Distribution

np

1
5
:
4
0
8
Therefore,

np =

2.896

2896
This implies that,

0.5792

5
Therefore, q

1 - p = 0.4208

Once the values of "p" and "q" are obtained, the fitted

P(X = x)

f(X)

= [J

(0.5792)'(0.4208)'-; x

www. itmu niversityonl i ne.org

binomial distribution

is:

0, 1, 2, 3, 4, 5

Page 245

to

I n t r o d u c t i o n to Q u a n t i t a t i v e T e c h n i q u e s

09.

Probability

Distribution:

eBook

Binomial

The probabilities for x = 0, 1, 2, 3, 4, 5 can be calculated as follows:

P(X = 0) = f(O) = () (0.5792) (0.4208)'-

P(X

= 1) = f(l)

0.0131

= ()

(0.5792) (0.4208),_

= 0.0908

P(X = 2) = f(2) = () (0.5792) (0.4208)'-'

= 0.2499

P(X

= 3)

f(3)

= () (0.5792) (0.4208)'-

= 0.344

P(X

4)

f(4)

= (!)

(0.5792)'(0.4208)'-'

= 0.2367

P(X

5)

f(S)

= (:)

(0.5792)'(0.4208)

5
_,

= 0.0651

www.itmuniversityonline.org

Page 246

Introduction to Q u a n t i t a t i v e T e c h n i q u e s

09.

Probability

Now,

the

Distribution:

expected

frequencies

by the total frequency

(N

eBook

Binomial

obtained

= 500)

by

multiplying

are represented

each

of the

probabilities

in the following table.

Observed
X = x

above

Expected

Frequency

P(X = x)

Frequency

= N x P(X = x)

f(x)

10

0.0131

6.597

51

0.0908

45.4076

111

0.2499

124. 9839

185

0.344

172.0312

95

0. 2367

118.3941

48

0.0651

32.5921

Total

500

0.9996

500.059

"'.
Table 9.Sf:

"'
.

500

Frequency D1strobut1on

Example 0 5 :

For a binomial distribution,

Solution

m ea n =

4 and v a r i a n c e =

6.

Comment if it is appropriate.

05:

It is known that for a binomial distribution with parameters "n" and


Mean =

E(X)

"p":

= np

Variance = V(X)

npq

V(X)

= 6

Given that,
E(X)

= 4 and

Therefore,

q = npq
np

This is not

= = 1. 5 > 1

possible. Therefore, the given data

www. itmu niversityonl i ne.org

is not appropriate.

Page 247

I n t r o d u c t i o n to Q u a n t i t a t i v e T e c h n i q u e s

09.

Probability

Distribution:

eBook

Binomial

9 . 9 Chapter S u m m a r y

A random

variable X,

of an experiment.

it means that a real

It can

take any

number is associated

of the various

with the outcome

possible values.

Each

one

has a

definite probability.

If

random

variable

distinct set of values,

If the

random

assumes

only

finite

or

countable

variable X can assume an

infinite and

uncountable

to be a continuous random variable.

If

variable

discrete

integer

and

it is known as a discrete random variable.

some interval, it is said


a

infinite

random

probabilities

p1,p2 ... Pn,

X is denoted

by E(X) o r and

takes

the

then the expected

is defined as:

values

x 1 , x2 . . . x,

value of t h i s discrete

E(X)

set of values

I p,x,

with

in

respective

random

variable

If

discrete

probabilities

random

variable

p1,p2 ... Pn,

and

takes

the

the expectation

values

of the discrete

E(X) o r , then the variance of variable X is given by:

Where,

For a b i n o m i a l distribution with

random
E(X

2)
-

respective

variable X
[E(X)]

is

E(X ) = L P ; x ; 2

parameters n and

variance = npq, and standard deviation =

The

V(X) =

with

x 1 , x2 . . . Xn

measure

of

closeness

between

p, where p + q =

1, m e a n =

,/ n p q .

expected

frequencies

and

the

observed

frequencies is called a "goodness of fit."

www.itmuniversityonline.org

np,

Page 248

Probability Distribution:
Poisson a n d N o r m a l

I T M

I n t r o d u c t i o n to Q u a n t i t a t i v e T e c h n i q u e s

10.

Probability

Distribution:

Poisson

and

eBook

Normal

1 0 . 1 Introduction
The

previous chapter

importance
Poisson

distribution

also

and

used

distribution.
normal

its

discrete distribution,

This

chapter

importance

known as

introduces

and

binomial distribution,

another

applications.

For

discrete
rare

its

distribution,

events,

Poisson

is appropriate where a binomial distribution fails to estimate distribution.

chapter

widely

applications.

distribution,

distribution

This

and

introduced

introduces

its

importance

probability
Also,

as

Mathematician

and

in

concept
real

distribution,

the

distribution.

the

sample

This

life

as

size

Gauss

continuous

scenarios.

many

used

is

to

also

normal

distribution

Normal

natural

increases

distribution

physicist

of

distribution

phenomena
larger

known

distribution

value,

as

like
is

Normal

the

represent
it

tends

Gaussian

normal

to

this chapter,

you will

to evaluate the distribution

be able to:

Define Poisson distribution

Describe applications of Poisson distribution

Solve Poisson distribution examples

Define normal distribution

List the properties of normal distribution

Describe the various applications of normal distribution

Discuss graph of a normal distribution

Illustrate how to find

the area under the normal curve

www.itmuniversityonline.org

follow

distribution.

of errors in Astronomy.

After reading

most

Page 250

Introduction to Q u a n t i t a t i v e T e c h n i q u e s

10.

Probability

Distribution:

Poisson

and

eBook

Normal

1 0 . 2 Poisson Distribution
French

mathematician

Poisson

distribution.

situations

It

where the

the number of trials


of road

accidents,

reported

The

and
is

physicist,

distribution

probability
is very

the

Simeon
of

rare

of success

large.

It

Poisson

events.

( 1781-1840)

This

is

or occurrence of an

is used

n u m b e r of printing

Denis

because
event

to describe rare events,

mistakes

in a

developed

book or the

is

it

is

very

the

applied
small,

in
yet

such as the number


n u m b e r of suicides

in a particular city.

above

examples

have

one

common

element;

random variable that takes integer values,

that is,

they
0,

1,

can
2,

be

described

3, and

by

discrete

so o n .

Definition
A

random

variable

"X"

negative values and

is

said

to

follow

Poisson

its probability mass function

distribution

if

it

assumes

only

is given as:

e-;i..,._x
P(X = x) = f(x) =

; x =

o,

1, 2 . . . oo

x!

Where,
e =

Base of natural

logarithm,

its

>, =

Parameter of the distribution

X =

N u m b e r of occurrences

numerical value is 2. 7 1 8 2

Conditions for the applicability of Poisson d i s t r i b u t i o n :

The variable is a discrete variable.

The occurrences are random.

The occurrences are rare.

The

number of trials

is infinitely

www. itmu niversityonl i ne.org

large.

Page 2 5 1

non

I n t r o d u c t i o n to Q u a n t i t a t i v e T e c h n i q u e s

10.

Probability

Distribution:

Poisson

Characteristics of Poisson

The

occurrence

one event

eBook

Normal

Probability Distribution

The characteristics of a Poisson

and

of the

probability distribution are:

events

is

independent.

This

implies

that

the

occurrence

of

has no effect on the probability of the occurrence of another event.

An

infinite number of occurrences of the event are possible.

If an event occurs once in a given

interval,

then

its probability is proportional to

its

interval length.

In a very
event

small

length of interval,

Poisson and

Poisson distribution differs from


Poisson

time,

probability of two or more occurrences of an

is negligible.

Difference between

the

distribution

distance,

etc.

Distribution

B i n o m i a l distribution

operates

area,

Binomial

continuously

However,

in the following

over

binomial

the

given

distribution

ways:

occurrences,

operates

such

over

as

discrete

trials.

Binomial

Poisson

distribution

distribution

produces

produces

sequence

successes

of

successes

occurring

at

and

random

failures.

points

in

However,

specified

time.

Mean and Variance of Poisson

Distribution
00

The mean of a Poisson distribution is given

by:

Lxx

E(X) =

P(X = x)

x=O

E(X)

e->,.x

x!

x = 0

The variance of Poisson distribution

is defined as:

V(X)

E(X

- h)

V(X)

L (x
x=

V(X)

- 1-)

P(X

x)

(x - l.)

e-'i.'

x= o

x!

V(X) = 1-

Therefore, the mean and


Mean

Variance

variance for Poisson distribution

is the same.

www.itmuniversityonline.org

Page 252

Introduction to Q u a n t i t a t i v e T e c h n i q u e s

10.

In

Probability

addition,

Distribution:

the

standard

parameter, which
Therefore,

F o r >- =

i f >.

Poisson

deviation

and

is

eBook

Normal

cr =

,Ji .

Poisson

distribution

depends on

only

one

is >..

is known, a l l probabilities for the different values of X can

be calculated.

1:

P(X = 0) = e-'i!

= e-110

x!
1

= e-

O!

.!.

= 0.368,

Since, 1

= 1 and O! = 1

-l

11

1
1
= e-

P(X = 1) = e

x -

1!

1!

1
= - = 0.368

-1

= e

x -

21

= - x - = 0 . 1 8 4

Poisson

distribution

and "p" is s m a l l .

may

be

used

It is, therefore,

to approximate

regarded

as the limiting

1 0 . 3 A p p l i c a t i o n s of Poisson
Poisson

distribution

management science,

has
and

been

distribution

when

"n" is

large

case of binomial d i s t r i b u t i o n .

Distribution

extensively

operations

binomial

research.

used

in

various

business

applications,

Some applications of Poisson distribution

in real-life scenarios are:

The

demand

of

product

can

be

analyzed

with

the

help

of

Poisson

distribution

tool.

Printing errors that occur in the pages of a book can

One can compute the

The

One

number of visitors on a particular webpage in a day.

number of calls made to a customer-care department of a bank ( i n a specified

time,

be estimated.

such as 60 minutes) can


can

also

estimate

the

be calculated.

number

of customers

entering

shopping

centre

shop.

www. itmu niversityonl i ne.org

Page 2 5 3

or

I n t r o d u c t i o n to Q u a n t i t a t i v e T e c h n i q u e s

10.

Probability

Though

Poisson

to estimate
but

Distribution:

Poisson

distribution

and

eBook

Normal

has a theoretical

approach,

it

is widely

probabilities of uncertain events. The results may vary,

owing

to

less time consumed

in

the

usage of the

tool,

it

has

used
in

by

researchers

terms of accuracy,

extensive

benefits for

researchers.

Some Examples of Poisson

Distribution

Example 0 1 :

Consider

that

one-hour
received

telephone

period.

office

Calculate

the

received

on

probability

an

for

average

exactly

two

three

customer

customer

complaints

complaints

in

to

a
be

in the next hour.

Solution O 1 :

Given that, >, = 2 and e = 2. 7 1 8


e- =

(2. 7 1 8 ) - =

0.135
3

the value o f >, '

for x = 3,

f(x) =

= 2

= 8

e-'-,.'
x!

f(3)

(0.135)(8)

0.18

3!

Therefore,

there

is

an

18%

chance

that

three

customer

complaints

will

be

received

in

the next hour.

Example 0 2 :

In

bottle

defective.

manufacturing

Find

company,

the probability that

3%

of

the

bottles

in a sample of 60

produced

were

bottles

produced,

cannot

be

at

found

to

be

least three will

be defective.

Solution O 2:

Given that,

n = 60 and

Since,

30

;,

distribution

and

can

be

p = 3/o
s

0.05,

used

as

= 0.03
binomial

an

distribution

approximation

to

binomial

used.

However,

distribution

and

Poisson

its

becomes:

>, = np = 60

x 0.03

1.8

Let variable X denote the number of defective items


To find

out the probability of getting

www.itmuniversityonline.org

in a sample of 60 units.

at least three defectives

is given

by:

Page 254

mean

Introduction to Q u a n t i t a t i v e T e c h n i q u e s

10.

Probability

Distribution:

P(at least 3 defectives)

Here

follows

distribution

f(x) =

x = 0,

and

eBook

Normal

1 - P(at most 2 defectives)

1 - P(X

1 - [P(X

Poisson

is given

e-,: ,

Poisson

2)

= 0)

distribution

+ P(X = 1) + P(X = 2)]

with

parameter

>-

1.8.

The

p.m.f.

of

Poisson

below:

1, 2 . . .

oo

x!

Th at 1s,

f(

e-1.(1.8)'

O
x =

1
,

2
,

...

oo

x.1
Substituting
computed

the

values

as shown

of

O,

1,

and

2;

the

probabilities

for

O,

1,

and

can

be

below:

= e-18 x 1 = _
l
_

P(X = 0) = f(O) = e-1.s(l.8)

O!

e1.a

= 0.1652

18

eP(X

(1.8)

-1.B

= 1) = f(l) =

1.8

= 0.1652

x 1.8

11

= 0.2975

18

P(X

2)

f(

2)

e-

(1.8)

= 0.1652

2'

3.24

= 0.2678

Therefore, the required

probability,

P(at least 3 defectives)

1 - [P(X

1 - ( 0 . 1 6 5 2 + 0.2975 + 0.2678)

P ( X ;,

3)

= 0)

+ P(X

1)

+ P(X = 2)]

1 - 0.7306

= 0.2694

Example 0 3 :
The

four.

average

Find

number of customers

the probability that d u r i n g

arriving

counter

of a

retail

store

per

minute

a given m i n u t e :

No customers arrive

Three or more customers arrive

www. itmu niversityonl i ne.org

at

Page 2 5 5

is

I n t r o d u c t i o n to Q u a n t i t a t i v e T e c h n i q u e s

10.

Probability

Distribution:

Poisson

and

eBook

Normal

Solution O 3 :

To find

the required

Poisson distribution with rnean x = 4 can

probabilities,

be used.

e-)..1/
f(x) =

x =

o,

oo

1, 2 . . .

x!
P ( N o customers arrive)

f(O)

e-

= P(X

= O)

= 0. 0 18 3

P(Three or more customers arrive)

This

probability can

be obtained

0)

P(X

3) +

P(X

2:

3)

+ P(X

P(X

P(X

P(X

3)

+ P(X

1)

+ P(X

2)

4)

+ P(X

5) +

1 - [P(X

P(X

0)

+ P(X

0), P(X

4)

+ P(X

5)

+ ... ]

P ( X 2: 3)

+ P(Three or more customers arrive) = 1

1 - [P(X

Now evaluate,

as:

P(At the most two customers arrive)


P(X

[P(X

1) + P(X

1),

P(X

0)

+ P(X

1)

+ P(X

2)]

2)]

2)

0) = 0 . 0 1 8 3 2 has already been calculated

e-441
1)

f(l)

=
11

= 0.0183
=

x 4

0.0732

e-442
P(X

2)

f(2)

= 2!
=

0.0183

x 8

= 0.1465

So,

P(X

0)

Therefore,
P(X

3) +

+ P(X

1)

the required
P(X

4)

+ P(X

2)

0.0183 + 0.0732 + 0 . 1 4 6 5

0.2381

p r o b a b i l i t y is:

+ P(X

www.itmuniversityonline.org

5) + . . .

1 - 0. 2381

0. 761 9

Page 256

Introduction to Q u a n t i t a t i v e T e c h n i q u e s

10.

Probability

Distribution:

Poisson

and

eBook

Normal

1 0 . 4 N o r m a l Distribution
Until

now,

one

discussed.

This

within

given

of

the

topic

discrete

will

range

probability

elaborate

and

the

the

distributions,

cases

probability

in

which

distribution

Poisson

the

is

distribution,

variable

can

continuous,

take

that

is,

has
any

been
value

continuous

distribution.

Normal

distribution

is one

is a useful s a m p l i n g
practically
physical

and

distribution.

theoretically

sciences,

Moivre in

of the

and

most
It

important

has a u n i q u e

important.

many

allied

It

is

fields.

continuous
role

used

Normal

in

in

probability

distributions.

probability theory,
Economics,

distribution

as it

business,

was

is,

both,

social

discovered

It

by

and
De

1733.

Definition
A continuous
and

a2,

variable "X"

if its density function,

f(x)

random

is

said

to

have

normal

distribution

with

parameters

is given as:

--:A,(x-)'

, - oo <

2o

x <

oo, - co < < co,

cr

> 0

cr,12;
=

Where,
3.1415,

, otherwise

e and

1C are

two

irrational

numbers whose values are approximately 2. 7 1 8 2 and

respectively.

Remark:

Notation
normal

"X

distribution

variance of X,

If
The

0 and

a2

1,

--e

parameters

continuous

and

02,

then the normal variable

normal

function f(z)

f(z)

with

the

where

random

and

variable

a'

denote

follows

mean

variable

is

is represented

generally

is called
denoted

a standard
by

"Z"

normal

and

its

variable.

probability

as:

--2

2
2

co < z < co

,ff;;
=

and

respectively.

standard

density

ri')" denotes that

N(,

, otherwise

www. itmu niversityonl i ne.org

Page 2 5 7

I n t r o d u c t i o n to Q u a n t i t a t i v e T e c h n i q u e s

10.

Probability

Distribution:

Properties of Normal

Graphical

Poisson

and

eBook

Normal

Distribution

representation of normal

probability curve is given

in

Fig.

10.4a.

Mean, Median, Mode

Fig.

With the help of Fig.

The

curve

10.4a:

Probability Curve

the following characteristics can

10.4a,

has

Normal

single

bell

curve

line,

thus

it

is

be understood

unimodal.

The

as:

mean

lies

at

the

center of the curve.

The

normal

probability

and

mode lie in the center and

The

two

tails

of

the

curve

normal

is

symmetric.

are equal

Due

to

the

symmetry,

mean,

median,

in value.

distribution

extend

indefinitely

and

never

touch

the

horizontal axis.

The probability of total area under the normal curve is one.

If

and

is denoted

O and cr

Normal

= 1 , then the normal variable

is called

the standard

normal variable

by " Z . "

distribution

is an approximation

to

b i n o m i a l distribution

when

the value of

"n" becomes large.

Area u n d e r Normal Curve

Irrespective of the
1.

values o f and

Diagrammatic

10.4b.

If f'

is the

representation

mean

its symmetric nature,

68/o
the

values

in

lies

the total area

of the

of distribution

it can

of the data

2,

cr

area

and

the

between

normally

f'

cr and

distributed

under

cr is the

be conveniently stated

under the

normal

normal

standard

curve

deviation,

curve
is

is equal

shown

then

in

Fig.

because

of

that:

+ cr, that is, approximately 68% of all

population

lie

within

standard

deviation

from the mean.

www.itmuniversityonline.org

to

Page 258

Introduction to Q u a n t i t a t i v e T e c h n i q u e s

10.

Probability

Distribution:

Poisson

and

eBook

Normal

,+------99. 7% Data------.
e----95% Data-----..

-:\CT

-2CT

Fig.

- CT

10.4b:

+CT

the

values

in

normally

+::kr

Area under Normal Curve

95% of the data lies between f' - 2cr and f' + 2cr,

+2CT

distributed

population

that
lie

is,

approximately 9 5 % of all

within

standard

deviations

from the mean.

99. 7/o of the data

of all

values

in a

lies

between

3cr and

normally distributed

f'

+ 3cr, that is, approximately 99. 7%

population

lie within

standard

deviations

from the mean.

Relation
The

between

three

earlier,
zero

probability

when

and

Binomial,

"n"

"np"

is

is

Poisson, and

distributions

large

and

finite

the

are

Normal

very

closely

probability

constant,

then

Distribution
related

of occurrence

the

binomial

to

each

of an

other.

event

distribution

"p"

tends

As
is

stated

close

to

to

Poisson

distribution.

In a similar manner,

normal distribution

is a

limiting

form

of binomial distribution

under

the conditions that:

=, it is read as n tends to infinity or n steadily grows large

n ->

Neither p nor q

Thus,

it

can

be

is very small

stated

that

binomial

distribution

approaches

normal

distribution

with

X - np
standardized

normal

variable,

That

is,

here

follows

normal

distribution

npq
with mean O and

variance

1.

www. itmu niversityonl i ne.org

Page 2 5 9

I n t r o d u c t i o n to Q u a n t i t a t i v e T e c h n i q u e s

10.

Probability

Distribution:

Poisson

and

eBook

Normal

1 0 . 5 Applications of N o r m a l Distribution
Normal

distribution

decision m a k i n g

Normal

It

can

is

used

then

be

items under consideration.

To

estimate

tires, and

wide

by estimating

distribution

quantities.

has

the

range

of

applications.

It

is

to

study

used

to

many

such

generalize

cases

results

for

in

managerial

entire

with

population

large
of

the

Some a p p l i c a t i o n s of normal distribution are:

average

lifetime

of

necessary

goods,

such

as

bulbs,

batteries,

electronic items.

To estimate the height and

To calculate the weekly sales of a shop or sales of an

To calculate yield

To estimate the amount of cash withdrawal for a given

distribution
curve.

by students

weight of children

in an examination.

by a doctor, at the time of birth.


item

in the shop.

of a particular fertilizer used for crops.

1 0 . 6 Graph of N o r m a l

shaped

in

consideration

the

To compute the aggregate marks obtained

bell

used

the various aspects of business.

Normal

mainly

has

gained

period.

Distribution
importance

in

almost

every

field

of

research

due

to

its

The most important features of normal curve are:

f(x) =

(x-,,f

20
'

a.ff;.

f(x)

Fig.

The

normal

curve

takes

curve is symmetrical

single

point,

which

in

10.6a:

its

values

within

nature, that is,

divides the

www.itmuniversityonline.org

Normal

curve

Distribution

the

range of

the mean,

in

to two

-oo

mode,

eq u a l

to

and

parts,

co,

The

bell

shaped

m e d i a n coincide on a

with

respect

to,

Page 260

both,

Introduction to Q u a n t i t a t i v e T e c h n i q u e s

10.

Probability

shape

Distribution:

and

size.

The

which is equal to

Since the area

Poisson

total

and

area

under

u n d e r the

curve

is

1,

in the curve becoming flatter.

If X

N(,

2
),

the

curve

represents

the

total

probability,

1.

results

eBook

Normal

then

the

standard

then

for a given

normal

variable

value

is

of ,

an

defined

increase

as,

in a

Z = X -

CJ

N(O,

1).

Then the curve of the

standard

normal distribution

has the same shape as

a normal distribution.

f(z)

50%

50%

L:::::::::::__L_..:::::==--.z
Z = O

Fig.

1 0 . 7 How to

10.6b:

Standard

F i n d t h e Area

The area under a normal curve can

Normal

Distribution

u n d e r a N o r m a l Curve

be calculated

as shown

in

Example 04.

Example 04:

The

life

1500

of a

hrs and
What

bulb

Standard

is the

or equal to

What

is the

1470

hrs?

What

What

is

Deviation

probability
1470

by

that

the

the

normally

distributed

with

mean

of

hrs.

life of a

bulb

selected

at

random

will

be

less t h a n

that

the

life of a bulb

life of a

bulb

selected

selected

at

random

at

will

random

be more t h a n

will

be

less t h a n

hrs?

probability

t h a n or e q u a l to

of 30

is

hrs?

probability
1540

company

(S.D.)

p ro b a b i l i t y that the

is the

or equal to

manufactured

1540

www. itmu niversityonl i ne.org

that

the

life

of a

bulb

selected

at

random

will

be

greater

hrs?

Page 2 6 1

I n t r o d u c t i o n to Q u a n t i t a t i v e T e c h n i q u e s

10.

Probability

Distribution:

Poisson

and

eBook

Normal

Solution 0 4 :

Let X be the life of a b u l b .


hrs and

standard

The
to

deviation o = 30

probability

1470

hrs=

1470

-ro

It

is

that the

P(

-oo

1500

hrs.

life of a bulb

s X ,;

X follows normal distribution w i t h

1470).

selected

at

random will

Mathematically

it can

be

less than or equal

be written as:

-x-)2

In given example,

2a2

a..Jz;
difficult to

standard

solve the above

integral.

So

the

normal variable Z using the formula

normal

variable

is converted

to

Z = X -
CJ

.
Smee,

1470 - 1 5 0 0
=

1 5 0 0 and

5. D.

(o)

30 then,

Z =

= -1

30

the above integral becomes

,,-

_00

Since,

the

standard

value

of an

normal curve;

integral

between

_.!.zz

-1

Therefore,

"Y2n

two

limits

this probability is computed

is

equal

using

to

the

area

u n d e r the

normal probability table.

Note:

To

compute

standard

the

normal

probability from
=

1.0

interval

value

corresponding

in

probabilities

distribution
=

0 to any

the

first

table

standard

given

column

of

normal

value in the column 0 . 0 which

10.7a:

Z = O

Area under Standard

www.itmuniversityonline.org

in

positive value of Z.

Z = -1

Fig.

of

normal

the

variable

appendix.

To compute
probability

Z,

The

P(O

,;

table,

refer

to

table
Z s

then

1),

Z =

Normal

gives
see

see

is equal to 0 . 3 4 1 3 .

Distribution

Page 262

the

the

Introduction to Q u a n t i t a t i v e T e c h n i q u e s

10.

Probability

It

is

Distribution:

known

that

symmetrical.
1, and

the

total

Therefore,

-1)

-co

= P(l
=

P(O

= 0.5

and

area

the

from

is,

eBook

Normal

under the

area

is equal to 0 . 3 4 1 3 , that

Since the area from

P(-oo

Poisson

-1

normal
to

s Z s

P(O

1)

curve

is the

s Z s
- P(O

and

that

the curve

same as the area


s Z s 0)

= P(-1

to O is 0 . 5 , then the area from


$

is

from

is

Oto

= 0.3413

-co to - 1

is given as:

co)
co)

s Z

- P(O
$

s Z s

1)

1)

= 0. 5 - 0 . 3 4 1 3
=0.1587
Therefore,

the

random will

2.

The

probability

be up to

probability

1470

that

an

that

the

life

of

an

electronic

component

selected

at

hrs is 0 . 1 5 8 7 or 1 5 . 8 7 % .

electronic

component's

life

will

be

more

than

1470

hrs

is

life of an electronic component selected

at

given a s :

P(X

"

1470)

p ( X : f' " 147 -

p(X-1500

1470-1500)
2

30

30

-1)

= P(Z ;,

P(O

P(O

s Z s co)

= 0.5

Z $ co)

+ P(-1

Z $ 0)

+ P(O s Z s 1)

+ 0.3413

= 0.8413
T h i s is shown

in

Fig.

10.7b.

-;,Lf,1----f--P( Z

Z = - 1
Fig.

Now,

to find

random
Fig.

will

10.7b:

out the
be

iess

1)->.----

Z = O

Area under Standard

probability that the


man or equal to

> -

1540

Normal

Distribution

hrs, g r a p h i c a l l y ,

it

is

represented

10.7c.

www. itmu niversityonl i ne.org

Page 263

in the

I n t r o d u c t i o n to Q u a n t i t a t i v e T e c h n i q u e s

10.

Probability

Distribution:

Poisson

and

eBook

Normal

P( Z > 1.33)-+

+---r-P( Z s - 1.33)

fl=

1500

X =

Z = 0
Fig.

Here,

10.7c:

40

30
Therefore,
to

1.33

the

under

Z = 1.33

Area under Standard

1540 - 1500

1540

Normal

Distribution

1.33

30

required
the

probability

standard

is

the

normal

area

curve.

from
It

-oo

can

to

be

1540

hrs;

obtained

or

from

using

-oo

normal

probability table as follows:

P(X

1540)

P(X:

P(x

154

- J

-1500

,, 1540 - 1 5 0 0 J

30

30

P(Z s 1 . 3 3 )
P ( - oo ,;

= 0.5

Z ,;

0)

+ P ( O ,; Z ,;

+ P ( O ,; Z ,;

1.33)

1.33)

= 0. 5 + 0 . 4 0 8 2
= 0.9082

Since the area from


=

to

o. 9082.
1540

-oo t o o

is 0 . 5 , the total area from

This means that the p r o b a b i l i t y that the

hrs

is

0.9082

or

90.82%

of the

-oo to

1.33

is,

0.5

+ 0.4082

life of the component w i l l

components

will

have

life

up

to

be up
1540

hrs.

The

probability

calculated

P(X

;,,

that

the

life

of

the

components

will

be

more

than

1540

as:

1540)

1 - P(X

;,, 1540)

1 - 0.9082

= 0.0918

www.itmuniversityonline.org

Page 264

hrs

is

Introduction to Q u a n t i t a t i v e T e c h n i q u e s

10.

Probability

Distribution:

Poisson

and

eBook

Normal

Example 0 5 :

The

marks

scored

management
standard

by

the

institute's

deviation

students

MBA

10.

If

in

course

350

the

first

follows

students

semester

normal

appeared

accounts

distribution,

for

the

examination

with

mean

examination,

of

65

estimate

and
the

number of students scoring:

Less than 60 marks

More than 80 marks

Solution

05:

Assume that X represents the marks.


Given,
To find

65 and

<

P(X

S.D.

60)

and

(a)=

P(X

be used, which is given


Converting

Z = --=

that is,

From

P(-0.5

the

table,

0 . 1 9 1 5 and
P(O

s Z s

cr

10

X -

80-65

Z = -- =
cr

Since the curve

is

1. 5)

probabilities can

-5
= - = -0.5
10

10

P(O

for the area

from O t o

representation of these

10.7d.

15
= - = 1.5
10

symmetrical,

s Z $ 0)

the graphical

normal variable z, we g e t :

60-65

X -

For x = 80,

> 80),

in Fig.

x to standard

F o r x = 60,

10

1.5

the

probability

from

O to

-0. 5

is the

same as

O to

s Z $ 0.5)

u n d e r the

is 0 . 4 3 3 2 .

standard

That is,

P(O

normal curve, the area


$

Z $ 0.5)

from

Oto

0 . 1 9 1 5 and

0.4332

w w w . itmu niversityonl i ne.org

0. 5,

Page 2 6 5

0.5

is

I n t r o d u c t i o n to Q u a n t i t a t i v e T e c h n i q u e s

10.

Probability

Distribution:

Poisson a n d

eBook

Normal

--P(Z S -0.5)--,<---+I

X = 60

= 65

Z = -0.5

Z = 0

1+--__,..P(Z 2 1. SJ--+

= 65

Fig.

1.

P(X

:5

60)

p( X : ft s 60;

P(x - 65

10.7d:

Standard

X = 80
Z

Normal

1.5

Distribution

s 60 - 65)

10

10

= P(Z s -0.5)
=

2.

P(X

P(-oo :5 Z :5

-0.5)

= 0.5

- P ( - 0 . 5 :5 Z :5 OJ

= 0.5

- P(O :5 Z :5 0 . 5 )

0.5 - 0 . 1 9 1 5

0.3085

" 80)

P( X :

p(X-65
10

= P(Z

80
2

"
-

80-65)
10

1.5)

www.itmuniversityonline.org

Page 266

Introduction to Q u a n t i t a t i v e T e c h n i q u e s

10.

Probability

Distribution:

= 0.5

P(O s Z s

Poisson

and

eBook

Normal

1.5)

= 0. 5 - 0 . 4 3 3 2
= 0.0668

Therefore,

As

P(X

there are

less

than

number

60

of

< 60)

350

= 0 . 3 0 8 5 and

students,

marks

students

350

who

the
x

would

P(X

> 80) = 0 . 0 6 6 8

expected
0.3085
have

=
got

number of students

who

108 (approximately.)
more

than

80

marks

would

Similarly,
=

350

have

obtained

the

expected

0.0668

(approximately.)

www. itmu niversityonl i ne.org

Page 267

23

Potrebbero piacerti anche